Wikipedia:Auskunft/Archiv/2023/Woche 06

aus Wikipedia, der freien Enzyklopädie
Zur Navigation springen Zur Suche springen


Dies ist ein Archiv der Seite Wikipedia:Auskunft. Es enthält alle Abschnitte, die in der Kalender-Woche 06 im Jahr 2023 begonnen wurden.

Möchtest du in einer neuen Diskussion auf dieses Archiv verweisen? Nutze folgenden Link und ersetze ________ durch den Namen des Abschnittes. Die Nummer im Inhaltsverzeichnis gehört nicht dazu:

[[Wikipedia:Auskunft/Archiv/2023/Woche 06#________]]


← vorherige Woche Gesamtarchiv nächste Woche →

Befugnisse von Ordnern auf Demonstrationen

Gelegentlich sieht man auf AfD/Pegida/Querdenker etc. Demos Personen, die mit einem Kamerateam reden. Oft kommt dann ein Ordner herbeigeeilt, dem das ganze sichtlich peinlich ist und das Interview schnell abbrechen will. Nun meine Frage: Darf ein Ordner das?

--2003:D4:EF19:7B63:281A:1DE6:F769:4CB8 00:23, 6. Feb. 2023 (CET)

Ja

Erledigt|1=46.114.5.25 02:13, 6. Feb. 2023 (CET)

IMHO nicht erledigt. Ich halte den Bezug auf Hausrecht auch (sogar bei Versammlungen in geschlossenen Räumen) für falsch und irreführend. --88.72.111.240 03:51, 6. Feb. 2023 (CET)
Wie so oft kömmt es darauf an. In diesem Fall darauf, was „das Interview schnell abbrechen will“ praktisch bedeutet. Kamerateams nehmen in der Regel nicht an einer Demonstration teil. Sie sind also außerhalb des Demonstrationsgeschehens. Ein Gespräch mit wem auch immer am Rande einer Demonstration stört in keinem Fall die Ordnung und den Ablauf der Versammlung. Und Demonstranten haben das Recht, jederzeit eine Demonstration zu verlassen oder ihr beizutreten. Im öffentlichen Raum darf sich (ob Ordner oder nicht) allerdings auch jeder Mensch in ein Gespräch einmischen und seine Meinung sagen, aber nicht beleidigen, nötigen oder körperliche Gewalt anwenden.
Für Ordner und die Versammlungsleitung gilt (außer in Bayern, Berlin, Niedersachsen, Sachsen, Sachsen-Anhalt und Schleswig-Holstein) das Gesetz über Versammlungen und Aufzüge, auch: Versammlungsgesetz (VersG, Gesetzestext), ergänzt durch die Regelungen des jeweiligen Bundeslandes. Beachte insbes. die §§ 18, 19 und 24. Kerngedanken sind hier der „ordnungsmäßige Ablauf des Aufzuges“ und „die [zur] Aufrechterhaltung der Ordnung getroffenen Anordnungen des Leiters oder der von ihm bestellten Ordner“. Das Handeln eines Ordners in seiner Eigenschaft als Ordner ist bei dem Szenario der Fragestellung rechtsunwirksam, weil die Ordnung und der Ablauf der Versammlung durch ein Interview außerhalb des Aufzuges in der Regel nicht gestört werden. Außerhalb des Aufzuges hätte der Ordner dann auch bei einer von ihm festgestellten Störung die Polizei hinzuzuziehen.
Es gibt wie immer Grauzonen. Es ist z.B. ein Unterschied, ob ein Ordner zu dem Interviewten sagt: „Also mit denen würde ich nicht reden.“ oder ob er sagt: „Hör sofort auf mit denen zu quatschen.“ oder ob er sagt: „Wenn du hier weiterquasselst will ich dich auf der Demo nicht mehr sehen.“ (was bereits als Drohnung ausgelegt werden könnte, sehr wahrscheinlich aber keinen Bestand hätte). Es ist auch ein Unterschied, ob ein Ordner zu dem Kamerateam sagt: „Also das find ich echt doof, dass sie hier filmen.“ oder sagt: „Hör sofort auf, hier zu filmen.“ In jedem Fall spricht er hier aber (weil er dazu nicht berechtigt ist) nicht in seiner Eigenschaft als Ordner sondern als normaler Staatsbürger.
Es ist sicherlich auch die Frage, wie und ab wann ein Gespräch dauerhaft und nachhaltig durch Einmischung und Dazwischenreden gestört wird und wie das dann rechtlich einzuordnen ist (zumal, wenn möglicherweise emotionale Erregtheit für eine Bewertung mildernd angeführt werden könnte). Auch hier gibt es Grauzonen.
Etwas ausführlicher wird über die Rechte und Pflichten von Versammlungsleitung und vor allem auch von Ordnern hier informiert: Rechte und Pflichten der Versammlungsleitung (versammlungsrecht.org). --88.72.111.240 03:51, 6. Feb. 2023 (CET) Ergänzend: Zum Diskurs über den geplanten Straftatbestand Störung der Tätigkeit der Presse (siehe auch: Schutz der Pressefreiheit im Strafgesetzbuch, FAZ 24. April 2021, Braucht es einen Straftatbestand: „Störung der Pressetätigkeit“? dlf 1. Februar 2022, 4:32 Min.) finde ich keine weiteren aktuelleren Informationen. --88.72.111.240 04:50, 6. Feb. 2023 (CET)

Login schlägt immer wieder fehl, auch bei den Commons

Ich habe gestern die Passwörter geändert und jetzt Probleme. Woran kann es liegen? (--Christian der Vierte (Diskussion) 04:37, 6. Feb. 2023 (CET)) --Christian der Vierte (Diskussion) 04:37, 6. Feb. 2023 (CET)

Beim Ändern vertippt und deswegen schlägt es fehl? Vielleicht hattest du bei der Änderung die Feststelltaste aktiviert. Falls du meinst, das alles richtig eingegeben wird, dann vielleicht mal alle Cookies löschen. --Christian140 (Diskussion) 08:03, 6. Feb. 2023 (CET)
Möglicherweise enthält das neue Paßwort ein Sonderzeichen, das der verwendete Browser nicht korrekt überträgt, das aber bei der Änderung nicht bemängelt wird und auch bei der Eingabe nicht bemängelt wird, trotzdem aber nicht funktioniert. --92.73.148.178 10:23, 7. Feb. 2023 (CET)

ChatGPT schickt kein Email

Hallo liebe Wikipedianer, ich versuche seit Tagen, mich bei ChatGPT zuu registrieren, ich habe auch korrekt meine Emailadresse eingetippt. Dann sagt er mir, er schickt mir ein Email mit einem Bestätigungscode. Diese Email kommt aber nie. Meine Emailadresse habe ich korrekt eingegeben, das Email war auch nicht im Spam-Ordner. Der vergleichbare Vorgang hat bei WeTransfer in diesen Tagen mehrfach einwandfrei funktioniert. Hat jemand einen Tipp, was ich tun kann? Vielen Dank im voraus.--2003:DE:FF3A:A900:391F:7DD8:6CBD:3402 09:50, 6. Feb. 2023 (CET)

Andere E-Mailadresse verwenden? Ein Versuch wert. In der Zwischenzeit ohne Anmeldung kannst du YouChat benutzen. --KurtR (Diskussion) 10:08, 6. Feb. 2023 (CET)
Tja. So sieht das aus, wenn KI's die Weltherrschaft übernehmen. Erst verarschen sie dich beim registrieren, dann sperren sie dein Konto, und zum Schluß schicken sie dich in den Untergrund. Besorg dir am besten schon mal die Terminator-Baupläne! ;) -Ani--46.114.154.158 10:31, 6. Feb. 2023 (CET)
Kann sein, dass da irgendein schlecht eingestellter Spamfilter zu aggressiv löscht. Wenn du auf dein Email Konto mit verschiedenen Geräten zugreifst kann der Filter überall sein. Ich hatte mal eine Android App (aus dem Play Store mit tollen Features) mit meiner Arbeitsemail gelinkt, und erst nach Monaten gemerkt, dass sie ganz fleissig jede Menge echte emails als Spam "erkannt" und auch gleich gelöscht hat (ohne sich die Mühe mit einem Spam folder oder so zu machen). Die Optionen waren weit unten im Einstellungsmenü der App versteckt... 2A02:C7C:CAC2:2300:2054:5B61:B520:E2D 11:59, 6. Feb. 2023 (CET)
Der Filter kann auch bei einem der Email-Provider sein. Wenn irgendwer plötzlich große Mengen Emails verschickt, ist das ein Indiz für Spam, und wird dann möglicherweise von Deinem Provder geblockt. --Erastophanes (Diskussion) 13:38, 6. Feb. 2023 (CET)

Momentan ist diese Seite, speziell von Schülerinnen und Schülern in aller Welt, sehr beliebt. Kann sein, dass die Rechner schlicht in die Knie gegangen sind und mit dem Verkehr nicht mehr klarkommen. --Elrond (Diskussion) 11:34, 6. Feb. 2023 (CET)

+1. Unter der Woche ab ~ 22 Uhr kann man sich mit dem Bot ganz entspannt unterhalten. --95.116.105.100 13:36, 6. Feb. 2023 (CET)
Das widerspricht einer weltweiten Nutzung? --92.73.148.178 18:57, 6. Feb. 2023 (CET)
Nur wenn die Nutzungsdichte auf alle Zeitzonen homogen verteilt ist. --Kreuzschnabel 19:30, 6. Feb. 2023 (CET)
Hab deine Frage oben gerade bei ChatGPT eingegeben.
Antwort: Ich bin lediglich ein AI-Modell, das von OpenAI trainiert wurde, und habe keine Fähigkeiten, eine E-Mail-Registrierung zu überwachen oder zu verwalten. Es wäre am besten, wenn Sie den Support direkt kontaktieren, der Ihnen am besten helfen kann, das Problem zu lösen. Stellen Sie sicher, dass Sie alle relevanten Details bereitstellen, einschließlich der von Ihnen verwendeten E-Mail-Adresse und jeglicher Fehlermeldungen, die Sie erhalten haben.
Nachfrage: Was kann ich denn falsch gemacht haben, dass es mit der Registrierung nicht klappt?
Antwort: Es gibt eine Reihe von Gründen, warum eine Registrierung scheitern kann, wie zum Beispiel:
  • Falsche E-Mail-Adresse: Stellen Sie sicher, dass Sie die richtige E-Mail-Adresse angegeben haben.
  • Vollständiger Name: Manche Dienste erfordern, dass Sie Ihren vollständigen Namen angeben.
  • Unterstützte Länder: Es kann sein, dass der Dienst in Ihrem Land nicht verfügbar ist.
  • Netzwerkprobleme: Überprüfen Sie, ob Sie eine stabile Internetverbindung haben.
  • Kapazitätsprobleme: Es kann sein, dass der Dienst überlastet ist und nicht in der Lage ist, neue Anmeldungen zu verarbeiten.
Es wäre am besten, wenn Sie den Support direkt kontaktieren, um die genauen Gründe für das Scheitern Ihrer Registrierung zu erfahren.
Grüße, --2A02:3030:80E:481E:3DFD:E305:F14F:C803 23:01, 6. Feb. 2023 (CET)
Hab soaßeshalber weitergefragt:
Nachfrage: Wie würde ein Holzfäller das sagen?
Antwort: Ein Holzfäller würde es wahrscheinlich so ausdrücken:
"Schau mal, ich schwinge jeden Tag meine Axt und weiß, wie man Dinge erledigt, aber bei dieser Registrierungsgeschichte bin ich ein bisschen verloren. Ich hab' meine E-Mail-Adresse richtig eingegeben, aber keine Bestätigung bekommen. Vielleicht sollte ich mal den Support kontaktieren und schauen, ob die mir weiterhelfen können."
Nachfrage: Und wie würde ein Troll das erklären?
Antwort: Ein Troll würde es wahrscheinlich auf folgende Art und Weise erklären:
"Ha! Du hast dich bei ChatGPT registrieren wollen, aber es hat nicht funktioniert? LOOOL, was bist du denn für ein Versager! Du kannst nicht mal eine einfache Registrierung hinbekommen! Lass mich raten, hast du deine E-Mail-Adresse falsch eingegeben? Haha!"
--2A02:3030:80E:481E:3DFD:E305:F14F:C803 23:15, 6. Feb. 2023 (CET)

Der Bestätigungscode wird per SMS zugeschickt. Jedenfalls war das vor 1 Monat noch so.--Lectorium (Diskussion) 00:06, 7. Feb. 2023 (CET)

Hilfe mit QGIS

Moin, Ich hoffe wir haben hier wen, der mir helfen kann. Gerade wurde nach vierjähriger Arbeit das neue Kartenmaterial zu Grönland veröffentlicht. Ich versuche das bei QGIS reinzubekommen, aber habe nicht so viel Ahnung davon. Ich habe die frühere Karte runtergeladen gekriegt irgendwie, aber soweit ich verstehen kann, waren das Shapefiles und jetzt sind es .gpkg-Dateien. Ich weiß nicht recht, was ich machen muss, um die zu öffnen. Die Karte ist aufgeteilt in 216 verschiedene Quadranten, sprich ich habe einen Ordner mit 216 .gpkg-Dateien. Ich hätte gerne eine Karte, wo alle diese 216 Quadranten miteinander verbunden zu sehen sind. Was muss ich anklicken? Bitte eine Schritt-für-Schritt-Anleitung. Danke :) --Kenny McFly (Diskussion) 20:09, 6. Feb. 2023 (CET)

Wenn die Karte frei verfügbar ist, bitte mal einen Link angeben. --BlackEyedLion (Diskussion) 20:18, 6. Feb. 2023 (CET)
[1] Allerdings braucht man ein Benutzerkonto und der Download ist etwas kompliziert und zeitaufwendig. --Kenny McFly (Diskussion) 20:21, 6. Feb. 2023 (CET)
Wahrscheinlich ist sie in diese Anzahl von Quadraten (bzw. Rechtecken oder "Kacheln" - 'tiles') aufgeteilt, denn Quadranten kann es definitionsgemäß immer nur insgesamt vier geben. Soviel zum Thema "furniertes Wissen"... --95.116.92.158 02:46, 7. Feb. 2023 (CET)

Okay, ich habe rausgefunden, wie ich theoretisch alle 216 Quadranten öffnen kann (habe erstmal zwei genommen, weil ich für jeden einzeln bestätigen muss). Aber dann werden auch alle Layer auf den jeweiligen Quadranten einzeln angezeigt. Also man hat dann zum Beispiel 216 verschiedene See-Layer, was nicht so sinnvoll ist, wenn man sie umfärben will. Kann ich die verbinden? --Kenny McFly (Diskussion) 20:35, 6. Feb. 2023 (CET)

Laut en:GeoPackage ist gpkg ein SQLite-basiertes Format. Du müsstest also ein Tool bemühen, das Dir die 216 SQLite-Datenbanken in eine einzige zusammenführt. Vielleicht gibt es hier ja SQLite- oder gpkg-Fachmenschen, die wissen, wie das geht. --Rôtkæppchen₆₈ 00:44, 7. Feb. 2023 (CET)

Editor für Dos-Dateien

Suche einen Editor für alte Dos-Dateien, der mir die Umlaute nicht zerschießt. --2003:C1:970E:C500:CD6B:28AD:5C82:9AE6 08:41, 7. Feb. 2023 (CET)

Notepad++ kommt mit so gut wie allen Codierungen klar. --Magnus (Diskussion) 08:47, 7. Feb. 2023 (CET)
Unter Linux nimmst Du Geany dafür, unter Windows nimmst Du Notepad++. --92.73.148.178 08:49, 7. Feb. 2023 (CET)
Danke, funktioniert mit Einstellung OEM-US. Leider kann ich es anscheinend nicht so einstellen, daß existierende Dateien (einer bestimmten Endung, etwa .txt) immer in dieser Codierung geöffnet werden.--2003:C1:970E:C500:CD6B:28AD:5C82:9AE6 09:22, 7. Feb. 2023 (CET)

Fotografie(?)-Zeitschrift gesucht

Kann jemand anhand Stil, Aufmachung und Format (22x28 cm) eingrenzen, aus welcher Zeitschrift der Artikel in dieser Ebay-Anzeige stammt: 1?. Unser Artikel zu Anton Sahm bedarf einer Erweiterung, an der ich gerade sitze, und ggf. wäre der Artikel eine gute Quelle (aber nicht für den Preis, und nicht, wenn die Herkunft nicht klar ist). Der Anzeigenersteller meint, Autor und Zeitschrift seien im Artikel erkennbar, verrät beides aber logischerweise nicht. Von der Aufmachung her mit der VÖ von großformatigen Bildern auf hochwertigem Papier würde ich auf eine Zeitschrift tippen, die sich an ein fotografieaffines Publikum richtet, evtl. auch an ein allgemein kunstinteressiertes Publikum, wenn auch kein Fachpublikum. Das Format schließt Westermanns Monatshefte aus, die kleiner waren. Da es sich anscheinend um einen Artikel aus Anlass des 70. Geburtstages (August 1961) handelt, würde auch das Foto-Magazin entfallen, die einen anderen Artikel gebracht haben (Altmeister des Porträt-Fotos: Anton Sahm 70 Jahre). Anscheinend ist es eine Zeitschrift mit durchgängiger Zählung der Seiten, S. 224 bei einer VÖ im Sommer 1961 könnte ein monatliches Erscheindatum bedeuten (Umfang ca. 30 Seiten pro Ausgabe). --Paulae 21:19, 6. Feb. 2023 (CET)

(quetsch, und kein Ergebnis:) Nur dass so dünne Hefte, 32 Seiten wären ja drucküblich, eher unwahrscheinlich sind. Ausgaben 1961 von Du und von Magnum – beide kämen eigentlich in Betracht, auch vom Layout her – haben beide um die 100 Seiten pro Heft und, nebenbei, paginieren auch nicht den Jahrgang durch. Merian machte so etwas Aktuelles eher nicht, Velhagen & Klasings Monatshefte waren 1961 bereits eingestellt. Westermanns mag nicht nur vom Format her nicht hinkommen, bei ebay gibts bei manchen Angeboten auch Scans der Inhaltsangaben, und im August 1961 ist Essig mit Sahm, von der unpassenden Pagina abgesehen. Ebenso Das Kunstwerk mit Register im Netz. Die Wochenhefte des Stern hatten übrigens zu der Zeit einen Umfang von 80 bis 100 Seiten; doppelt dicke Sommerhefte gab es damals m. W. noch nicht. Ein Zweimonatsheft? Eine Reproduktion aus einem später erschienenen Band kann es wegen der Seitenverweise ebenfalls nicht sein. Daher bietet die Seitenzahl sicher den Schlüssel. Aber wo das Schloss ist, weiß ich leider auch nicht, nur, wo es nicht ist. --Aalfons (Diskussion) 22:57, 7. Feb. 2023 (CET)

Stadtarchiv München fragen --Historiograf (Diskussion) 19:48, 7. Feb. 2023 (CET)

Warum wird mein Beitrag zur Honorarordnung HOAI nicht beachtet?

man lese https://www.haufe.de/recht/weitere-rechtsgebiete/wirtschaftsrecht/der-architekten-und-ingenieurvertrag-im-neuen-bauvertragsrecht_210_445034.html

--Werner2022 (Diskussion) 10:24, 7. Feb. 2023 (CET)

Da bist du hier vollkommen falsch. Fragen zu einem Artikel gehören auf die Diskussionsseite des Artikels; hier werden nur allgemeine Wissensfragen beantwortet. --Jossi (Diskussion) 11:23, 7. Feb. 2023 (CET)
(BK)Weil die meisten hier die HOAI nicht interessiert, darum achte ich jedenfalls nicht darauf. Ok, Scherz beiseite, Deine letzte Änderung ist im Artikel und wurde nicht zurückgesetzt. Wenn Du noch mehr im Artikel ändern oder wieder einbringen möchtest, solltest Du es mit der Artikel-Disk. versuchen. Nicht zuletzt ist das hier die Auskunft, Fragen zur Wikipedia sollten bei Wikipedia:Fragen zur Wikipedia gestellt werden. Gruß, --G-41614 (Diskussion) 11:28, 7. Feb. 2023 (CET)
Deine Änderung war deswegen nicht sofort sichtbar, weil du kein Sichter bist. --77.10.81.27 12:35, 7. Feb. 2023 (CET)

Suche ein "skurriles Museum", in dem ich vor ca. 25 Jahren mal war, kann mich aber kaum an etwas erinnern.

Das private "Museum" war wohl aus einem Projekt eines eigenwilligen Künstlers entstanden. Es bestand aus einem Außenbereich (Ausstellungsgelände) und mindestens zwei Gebäuden. Es lag am Rande eines kleines Ortes, ich meine in Niedersachsen(?). Auf dem Außengelände waren Kunstobjekte ausgestellt, u.a. aus Metall (?). In dem Haupthaus, ggf. ein "Forsthaus" oder Bauernhaus (?), waren u.a. auch Bilder von Arcimboldo aufgehängt (Daran erinnere ich mich sicher). Daneben gab es ein Gebäude(?) mit einem großen bunten (blauen?) grottenähnlichen Raum in Eingangsnähe, oder so ähnlich. So weit meine dunklen verschwommenen Erinnerungen. Habe das Museum im Netz oder bei Wikipedia innerhalb 15 Minuten nicht gefunden. --Doc Schneyder Disk. 17:02, 7. Feb. 2023 (CET)

Du meinst jetzt aber nicht die Künstlerkolonie Worpswede? Dort ist ja alles mögliche in verschiedenen Häusern versammelt, und das fällt mir spontan zu Niedersachsen ein. Mehr Chancen hättest Du, wenn es in Italien gewesen wäre: Einer der zahlreichen Skulpturengärten in Italien in der Nachfolge von Bomarzo (Parco dei Mostri) käme in Frage. Schau' mal durch, was unsere Kategorie dazu hergibt. --92.73.148.178 17:25, 7. Feb. 2023 (CET)
Nachtrag: Hier noch etwas, das ungefähr auf die Beschreibung passen könnte, und auch in Niedersachsen liegt: Kunst-Landschaft Springhornhof. Schau Dir also auch mal die dazu gehörende Kategorie durch. --92.73.148.178 17:30, 7. Feb. 2023 (CET)
Suchst Du Iserhatsche bei Bispingen? Skurril trifft es jedenfalls.--Mhunk (Diskussion) 18:05, 7. Feb. 2023 (CET) Ergänzung: Hier gibt es eine Fotogalerie mit Arcimbolo (Nachbildung), blauer "Grotte" und Skulpturen.--Mhunk (Diskussion) 18:14, 7. Feb. 2023 (CET)
Volltreffer, versenkt, vielen Dank. Hat bei Wikipedia auch ein Lemma: Iserhatsche, ist aber nicht in Museumslisten aufgeführt. Wie sehr Erinnerungen verschwimmen können, ist schon irre. Das Forsthaus war die Jagdvilla. Und der "grottenähnliche Raum" war der Hochzeitsaal im Kellergewölbe des Phantasieschlosses Montagnetto. Das intensive "Blau" der Säulen dort war im Gedächtnis hängengeblieben. --Doc Schneyder Disk. 18:26, 7. Feb. 2023 (CET)
Das kenn ich auch, hab es anhand Deiner Beschreibung aber nicht erkannt. Hättest Du was von der größten Bierflaschensammlung der Welt erzählt, wär es mir eingefallen... --2A02:3030:811:50E8:A1F1:8710:8536:3376 18:54, 7. Feb. 2023 (CET)
Ich meine, die Bierflaschensammlung gab es damals noch nicht --Doc Schneyder Disk. 21:52, 7. Feb. 2023 (CET)
🤦--2A02:3030:81A:554F:991D:BB83:6D4F:9914 22:50, 7. Feb. 2023 (CET)

Jakob Freud und seine ersten beiden Frauen

Habe gerade den Artikel über Sigmund Freud gelesen und bei seiner Abstammung gesehen, dass sein Vater, Jakob Freud, insgesamt dreimal verheiratet war. Es geht aus dem Artikel aber nicht hervor, ob die beiden vorherigen Ehen von Jakob Freud geschieden worden waren, oder die Frauen gestorben sind. Das sollte vielleicht im Artikel kurz erwähnt werden. Oder gab es damals überhaupt noch keine Scheidung? Helgomar --178.191.193.231 23:32, 7. Feb. 2023 (CET)

„Freuds Vater Kallamon Jacob (1815-1896) war 1840 nach Freiberg gekommen. Er stammte aus dem galizischen Städtchen Tismenitz (jetzt Tysmenica). Sechzehnjährig hatte er in Tismenitz Sally Kanner geheiratet. Zwei Söhne entsprossen dieser Ehe: Emanuel (1834-1915) und Philipp (1838-1912); beide folgten dem Vater nach Freiberg. 1852 starb Jacob Freuds Frau. Über seine zweite Frau ist nur bekannt, daß sie Rebekka hieß. Sie muß bald nach der Eheschließung gestorben sein, denn Jacob Freud heiratete 1855 in Wien Amalie Nathanson (1835-1930), Sigmund Freuds Mutter.“

Kurt R. Eissler: Eine biographische Skizze. In: Sigmund Freud. Sein Leben in Bildern und Texten. Suhrkamp Verlag, Frankfurt 1976, S. 10–39, hier S. 10–11.
--Jossi (Diskussion) 01:02, 8. Feb. 2023 (CET)

Wie heißen diese Bällchen aus Pflanzenresten etc...?

Mir fehlt gerade ein Wort. Wie nennt man die Bällchen aus Pflanzenfasern etc., die in entsprechend flachen Gegenden der Wind vor sich hertreibt? In Filmen kommen sie vor, wenn Einöde und Trostlosigkeit symbolisiert werden sollen, aus eigener Erfahrung kenne ich sie von süditalienischen Stränden. --Established 1620 (Diskussion) 12:47, 9. Feb. 2023 (CET)

Z.B. Tumbleweed. --92.116.122.26 12:50, 9. Feb. 2023 (CET)
Danke. Das führte mich zu Chamaechorie. Es macht wohl keinen Unterschied, ob die Dinger aus Vermehrungsstrategiegründen rollen oder ob es sich eher (meinem Eindruck nach) um "Abfallprodukte" handelt, die zwar auch kugelförmig zusammengeballt sind, aber sich wahrscheinlich gar nicht mehr vermehren können? --Established 1620 (Diskussion) 12:55, 9. Feb. 2023 (CET)
Chamaechorie wär´s nur, wenn es auch der Samenverbreitung dienlich ist. Die im Artikel genannten deutschen Namen sollten aber immer verwendbar sein.--Meloe (Diskussion) 14:09, 9. Feb. 2023 (CET)
Tumbleweed Wollmaus--2003:DE:6F2E:A3C2:E8E0:7BD6:CC96:DBEC 12:59, 9. Feb. 2023 (CET) PS: äolische Agglomeration ;)
Wenn es um die filzigen Dinger am Meeresstrand geht, siehe auch Seeball. --Fice (Diskussion) 14:18, 9. Feb. 2023 (CET)
Ich kenn das als Rose von Jericho. --46.79.54.74 16:10, 9. Feb. 2023 (CET)
Ruthenisches Salzkraut und da Ruthenisches Salzkraut#Symbolik. --Optimum (Diskussion) 16:24, 9. Feb. 2023 (CET)
Ah, das Salzkraut kannte ich nicht. Danke! Die Wollmäuse sind ja indoor und passen nicht, aber ansonsten bin ich jetzt wirklich einen Schritt weiter.
Archivierung dieses Abschnittes wurde gewünscht von: Established 1620 (Diskussion) 16:55, 10. Feb. 2023 (CET)

Kino des Zorns?

Filme lassen sich, grob gesprochen, danach einteilen, welche Gefühle sie auslösen: Manche lassen die Zuschauer lachen, manche machen ihnen Angst, manche erregen sie sexuell, manche machen traurig, andere glücklich (spätestens beim Happy End). Alle Gefühle werden bedient, außer Zorn und Wut. Gibt es keine Filme, die aggressiv machen, und wenn ja, warum ist das so? --Φ (Diskussion) 15:33, 7. Feb. 2023 (CET)

Ich werde bei amerikanischem Sprechgesang und albernen Gesten und Klamotten aggressiv. Davon gibt's reichlich in Filmen. Ralf Roletschek (Diskussion) 15:39, 7. Feb. 2023 (CET)
Ja, aber die zielen doch nicht absichtlich darauf, dieses Gefühl bei dir hervorzurufen. --Φ (Diskussion) 15:44, 7. Feb. 2023 (CET)
Propaganda und AgitProp sind Bereiche, die ganz gezielt mit dieser Emotion arbeiten. Jud Süß sollte zum Beispiel ganz bewusst zornig machen.--91.221.58.20 15:46, 7. Feb. 2023 (CET)
Wie Jud Süß diesbezüglich funktionieren soll, verstehe ich eigentlich nicht: Die Geschichte geht doch (mal unabhängig von historischen Tatsachen) "gut" aus: der Schurke wird aufgehängt! Klimax, Katharsis, Erleichterung: Angenehmen Heimweg! (Der Film soll ja wohl auch nicht so gut angekommen sein.) --77.10.104.207 12:21, 8. Feb. 2023 (CET)
Z könnte in das Genre passen. --Schlesinger schreib! 15:47, 7. Feb. 2023 (CET)
Politfilme sollen zornig machen. Danke, das ergibt Sinn. --Φ (Diskussion) 15:50, 7. Feb. 2023 (CET)
Eigentlich alle Tragödien machen einen (auch) zornig, da es ja meist einen Verursacher des schlechten Ausgangs gibt. Das ist Z ebenso wie Das Leben ist schön. (nicht signierter Beitrag von Hachinger62 (Diskussion | Beiträge) 16:06, 7. Feb. 2023 (CET))
Nein: In der Trägodie wird der Protagonist schuldlos schuldig, er ist Opfer einer Verstrickung oder des Schicksals. Auf Ödipus oder Hamlet wird der Zuschauer nicht zornig. Die Tragöddie zielt vielmehr auf Schrecken, Furcht, nach Lessing auch auf Mitleid. --Φ (Diskussion) 16:19, 7. Feb. 2023 (CET)
Dazu würde noch Zug des Lebens als Tragikomödie gehören. --Schlesinger schreib! 16:15, 7. Feb. 2023 (CET)
Wieder mal unter ferner liefen: der Dokumentarfilm. Mich hat z.B. Workingman’s Death nicht nur emotional berührt sondern (auch) in gewisser Weise wütend gemacht. --88.72.111.240 16:44, 7. Feb. 2023 (CET)
Aggressivität ist keine Emotion, sondern ein Verhalten. Emotionen wie Wut/Zorn können aber Auslöser dafür sein, müssen aber nicht. Vielleicht siehst Du die falschen Filme bzw. bist in Deiner Filmauswahl einfach zu sehr eingeschränkt. Es gibt jede Menge Filme, die mich am Ende wütend, fassungslos, z. T. zornig zurücklassen: Sophie Scholl – Die letzten Tage, Schrei nach Freiheit bzw. wo kleinere (Werk ohne Autor) oder große Teile des Films (Schindlers Liste, Das Leben der Anderen) solche Emotionen hervorrufen. Sie haben gemeinsam, dass sie wahre Geschichten erzählen bzw. reale Umstände beschreiben. Auslöser für die Wut ist jeweils Fassungslosigkeit über die barbarische und unmenschliche Art und Weise, wie Menschen miteinander umgehen, wie sich bestimmte Gruppen über andere stellen, sie erniedrigen, quälen, töten, sich in ihrer Verblendetheit als Elite sehen und dies weitestgehend ungestraft bleibt. Bei den genannten Filmen handelt es sich natürlich um "unsere" eigene Geschichte. Ein anderer Film der mir einfällt ist Systemsprenger, wo - im Gegensatz zu den zuvor aufgezählten - ganz aktuelle reale Umstände beschrieben werden und Kinder leiden müssen und offenbar keine Chance besteht, dass sich daran mittelfristig etwas ändert, obwohl es viele gibt, die sich für sie einsetzen, letztendlich aber oft scheitern und dabei selbst daran buchstäblich zusammenbrechen. Na und dann wären da natürlich Dokumentationen z. B. über Massentierhaltung, "Umerziehungslager", US-Waffengesetz, Nordkorea, soziale Ungerechtigkeiten usw. --Blutgretchen (Diskussion) 21:02, 7. Feb. 2023 (CET)

Etymologie Jangeli

Hallo, die Schmalspurbahnen in Luxemburg hatten wohl den Spitznamen Jangeli bzw. Jhangeli (außer die Strecke nach Echternach, die Charly hieß). Charly lässt sich ja noch irgendwie herleiten, das wird wohl ein Name sein. Aber woher bitte kommt Jangeli? Ist das luxemburgisch, was ich nicht interpretieren kann? Oder irgendwie lautmalerisch? --2A02:8071:2286:4C80:8DA1:B739:31DD:9200 23:33, 7. Feb. 2023 (CET)

Ein Jangeli ist ein kleiner Jang, d. h. Jean; laut diesem Artikel benannt nach lb:Jean Knaff. Grüße Dumbox (Diskussion) (ohne (gültigen) Zeitstempel signierter Beitrag von Dumbox (Diskussion | Beiträge) 23:41, 7. Feb. 2023 (CET))
Siehe auch Schängel#Begriff. --jergen ? 18:33, 8. Feb. 2023 (CET)

CO aus Gastherme?

In einem Einsatzbericht der Feuerwehr war zu lesen, daß in einem Objekt die CO-Warnmelder angesprochen hätten. Ursache wäre eine defekte Gastherme gewesen. Da es nun weit und breit kein Stadtgas (Kokereigas) mehr gibt - in der betreffenden Stadt im Ruhrgebiet auch nicht - frage ich mich, wie das sein kann: wenn Gas, egal ob Flaschengas oder Erdgas aus der Leitung, an die Luft kommt, dann brennt es, oder es brennt eben nicht. CO entsteht aber in keinem Fall. Was könnte da vorgefallen sein? --77.10.104.207 12:10, 8. Feb. 2023 (CET)

Kohlenstoffmonoxid#Vorkommen_in_Gebäuden --2A02:3030:803:5EF2:693A:8FE1:AD15:76B5 12:21, 8. Feb. 2023 (CET)
Es entsteht also angeblich bei der Verbrennung. Wie, steht da aber auch nicht. (Bei Zigaretten und Holzkohle kann ich mir das ja noch vorstellen, aber bei Gasflammen?) --77.10.104.207 12:26, 8. Feb. 2023 (CET)
Das Stichwort ist "unvollständige Verbrennung", s.a. Kohlenstoffmonoxidintoxikation#Quellen von Kohlenmonoxid, wo als Grund geringe Sauerstoffzufuhr genannt wird. 62.157.7.177 12:39, 8. Feb. 2023 (CET)
Und wie hat man sich "geringe Sauerstoffzufuhr" bei Gasflammen vorzustellen? --77.10.104.207 13:35, 8. Feb. 2023 (CET)
Unterstöchiometrische Sauerstoffzufuhr, bei Erdgas etwa . --Rôtkæppchen₆₈ 16:14, 8. Feb. 2023 (CET)
Wenn eine Gastherme richtig eingestellt ist, entsteht auch nur sehr wenig CO, allerdings kann es durch allerlei Fehler passieren, dass zuviel Gas, bzw. zu wenig Luft eingeführt wird, oder die Lambdasonde falsch misst und falsche Signale sendet und man wieder beim falschen Mischverhältnis landet. Daher kommt auch einmal im Jahr der schwarze Mann und misst die Abluftzusammensetzung, um ggf. solche Sachen zu entdecken. Das sollte auch bei der (leider nicht vorgeschriebenen) jährlichen Inspektion entdeckt werden, aber die ist ja, wie schon geschrieben keine Pflicht und wird von sparsamen Menschen gerne nicht durchgeführt (ist doch alles nur Geldschneiderei!). Eine Beerdigung ist allerdings deutlich teurer, kann aber ggf. von der Steuer abgesetzt werden. (Wer Ironie findet, darf sie gerne behalten). --Elrond (Diskussion) 14:25, 8. Feb. 2023 (CET)
Vielleicht habe ich ja die falschen Vorstellungen, aber was gibt es da denn einzustellen? Das Gas kommt aus Düsen, vermischt sich mit der stets im Überfluß vorhandenen Luft und verbrennt. Für eine unterstöchiometrische Verbrennung müßte zuviel Gas mit zu wenig Luft gemischt werden - ist das den überhaupt vorstellbar bzw. wie kann das in der Praxis vorkommen? --77.10.104.207 16:30, 8. Feb. 2023 (CET)
Wie es passierte weiß ich nicht, aber in dem Haus, in dem mein Ex-Schwiegervater einst lebte, ist ein junges Paar im Badezimmer an einer CO-Vergiftung erstickt, als sie gemeinsam unter der Dusche standen. Der Durchlauferhitzer hing im Badezimmer, eine nicht unübliche Installation, auch heute noch. --Elrond (Diskussion) 17:25, 8. Feb. 2023 (CET)
Den ist vorstellbar und passiert auch immer mal wieder. --31.212.72.57 16:56, 8. Feb. 2023 (CET)
Hier werden 7 mögliche Gründe für CO-Vergiftungen durch Gasthermen genannt: https://www.thermondo.de/info/rat/gas/kohlenmonoxid-heizung/ --Doc Schneyder Disk. 17:04, 8. Feb. 2023 (CET)
Ich bin überascht dass CO nicht schwerer als Luft ist. Ich dachte das wabert zuerst am Boden rum. Hat man nicht deswegen in Bergwerken auch Grubenlampen oder Vögel nahe am Boden getragen? --2001:9E8:A52E:4300:20B9:3FB9:68FF:879D 18:03, 8. Feb. 2023 (CET)
CO ist sogar geringfügig leichter als Luft (1,25 kg/m³ gegenüber 1,29 kg/m³ bei 0°C, Wert für trockene Luft, feuchte Luft ist etwas leichter). Der Dichteunterschied ist zu gering und die Bewegung der Luft in einer Wohnung groß genug, dass sich alles gleichmäßig vermischt. --Blutgretchen (Diskussion) 18:24, 8. Feb. 2023 (CET)
Das schwerere Gas, das sich am Boden sammeln kann, ist Kohlendioxid. 62.157.7.177 18:29, 8. Feb. 2023 (CET)
Und die Sicherheitsgrubenlampen und die Kanarienvögel sind für die matten bzw. schlagenden Wetter, also das Grubengas (=Methan), das sich im Bergwerk ansammeln kann. (Zu den Zahlen: durchschnittliches Molekulargewicht der atmosphärischen Luft ist irgendwas zwischen 28 und 29 - N2 hat 28 - Wasserdampf hat 18, CO 28, also praktisch identisch mit Stickstoff, O2 32 und CO2 wie Ar 44.) --77.10.104.207 20:19, 8. Feb. 2023 (CET)
Na gut, wenn das Abgas wegen Kaminverstopfung nicht abziehen kann, dann strömt auch nicht genug Luft in die Brennkammer nach, und die ist dann eben nicht im Überschuß vorhanden, und es kann tatsächlich zu einer unterstöchiometrischen Verbrennung kommen. Jetzt glaube ich es auch. (Die Gasdurchlauferhitzer, die ich alle putzmunter überlebt habe, wurden noch mit Stadtgas betrieben - die brauchten gar kein CO selbst zu produzieren, die bekamen das gleich aus der Leitung geliefert. Habe nie gehört, daß damit je etwas passiert wäre - gegen Gasaustritt gab es die Sicherheitsflamme mit dem Bimetallschalter, der die Gaszufuhr sperrte, wenn die Flamme ausging, und das wäre sie auch bei einem Abgasrückstau. Eigentlich waren die ziemlich idiotensicher konstruiert. Ok, da kam schonmal jedes Jahr der Schornsteinfeger und steckte oben in das Ofenrohr einen Fühler zum Messen rein. War aber nie was und eigentlich auch Unsinn: Störungen hätte man eigentlich immer sofort am veränderten Flammbild (gelbliche Flammen) und Brennergeräusch erkannt.) --77.10.104.207 20:47, 8. Feb. 2023 (CET)
Es hat auch schon Vogelnester in Abgasrohren gegeben, sodass das kohlenstoffmonoxidhaltige Abgas in die Wohnung statt ins Freie gezogen ist. Dadurch dass Kohlenstoffmonoxid dem Stickstoff isoelektronisch ist und auch fast dieselbe Dichte hat, verbreitet es sich wie Luft in der Wohnung. Geruchlos ist es obendrein. --Rôtkæppchen₆₈ 22:06, 8. Feb. 2023 (CET)

Natürliche Experimente

Ich verstehe das Konzept des natürlichen Experimentes nicht. Es scheint mir irgendwie problematisch zu sein. Wir können die Bedingungen dieser natürlichen Experimente nicht kontrollieren und deshalb erwünschte Seiteneffekte nicht ausschließen.
Selbst wenn wir einzelne andere Einflussgrößen kennen und statistisch herausrechnen können, gibt es doch immer noch ein Problem. Wir können unbekannte Einflussgrößen nicht sinnvoll ausschließen.--2A02:908:424:9D60:F92B:358A:70EA:76F 13:32, 8. Feb. 2023 (CET)

Problematisch ist die irreführende Bezeichnung als Experiment, denn es ist kein Experiment in dem Sinne, daß man Einflußgrößen verändert und die Effekte beobachtet. Der Artikel wird im Artikelnamensraum auch nur viermal verlinkt.
Das Problem unbekannter Einflußgrößen unterscheidet sich doch nicht grundsätzlich von Untersuchungen, in denen die untersuchte Einflußgröße gezielt variiert wird. Dort kann man vielleicht einzelne, vermutete Einflußgrößen durch die Versuchsanordnung ausschalten. In beiden Fällen kann man weitere Einflußgrößen vielleicht durch Befragung ermitteln, es können in beiden Fällen aber auch Einflußgrößen unerkannt bleiben. 62.157.7.177 14:30, 8. Feb. 2023 (CET)
Es kann natürlich sein, dass man bestimmte Einflussgrößen wie Luftdruck oder Schwerkraft gar nicht bemerkt. Mit der selben Berechtigung könnte man spekulieren, dass es innerhalb des Sonnensystems oder des Universums gewisse Einflussfaktoren gibt. --2A02:908:424:9D60:1432:241F:1AC6:7F9D 17:40, 8. Feb. 2023 (CET)

Artikelsichtung

Ich habe am 1. Januar einen Artikel für Wikipedia verfasst, der als Artikelentwurf unter meinen Benutzerbeiträge steht, aber bisher noch nicht gesichtet worden ist. Was muss ich tun, damit er gesichtet und veröffentlicht werden kann? --Tekakwitha (Diskussion) 15:57, 8. Feb. 2023 (CET)

Hinweis: Für Fragen speziell zur Wikipedia gibt es ein eigenes Diskussionsforum: Fragen zur Wikipedia.

Missverständnis. Verschieben in den Artikelnamensraum musst du ihn schon selbst, siehe Hilfe:Seite verschieben. Dort kann er dann gesichtet werden. In deinem Benutzernamensraum wird nichts gesichtet. Dein Artikel (ich nehme an, es geht um Benutzer:Tekakwitha/Artikelentwurf) enthält noch einige formale Fehler, etwa Überschriftsebenen; das ist schnell repariert. Schlimmer ist das Fehlen unabhängiger Belege. Schon die erste Aussage „führender Experte“ schreit geradezu nach einem Beleg. Behaupten kann das jeder. Wenn er führend ist, müssen unabhängige Fachmagazine oder so doch Bezug auf ihn nehmen. Das wäre ein gültiger Beleg. --Kreuzschnabel 16:07, 8. Feb. 2023 (CET)

Weltkarte mit Commons Bildern

Gibt es eine Webanwendung, mit der man auf einer Weltkarte die ganzen Fotos von Commons (welche Geotags haben) findet? --NichtA11w1ss3ndDiskussion 22:09, 8. Feb. 2023 (CET)

https://wikimap.toolforge.org/ --Rôtkæppchen₆₈ 22:18, 8. Feb. 2023 (CET)
Kuss --NichtA11w1ss3ndDiskussion 22:35, 8. Feb. 2023 (CET)

Ich suche eine Bank in den USA,,,Standard Treuhandbank

Ich suche eine Bank in den USA,,,Standard Treuhandbank

--78.48.134.72 14:53, 8. Feb. 2023 (CET)

[2] (die US-Amerikaner werden wohl kaum das Wort Treuhand benutzen, oder?) (Dieser Beitrag wurde nicht mit ChatGPT formuliert.) --88.72.111.240 15:48, 8. Feb. 2023 (CET)
"Welches Bank-Schweinderl (Standard Piggy Bank) hätten'S denn gerne?"
Hier eine Liste mit Orten. Dann im Detail ansehen, was sie an Diensten bieten. Nga Ahorangi (Diskussion) 18:18, 8. Feb. 2023 (CET)
Jetzt werde ich neugierig. Das Wort "Standard" kommt ja in der Liste nicht vor. In welchem Kontext verstehst du dieses Wort in der Fragestellung? (möglichst mit einer Übersetzung des dazugeörigen deutschen Begriffs) (Dieser Beitrag wurde nicht mit ChatGPT formuliert.) --88.72.111.240 18:57, 8. Feb. 2023 (CET)
Treuhand ist ja ein deutscher Begriff, der keine "direkte" Entsprechung im Amerikanischen hat. "Standard" (oben) verstehe ich als: "[etwa] so wie".
Es wäre einfacher, wenn der Frager erläutern würde, worum es geht. Aber: Hey! Wer sagt denn, dass das Leben einfach sein sollte !?! Nga Ahorangi (Diskussion) 10:12, 9. Feb. 2023 (CET)
Ich würde es genau anders interpretieren: „Standard“ ist in den USA als Namensbestandteil oder Eigenname bei Unternehmen und gerade Banken, Versicherungen usw. weit verbreitet. „Treuhandbank“ kann entweder generische Bezeichnung sein (‚Ich suche eine amerikanische Treuhandbank mit dem Namen Standard.‘) oder aber sogar – wie von 88.72.111.240 interpretiert – ein Übersetzungsversuch. Bis weit in das 20. Jahrhundert wäre das nicht unüblich, dass deutschsprachige Berichte amerikanische Unternehmensnamen eindeutschen. Dann wäre so etwas wie „Standard Trust Bank“ oder so durchaus naheligend. (Es geht hier nicht darum, ob das das passende Analogon für die deutsche Konzeption einer Treuhandbank ist, sondern ob man es, gerade ohne tiefere Fachkenntnis, versuchsweise so übersetzen könnte.) Beides hilft aber vermutlich ohne weitere Angaben nicht, denn Standard als Name ist extrem häufig.--91.221.58.22 11:12, 9. Feb. 2023 (CET)

Schlacht um die Krim

Der Artikel Schlacht um die Krim ist etwas verwirrend. Sehe ich das richtig dass es in dieser Schlacht darum geht, dass die Ukraine (bzw UdSSR) die Krim von Deutschland zurückerobert? Wann hat denn Deutschland die Krim erobert? Irgend wie fehlt diese Info im Abschnitt "Vorgeschichte". --2001:9E8:A52E:4300:20B9:3FB9:68FF:879D 23:32, 8. Feb. 2023 (CET)

Siehe Krim#Zweiter Weltkrieg, Vertreibung und Deportation. --Rôtkæppchen₆₈ 23:54, 8. Feb. 2023 (CET)
Zu den ideologischen, geostrategischen und rohstoffbezogenen Zielen und der militärstrategischen Bedeutung der Krim: Im Artikel Zweiter Weltkrieg: Der Überfall auf die Sowjetunion begann am 22. Juni 1941. Die Heeresgruppe Süd (Gerd von Rundstedt) sollte die Ukraine erobern. „Das Hauptziel der NS-Führung bestand darin, sich die Ölvorräte des Kaukasus und das Getreide in der Ukraine zu sichern. Dies würde Deutschland nach Hitlers Überzeugung unbesiegbar machen.“ (Quellenverweis: Antony Beevor: Der Zweite Weltkrieg. München 2014, S. 219., zit.n. Zweiter Weltkrieg) Im gleichen Artikel: „Tatsächlich war der Überfall auf die Sowjetunion im Wesentlichen ein ideologisch verbrämter Eroberungs- und Vernichtungskrieg mit dem von Hitler bereits Jahre zuvor formulierten Ziel der Gewinnung von „Lebensraum im Osten“. Damit war „ein blockadefestes Großimperium“ bis zum Ural und über den Kaukasus hinaus gemeint.“
Hinsichtlich der Kontrolle des Schwarzen Meeres im Artikel 17. Armee (Wehrmacht): „Die (sowjetische) Schwarzmeerflotte bekam mit dieser Operation (d.i. die Rückeroberung der Krim) ihre Hauptbasis Sewastopol zurück.“
Und weiterhin im Artikel Zweiter Weltkrieg: „Hitlers „Weisung Nr. 41“ vom 5. April 1942 („Fall Blau“) legte für die Sommeroffensive fest, dass zunächst Stalingrad an der Wolga, danach der Kaukasusraum bis zur türkischen und iranischen Grenze erobert werden sollte, um die dortigen Erdölzentren in die Hand zu bekommen. (...) Von Mai bis Juli wurden mit der Schlacht um Sewastopol und dem Unternehmen Trappenjagd Gebiete auf der Krim erobert, die als Sprungbrett für eine Offensive bis zum Kaukasus dienen sollten.“
Im Artikel Kuban-Brückenkopf: „ (...) war eine deutsche Auffangstellung auf der Taman-Halbinsel in Südrussland, die von Januar bis Oktober 1943 bestand. Nach dem Rückzug der deutschen Truppen aus dem Kaukasus war die Stellung ursprünglich am Unterlauf des Kuban gehalten worden, um einen erneuten Angriff auf die Ölquellen des Kaukasus zu ermöglichen. Nach dem allgemeinen Rückzug des deutschen Ostheeres auf die Panther-Wotan-Linie und im Zuge der sowjetischen Noworossijsk-Tamaner Operation wurden die im Brückenkopf befindlichen deutschen Truppen im Unternehmen Krimhild erfolgreich über die Straße von Kertsch auf die Krim evakuiert.“
Alle Behauptungen aus de.wp, die hier referiert wurden, setzen bei Weiterverwendung eine Prüung der für die Sachverhalte jeweils angegebenen Quellen voraus. (Dieser Beitrag wurde nicht mit ChatGPT formuliert.) --88.72.111.240 08:43, 9. Feb. 2023 (CET)

Fuß abgetrennt, draussen in der Natur, Opfer hat medizinische Kenntnisse: Überlebbar?

Zum Beispiel gestürzt beim überqueren von Bahngleisen, in Folge bewußtlos, ein Unterschenkel liegt quer über einer Schiene ... --46.114.2.85 18:45, 11. Feb. 2023 (CET)

Wenn das Opfer nach der Amputation bewusstlos ist, sehe ich einen schnellen Tod durch Blutverlust als wahrscheinlich an. Wenn es bei Bewusstein ist und etwas hat, mit dem es die Blutung stoppen kann (Tourniquet oder etwas improvisiertes) und das hinbekommt bevor der Blutverlust kritisch wird, kann es Überleben. Das ist aber von mehreren Faktoren abhängig. --SNAFU @@@ 19:35, 11. Feb. 2023 (CET)
Ist eine andere Person anwesend, die die Blutung auch bei evtl. einsetzender Ohnmacht stoppen kann? Ist nur der Unfall in der Natur oder ist es notwendig, dass das Ganze auch in der Natur heilt? Ist das Ganze in der Wildnis oder kann ein Rettungshubschrauber in kurzer Zeit eintreffen? Das beeinflusst die Überlebbarkeit dann doch erheblich. Bei entsprechend guter schneller Versorgung kann man vielleicht den Fuß sogar noch wieder halbwegs passabel retten und annähen. -- Jonathan 20:35, 11. Feb. 2023 (CET)
Vielen Dank für die umfassende Antwort. Die Situation war so gedacht, das zunächst keine Hilfe herbeigerufen werden kann, das Opfer also über Tage auf sich allein gestellt bleibt (aber unmittelbar nach dem Abtrennen des Fußes zu sich kommt und Selbsthilfe leisten kann).--46.114.2.85 20:47, 11. Feb. 2023 (CET)

Ob jemand an einer traumatischen Unterschenkelamputation "verblutet", kommt sehr auf den Unfallmechanismus an. Zudem: Bei US-Abtrennung auf Bahngleisen kommt es sehr auf körperliche Fitness an. Ein gesunder 16-jähriger Organismus hat natürlich ganz andere Kompensationsmöglichkeiten eine akute Massivblutung zu kompensieren als ein 60-jähriger Organismus.--Doc Schneyder Disk. 21:46, 11. Feb. 2023 (CET)

Aron Ralston hat "draußen in der Natur" seine Hand verloren (und zwar indem er sie selbst amputiert hat!) und überlebt. Wenn "draußen in der Natur" aber eine Bahnlinie verläuft, gibt es da vielleicht auch noch andere Infrastruktur. Wie ist der Verunglückte denn überhaupt auf die Schienen gelangt? Vielleicht über eine Landstraße? --Geoz (Diskussion) 22:14, 11. Feb. 2023 (CET)
Ja, ist durchaus überlebbar. Wenn jemand es schaft, sich mit einem abgerissenen Arm selber ins Spital einzuliefern und dafür 20 Kilometer Auto zu fahren Link zum profil.at Artkel. Dann ist es für jemand mit entsprechenden medizinischen Kenntissen auch möglich, während dieser Zeit -wo der Körper das Selbstschutzprogramm aufrecht erhalten kann- sich entsprechend zu versorgen, dass er danach nicht verblutet. Allerdings reicht das „Verbluten verhindern“ alleine zum Überleben nicht aus. Denn damit jemand mit so einer Verletzung ohne fremde Hilfe mehrer Tage überleben kann. Dafür muss auch noch einiges anderes stimmen, auch mental. Es gibt Menschen die haben einen starken Überlebenswillen um so etwas überleben zu können, die meisten werden unter den gefragten Umständen aber schlichtweg versterben.--Bobo11 (Diskussion) 23:00, 11. Feb. 2023 (CET)
Danke für den Aron Ralston-Link, die ähnliche Verletzung hilft, eine realistische Vorstellung möglicher Folgen zu kriegen. - Das Bahngleis war lediglich konstruiert. --46.114.2.85 23:07, 11. Feb. 2023 (CET)
Archivierung dieses Abschnittes wurde gewünscht von: 46.114.2.85 23:07, 11. Feb. 2023 (CET)

Gauß'sches Wegesystem ? Was ist das?

Die (von der Frage nach dem skurrilen Museum angeregte) Lektüre unseres Lemmas Iserhatsche bringt mich zu der Frage, was der dort im Abschnitt »Jagdvilla« schon von Anfang an enthaltene Begriff „Gauß'sches Wegesystem” bedeuten soll. Weder im Lemma Gauß noch sonst im Internet scheint dieser Begriff mit irgend einer Bedeutung verknüpft zu existieren. Außer in unserem Lemma über die Iserhatsche scheint es ihn nicht zu geben. Was also ist damit konkret gemeint? --92.73.132.89 07:31, 8. Feb. 2023 (CET)

Vielleicht das Gauß-Krüger-Koordinatensystem? --2A02:3030:814:B9EC:F8BF:58FD:57B1:1A3 08:15, 8. Feb. 2023 (CET)
Boah, was für eine Antiquität, aus 2005 (der Artikel). Nur ein Beleg (die eigene HP!). Und der Text, der bei uns seit der ersten Version unverändert ist, ist identisch zum Text auf der HP. Der erste Snapshot im Internet-Archiv, den ich gefunden habe, ist von 2007, und hat den identischen Text (hab nur das Jagdschloss verglichen). Also klar eine URV, es ist mir nur nicht klar, wer von wem abgeschrieben hat. --Erastophanes (Diskussion) 08:18, 8. Feb. 2023 (CET)
Die Diskussionsseite des damaligen Wikipedia-Autoren besteht nur aus Urheberrechts/Lizenzanfragen, also dürfte das ein relativ klarer Fall sein. -- 79.91.113.116 09:32, 8. Feb. 2023 (CET)
Und zum Wegesystem: Ich vermute eine Erfindung des Autors. Im Zusammenhang mit Geld und Gauß fällt mir am ehesten die Darstellung auf der Rückseite des 10-D-Mark-Scheins ein. Das ist aber ein Triangulationsnetz, kein Wegesystem. Hat irgendwer den Teppich schon mal gesehen? 62.157.7.177 09:47, 8. Feb. 2023 (CET)
Zu sehen ist er ausschnittsweise hier, ab ca. 4:08 --TheRunnerUp 10:47, 8. Feb. 2023 (CET)
Es dürfte sich tatsächlich um die Gaußsche Landesaufnahme handeln. Im Zentrum des Filmes ist Wilsede mit dem Wappen von Bispingen zu sehen. Von dort gehen sieben Strahlen aus, wie bei der Gaußsche Landesaufnahme.--Mhunk (Diskussion) 11:04, 8. Feb. 2023 (CET)
Ja, der Punkt Wilsede des Netzes gehört zu Bispingen und zwischen der 2. und 5. Richtung (gezählt von Süden im Uhrzeigersinn) gibt es eine Verbindungslinie, die auch auf dem Teppich dargestellt ist (der größte Ausschnitt wird bei 4:15 - 4:16 gezeigt). 62.157.7.177 11:20, 8. Feb. 2023 (CET)
Möglicherweise eine Redundanz…? chatGPT behauptet: "Es handelt sich um die gleiche Sache, die unter verschiedenen Namen bekannt ist:
  • Das Gauß'sche Wegesystem ist ein Verfahren zur geodätischen Vermessung von Land, das von Carl Friedrich Gauß entwickelt wurde.
  • Die Gaußsche Landesaufnahme ist ein ähnliches Konzept und bezieht sich ebenfalls auf das Verfahren zur geodätischen Vermessung von Land, das von Carl Friedrich Gauß entwickelt wurde.
  • Das Gauß-Krüger-Koordinatensystem ist ein Begriff, der das gleiche Verfahren beschreibt, das auf der Arbeit von Carl Friedrich Gauß und dem Mathematiker Johannes Krüger basiert. Es dient der Umrechnung von geografischen Koordinaten in rechtwinklige, meterbasierte Koordinaten.
In allen drei Fällen handelt es sich um dasselbe Verfahren zur Vermessung von Land." Mmmh… --77.6.170.165 14:55, 8. Feb. 2023 (CET)
Da sehen wir, was in den letzten Tagen schon öfter in den Medien war: chatGPT macht Fehler. Vermutlich gerade in fachlichen Spezialgebieten, wie auch Alexa Probleme mit speziellen Musikrichtungen hat (s. Diskussion unten). Was chatGPT da zusammengesucht hat, ist Unsinn ("gleiche Sache, die unter verschiedenen Namen bekannt ist"). Punkt 1 ist erfunden. In Punkt 2 ist kein ähnliches Konzept genannt. In Punkt 3 werden GK-Koordinaten beschrieben. Das ist aber etwas anderes und nicht das "gleiche Verfahren". Außerdem nennt WP den einen Mitbegründer Louis Krüger (Geodät), nicht Johannes. Könnte chatGPT schon diese Diskussion benutzt haben, um eine Verbindung zwischen den Begriffen herzustellen? 62.157.7.177 16:00, 8. Feb. 2023 (CET)
Das halte ich für äußerst unwahrscheinlich. Ich denke auch nicht, dass der Bot viele deutsche wp-artikel kennt, wenn er überhaupt welche kennt. --77.6.170.165 16:45, 8. Feb. 2023 (CET)
Korrekt, das Modell für ChatGPT wurde zwar ausgiebig mit Wikipedia-Inhalten gefüttert, aber da wohl nur aus der englischen Ausgabe. Zudem wurden keinerlei Daten nach 2021 verwendet, und ChatGPT lernt auch nicht mehr dazu. --Karotte Zwo (Diskussion) 17:37, 8. Feb. 2023 (CET)
Nach ein wenig nachbohren hat der Bot aber immerhin jetzt einen Link ausgespuckt, der vielversprechend klingt: "www.geod.uni-bonn.de/de/fachgebiete/vermessung/geschichte/carl-friedrich-gauss-und-das-gau%C3%9Fsche-wegesystem/", die Seite ist aber nicht erreichbar. Er behauptet jetzt: "Das "Gauß'sche Wegesystem" bezieht sich auf ein frühes Verfahren zur geodätischen Vermessung, das von Carl Friedrich Gauß entwickelt wurde. Es ist ein Vorläufer der Gaußschen Landesaufnahme und diente hauptsächlich dazu, geodätische Netze aufzubauen und Messungen durchzuführen, aus denen dann Karten erstellt werden konnten. Es war ein wichtiger Schritt in der Entwicklung moderner Verfahren zur geodätischen Vermessung." Hat er ja möglicherweise aus dem angegeben Link. --77.6.170.165 17:25, 8. Feb. 2023 (CET)
Es wird mysteriöser. Daß chatGPT Informationen aus einer wenige Stunden alten Diskussion verwendet, hätte ich auch kaum erwartet. Aber der Link zur Uni Bonn? Die Adresse geod.uni-bonn.de gab es mal, sie scheint aber nicht mehr aktuell zu sein. Die Adressen der Institute sind igg.uni-bonn.de usw. Der Link ist auch an anderer Stelle seltsam: einmal Gauß mit ss, dann in Unicode. Inhaltlich sagt mir das Wegesystem gar nichts, auch nicht als möglicher Vorläufer der Gaußschen Landesaufnahme. Rein technisch war die Triangulation (Dreiecksnetz) Vorläufer der Landesaufnahme (Erstellung der Meßtischblätter). Der Satz würde etwa passen, wenn man "Wegesystem" durch "Triangulation" ersetzten würde. Ob chatGPT uns etwas vorlügt, um seine Behauptungen zu bekräftigen? Oder sind es Fehler bei der Übersetzung und der Verbindung von Begriffen? 62.157.7.177 18:26, 8. Feb. 2023 (CET)
Dass der Bot etwas "vorlügt", halte ich für ausgeschlossen. Aus meiner Sicht müsste er dafür die Wahrheit kennen und gewollt verfälschen. Ein Übersetzungsfehler könnte sein, allerdings heißt es auf Englisch auch Triangulation. --77.6.170.165 22:16, 8. Feb. 2023 (CET)
Natürlich lügt der Bot. Und kennen tut er auch nichts: [3]. --Hareinhardt (Diskussion) 09:45, 9. Feb. 2023 (CET)
Ich halte es für grundfalsch einer sog. KI die Fähigkeit zuzusprechen, zu lügen. --77.8.44.247 10:33, 9. Feb. 2023 (CET)
"...Kommunikation einer subjektiven Wahrheit mit dem Ziel, im Gegenüber einen falschen Eindruck hervorzurufen" (aus Lüge). Der Bot weiss es nicht (wie sich durch nachfragen herausfinden lässt), gibt es aber nicht zu, sondern erzählt einem was vom Pferd. Ausserdem erzählt er zusätzliche "Fakten", nach denen nicht gefragt war. Warum tut er das? Um das vorhergesagte zu unterstreichen. Das geht schon als Lüge durch. Zur Frage, ob KI lügen kann: nun, die Programmierer haben sich wohl dafür entschieden, dass 1) der Bot nicht zugeben soll, keine Ahnung zu haben (wär ja dann ein grotten-langweiliger Bot), und 2) auch noch etwas mit Blödsinn ausholen soll. --Hareinhardt (Diskussion) 14:20, 9. Feb. 2023 (CET)
Das hat jetzt nichts mehr mit der Frage zu tun, dennoch: Der Bot gibt durchaus Fehler zu und entschuldigt sich auch dafür. --77.8.44.247 14:30, 9. Feb. 2023 (CET)
Jetzt werden hier drei Themen diskutiert:
  1. Das Wegesystem. Ich halte das für eine Interpretation eines Betrachters des Teppichs. Sieht aus wie ein Wegenetz, hat mit Gauß zu tun, ist also ein Gaußsches Wegesystem.
  2. Die Bewertung der Aussagen von chatGPT. Einzelne Aussagen sind richtig, die Zusammenstellung enthält aber auch Unsinn.
  3. Kann KI lügen? Nach meinem Verständnis ist Lügen nicht nur das Verfälschen der Wahrheit, sondern auch die Verbreitung von Falschem, wenn man die Wahrheit kennt. Man könnte KI sicher so trainieren, daß sie gezielt statt der Wahrheit etwas Falsche angibt. chatGPT hat aber wohl gar keine Kenntnisse, sondern setzt nur Texte nach Mustern zusammen. Das scheint auch für Links zu gelten. Auf eine Nachfrage bei der Uni Bonn wurde geantwortet, daß es die Webseite, die von chatGPT angegeben wurde, ziemlich sicher nie gegeben hat. Das Zugeben von Fehlern und das Entschuldigen entspricht Mustern von Gesprächen. Das kann chatGPT, weiß aber nicht, was es bedeutet. 62.157.7.177 18:02, 9. Feb. 2023 (CET)
Alle Achtung. Dass der Bot ggf. blühenden Unsinn erzählt, haben schon viele herausgefunden, man kann es leicht ausprobieren. aber dass er ganze Weblinks fabuliert, die es nie gegeben hat? Es geht also immer noch schlimmer als angenommen.--Meloe (Diskussion) 19:18, 9. Feb. 2023 (CET)

Alexa und die klassische Musik

Hallo zusammen. Ich setze Alexa ein, hauptsächlich (neben den Schalten einzelner Leuchten) um Musik zu hören. Bei Pop-Songs (im weitesten Sinne) klappt das hervorragend. Sage ich "Alexa, spiele Mensch von Grönemeyer" findet es den Song und spielt das Ding das ab. Problematischer ist es aber bei klassischer Musik. Sage ich "Alexa, spiele das Streichquartett Nummer 1 von Beethoven", so ist es mir zwar nicht ganz unwichtig, wer es spielt, aber vor allem möchte ich es ganz hören. Nicht nur Satz 2 (oder so). Ich will es ganz hören: Satz 1 bis Satz 4. Und (vielleicht) sogar in dieser Reihenfolge? Geht nicht. Oder ich bin zu blöde. Wie sage ich es richtig? Bitte nur qualifizierte Antworten, die sich genau mit diesem Problem beschäftigen. Nicht, ob Alexa oder Siri, ob Verschwörung, wie kann man nur Klassik hören oder ähnliches. Danke! --2003:D5:FF30:8A00:D81B:7CB:664:E33C 09:51, 8. Feb. 2023 (CET)

Vielleicht bin ich ja auch zu doof, aber so ohne weiteres geht das glaube ich nicht. Was wohl möglich wäre, wenn Du oft das gleiche Stück aus den Gleichen Teilen hören möchtest, eine Routine oder eine Playlist dafür anzulegen und die zu starten. --Windharp (Diskussion) 13:10, 8. Feb. 2023 (CET)
Die Konzepte, nach denen aktuelle „titelorientierte“ Musik verschlagwortet wird, also Titel, Album etcetera, lassen sich auf Klassisches (im weitesten Sinne, also Palestrina bis Stockhausen) nur eingeschränkt anwenden. Das Problem fällt immer wieder auf, wird aber nicht gelöst, wahrscheinlich ist das Interesse zu klein. Allein schon dass im Genre-Tag für den Pop-Rock-Bereich drölfhundert Stilrichtungen vorgesehen sind und alles davor einfach „Classical“ ist :D --Kreuzschnabel 13:48, 8. Feb. 2023 (CET)
Aus dem Grund hat z.B. Apple vor einigen Jahren den spezialisierten Klassik-Streaming-Dienst Primephonic gekauft und wollte 2022 auf Grundlage von dessen Katalog und Technik eine zweite App neben Apple Music starten, die eben auf die Bedürfnisse von Klassik-Hörern zugeschnitten ist. Eine Integration von Klassik-Features in die Pop-App hielt man da offenbar nicht für einen praktikablen Weg. (Nachdem 2022 ja nun schon eine Weile vorüber ist: Ein offizielles Statement zu einem möglichen neuen Veröffentlichungsfenster gibt es nicht, aber es gibt Anzeichen, dass Apple weiter an dem Projekt arbeitet und es nicht klammheimlich eingestampft hat.) --Karotte Zwo (Diskussion) 14:16, 8. Feb. 2023 (CET)
Wie willst du es denn untergliedern? Und du kannst nicht nach so etwas suchen wie "Baroque" oder "Renaissance"? Warum beginnt für dich die Musik erst mit Palestrina? Der Hauptunterschied ist, dass in der Pop- und Rockmusik sehr stark mit dem Interpreten verbunden wird, während in der Klassik (gemeint ist damit meist die europäische Kunstmusik aller Epochen) der Komponist immer ein sehr wichtiger Aspekt der Identifikation und Zuordnung ist. Aber die Ungebundenheit an den Interpreten gilt beispielsweise auch bei traditionellen Volksliedern (Folksongs) oder Jazzstandards (mag es auch bei in diesen Themen unbedarften Hörern manchmal so sein, dass sie ein Stück über die Aufnahme eines einzelnen Interpreten kennen und dann glauben, dieser sei der Urheber des Stückes). Der Wunsch nach Whiskey in the Jar oder Take the “A” Train dürfte beim Digitalassistenten auch unvorhersehbare Ergebnisse auslösen. Aber meines Wissens werden auch Klassik-Aufnahmen als Alben veröffentlicht. --Universal-InteressierterDisk.Arbeit 14:33, 8. Feb. 2023 (CET)
Schlag bitte unter Metonymie nach, was ich mit Johann und Karlheinz gemeint haben könnte. --Kreuzschnabel 16:14, 8. Feb. 2023 (CET)

Wenn das Lied "Mensch" in seinen Strophen abgespeichert wäre, hättest Du wahrscheinlich das gleiche Problem. Häufig werden klassische Werke nicht komplett, sondern (wahrscheinlich weil es sonst zu lang wird) in den einzelnen Sätzen gespeichert. Sag Deiner Alexa demnächst, dass Du das komplette Werk höern möchtest und wenn sie das nicht hinkriegt, dann nimm es hin, dass Du schlauer bist als die KI. --Elrond (Diskussion) 14:29, 8. Feb. 2023 (CET)

Das würde ich auch vermuten, aber bei der Ursache dafür denke ich an unterschiedliche Traditionen. Musikstücke, die Teil von Klassikalben sind, werden immer in mehrere Tracks für ihre einzelnen, eigenständigen Abschnitte aufgeteilt. Das hat nichts mit der Länge zu tun, sondern mit der Gliederung des Stückes selbst. Ich habe gerade die Trackliste des Albums Telemann (= Komponist Georg Philipp Telemann) von Dorothee Mields und Stefan Temmingh durchgesehen. Die Kantaten sind länger als die Sonaten, aber sind in weniger Abschnitte und somit Tracks des Albums gegliedert. In Rock und Metal werden wiederum selbst Stücke, die weit mehr als 10 oder auch mehr als 20 Minuten dauern, als einzelner Track veröffentlicht. --Universal-InteressierterDisk.Arbeit 14:53, 8. Feb. 2023 (CET)
Vielleicht funktioniert "Alexa, spiele die Aufnahme aus dem Jahr [hier Jahreszahl sagen] des Streichquartetts Nummer 1 von Beethoven"? Doof, wenn's in dem Jahr mehrere gab. Dann noch um den Namen des Quartetts ergänzen. --77.6.170.165 15:01, 8. Feb. 2023 (CET)
Nein, das funktioniert genauso wenig, wie "Spiele den X. Satz aus dem Streichquartett..." --2003:D5:FF30:8A00:D81B:7CB:664:E33C 15:53, 8. Feb. 2023 (CET)
Für letzteres erwartet Alexa möglicherweise z.B. "Track 2 aus dem Album Streichquartett Nummer 1 von Beethoven" --77.6.170.165 16:41, 8. Feb. 2023 (CET)
Hast Du es schon mit der Satzbezeichnung anstelle der Satznummerierung probiert? z.B. „Spiele das Presto aus dem Streichquartett d-Moll von ... in der Einspielung des ...-Quartetts Wien vom x.x.1979” ? Oder für das gesamte Werk: „Spiele alle Sätze aus dem Konzert für Klavier und Orchester von ... mit dem ... Philharmonikern unter dem Dirigenten ... in der richtigen Reihenfolge” ? Und schließlich sind Maschinen dafür bekannt, daß sie nur in Englisch programmiert werden, und bei allen anderen Sprachen ihre liebe Not haben, den Inhalt korrekt zu übersetzen. Probiere vielleicht einmal, Deinen Wunsch in Englisch anstatt in deutscher Sprache zu äußern. (alles mangels Versuchsobjekt nicht getestet, aber Erfahrungswerte aus anderen Bereichen der KI-Übersetzung) --92.73.132.89 16:42, 8. Feb. 2023 (CET)
Ich nennen Dir das Ergebnis: "Ich bin mir leider nicht sicher..." --2003:D5:FF30:8A00:D81B:7CB:664:E33C 17:16, 8. Feb. 2023 (CET)
Dann solltest Du das absurde Verhalten des Produkts vielleicht als Bug beim Hersteller melden. (Auch wenn ich nur wenig Hoffnung hege, daß die sich dafür sehr interessieren, solange nur wenige Menschen sich darüber beschweren). Denn die Fragen in der genannten Form sind ja an Eindeutigkeit kaum zu überbieten. Die Antwort „ich bin mir nicht sicher” ist also geradezu eine Frechheit von Seiten der Programmierer, die dieses System erstellt haben. --92.73.132.89 09:04, 9. Feb. 2023 (CET)
Ja, und dass dein Auto nicht fliegen kann, ist auch eine Frechheit. --79.225.237.122 09:11, 9. Feb. 2023 (CET)
Äpfel ←→ Birnen ? Die kannst Du aber schon auseinanderhalten?
Ein Auto ist i.d.R. nicht zum Fliegen konzipiert. Einer der Hauptzwecke dieses Gerätes jedoch ist es, Antworten zu liefern. Und auf präzise Fragen erwartet man daher schon ein bißchen mehr als die Antwort eines Dreijährigen. Zum Beispiel eine klare Aussage, daß dieser Sektor nicht im Funktionsumfang enthalten ist. Das erspart weitere sinnlose Zeit in weitere gleichartige Fragen zu investieren.
Nicht ein Auto das nicht fliegen kann, sondern ein Auto, das nicht fahren kann, das wäre eine vergleichbare Frechheit. --92.73.132.89 13:42, 9. Feb. 2023 (CET)
Hier wird erläutert, dass ein Amazon Music Abo eine erweiterte Musiksteuerung beinhaltet. Hast du das Abo? --77.8.44.247 18:01, 9. Feb. 2023 (CET)

Wie kann ich meinen Artikel auf Nutzerseite auf die Hauptseite verschieben?--Wilfried Steen (Diskussion) 09:24, 9. Feb. 2023 (CET)

--Wilfried Steen (Diskussion) 09:24, 9. Feb. 2023 (CET)

Im zweifelsfall, indem Du auf den Reiter "Verschieben" in der Leiste über dem großen Fenster klickst. Aber ich fürchte, Dein Artikel wird einen schweren Stand habe,
  • Er hat keine Belege, also externe Quellen.
  • Du solltest vorher bei Wikipedia:Relevanzcheck prüfen lassen, ob eine solche vorliegt.
  • (Nachedit) Der Stil ist eher prosaisch und nicht lexikalisch und eine Unterschrift braucht es auch nicht
Sonst kann es passieren, dass die viele Arbeit, die Du zweifelsfrei investiert hast, für die Katz ist.
--Elrond (Diskussion) 10:02, 9. Feb. 2023 (CET)
Verstehe ich Dich richtig, Du möchtest den Essay zu Dienste in Übersee in den ANR verschieben? Den Artikel gibt es da schon. --Erastophanes (Diskussion) 10:03, 9. Feb. 2023 (CET)
Du hast den 2010 sogar selbst erstellt ;-) --Erastophanes (Diskussion) 10:04, 9. Feb. 2023 (CET)
Wen das schon verschoben wurde, warum gibt es dann Benutzer:Wilfried Steen/Entwurf immer noch?? -- 193.47.104.35 10:49, 9. Feb. 2023 (CET)
Das wurde nicht verschoben, sondern per Copypaste rüberkopiert, wobei der Entwurf erhalten blieb. Urheberrechtlich ist das hier unproblematisch, da der kopierte Entwirf ausschließlich vom Rüberkopierer stammt. --Rôtkæppchen₆₈ 10:57, 9. Feb. 2023 (CET)
Nein, es geht um Aktion Arbeitslosen-Abgabe Braunschweig (bitte das Datum der Edits beachten. "Dienste in Übersee" stammen aus 2010. --2003:D5:FF37:4A00:F50D:8D34:312D:1109 17:52, 9. Feb. 2023 (CET)
Und wenn Benutzer:Wilfried Steen/Entwurf nicht mehr benötigt wird, dann setz dort ein {{SLA|nicht mehr benötigt --~~~~}} am Anfang ein, dann ist das schnell weg. --Kreuzschnabel 18:31, 9. Feb. 2023 (CET)

Hinweis: Für Fragen speziell zur Wikipedia gibt es ein eigenes Diskussionsforum: Fragen zur Wikipedia.

Unklare Quellenangabe

Hallo in "Potentielle und aktuale Unendlichkeit" ist unter Einzelnachweise Punkt 3. die nicht nachvollziehbare Angabe Physik 3, 207a1 zu finden. --EbertHeinrich (Diskussion) 12:43, 9. Feb. 2023 (CET)

Offensichtlich eine Zitatstelle aus Physik (Aristoteles). Solche Fragen aber bitte auf der Diskussionsseite des betreffenden Artikels klären. --FordPrefect42 (Diskussion) 12:54, 9. Feb. 2023 (CET)

Hinweis: Für Fragen speziell zur Wikipedia gibt es ein eigenes Diskussionsforum: Fragen zur Wikipedia.

Gleitsichtigkeit

Ich bin auf der Suche nach Informationen über den Unterschied zwischen Weitsichtigkeit und Gleitsichtigkeit.

Könnt ihr mir bitte erklären, was diese Begriffe bedeuten und wie sie sich voneinander unterscheiden? Welche Symptome treten bei jedem dieser Zustände auf und gibt es spezifische Behandlungsmethoden? Wie genau wirkt eine Gleitsichtbrille?

Ich wäre euch sehr dankbar, wenn ihr mir weitere Informationen und möglicherweise sogar Quellen zur Verfügung stellen könntet, damit ich mein Verständnis vertiefen kann.

Vielen Dank im Voraus für eure Hilfe.

--Hemtsar (Diskussion) 22:10, 8. Feb. 2023 (CET)

siehe Gleitsichtglas. --tsor (Diskussion) 22:16, 8. Feb. 2023 (CET)
(BK) Es gibt keine „Gleitsichtigkeit“. Eine Gleitsichtbrille kombiniert eine Fernsichtbrille (bei Kurzsichtigkeit) mit einer Lesebrille (bei Weitsichtigkeit) mit fließendem Übergang, so dass du für jede Entfernung von Unendlich (oberer Rand) bis Leseentfernung (unterer Rand) eine passende Stelle hast. Man fokussiert sozusagen per Kopfneigung. --Kreuzschnabel 22:16, 8. Feb. 2023 (CET)
+1 zu tsor, und siehe auch Weitsichtigkeit. Von Gleitsichtigkeit als Störung der Sehfunktion habe ich noch nie gehört, kann mir aber vorstellen, dass damit Brillen oder Kontaktlinsen gemeint sind, die sowohl Kurz- als auch Weitsichtigkeit ausgleichen können. --77.6.170.165 22:32, 8. Feb. 2023 (CET)
Hier: https://www.brillen.de/gleitsichtbrille ist die Gleitsichtbrille ganz gut beschrieben. Infos über die altersbedingte Weitsichtigkeit findet man hier: Presbyopie Die Krankheit "Gleitsichtigkeit" gibt es nicht. --Doc Schneyder Disk. 22:36, 8. Feb. 2023 (CET)
Wäre eine schöne Antwort. Bist Du kurz- oder weitsichtig? Gleitsichtig! --46.79.54.74 18:15, 9. Feb. 2023 (CET)
Optiker-Standardwitz: „Ich möchte eine Brille.“ – „Kurz- oder weitsichtig?“ – „Durchsichtig!“ --Kreuzschnabel 06:38, 10. Feb. 2023 (CET)

Auf welcher Rechtsgrundlage kann ein Fußballverein seine Spieler wegen eines Straßenverkehrsverstoßes mit Geldstrafen belegen?

Zwei HSV-Fußballprofis stehen unter dem Verdacht sich in Hamburg ein illegales Straßenrennen geliefert zu haben. Einer als Fahrer eines BMWs, der andere als Beifahrer eines Mercedes. Der BMW-Fahrer verlor die Kontrolle und fuhr sein Auto in eine Mauer und in eine (zum Glück leere) Bushaltestelle und entfernte sich von der Unfallstelle (im Auto des Sportkameraden). So weit, so schlecht. https://www.ndr.de/sport/fussball/Nach-Unfall-Empfindliche-Geldstrafen-fuer-HSV-Profis-Dompe-und-Mikelbrencis,hsv26626.html Das muß natürlich juristisch untersucht und entsprechend bestraft werden, insbesondere auch die Fahrerflucht. Darum soll es jetzt hier nicht gehen. Mich wunderte, dass auch der Arbeitgeber, also der HSV, "empfindliche Geldstrafen" aussprechen kann. Seit wann kann ein Arbeitgeber Angestellte mit Geldstrafen belegen? Ggf. kann man Boni streichen, wenn Spieler nicht eingesetzt werden. Oder ihn entlassen, was arbeitsrechtlich auch schwierig werden könnte. Aber Geldstrafen verhängen? --Doc Schneyder Disk. 18:22, 8. Feb. 2023 (CET)

Ohne Beleg: Sportlern wird eine Vorbildfunktion zugesprochen, arges auf die Kacke hauen wird inzwischen, vertraglich geregelt, geahndet, weil sich ja sonst keiner mehr dran hält und solche Sachen auch den Verein schaden (könnten). --77.6.170.165 18:39, 8. Feb. 2023 (CET)
Mit Beleg, aber leider hinter der Bezahlschranke. Er kann: Vertragstrafe [4].--Mhunk (Diskussion) 18:41, 8. Feb. 2023 (CET)
Und die argumentative Basis solcher Vertragsbedingungen ist, dass Angestellte das Ansehen des Vereins schädigen. Das kann, je nach Stärke des Danebenbenehmens, sogar bis hin zur vorzeitigen Vertragsauflösung führen. --Wwwurm Paroles, paroles 18:49, 8. Feb. 2023 (CET)
Zusatzfrage: In welchem rechtlichen Verhältnis stehen Sportler zu ihrem Verein? Sind sie dort angestellt? --193.83.26.49 19:14, 8. Feb. 2023 (CET)
Nach diesem Bericht ganz normale Angestellte. --TheRunnerUp 19:24, 8. Feb. 2023 (CET)
...evtl. sind das auch Firmenwägen und da greift, wie bei jedem anderen Firmenwagenfahrer ggf. eine Firmenwagen / Dienstwagen Compliance, Car Policy, Dienstwagenrichtlinie... oder wie auch immer das der Arbeitgeber nennt. Zudem wird so ein Fußballprofi als jemand der eine Vorbildfunktion hat mit / in seinem Arbeitsvertrag mit Sicherheit einem Verhaltens-Codex zugestimmt haben. --Btr 19:32, 8. Feb. 2023 (CET)

Prinzipiell werden Sportlern auch gerne vertraglich gesundheitsgefährdende Handlungen untersagt. Normalerweise sind damit andere Risikosportarten gemeint. -- southpark 21:13, 8. Feb. 2023 (CET)

Vermutlich ist Vertragsstrafe ein entscheidender Hinweis. Dann gibt es auch noch die Betriebsbuße, die aber in geschildertem Fall wohl nicht in Frage kommt. --Doc Schneyder Disk. 10:23, 9. Feb. 2023 (CET)

Alles, was nicht gesetz- oder sittenwidrig ist, kann per Vertrag beschlossen werden. Auch dementsprechend jede Strafe wegen vertragswidrigem Verhalten als Sanktion. Findet man vielerlei im Wettbewerbs- und Arbeitsrecht. Und dementsprechend ist die gesetzliche Grundlage das BGB. --2003:D5:FF37:4A00:F50D:8D34:312D:1109 10:29, 9. Feb. 2023 (CET)

Bliebe die Frage, wie und ob so ein Fehlverhalten in so einem Vertrag formuliert sein mag. Eigentlich ist Kündigung oder Sanktion nicht ohnes weiteres möglich, wenn eine Straftat "außerdienstlich", also nicht im Zusammenhang mit der Tätigkeit geschah.--Doc Schneyder Disk. 14:23, 9. Feb. 2023 (CET)
Es handelt sich beim Vorfall offensichtlich und ein strafbewehrtes Straßenrennen. Straftaten vertraglich mit zivilrechtlichen Forderungen zu ahnden, ist nicht wirklich schwer. --46.114.172.71 15:31, 9. Feb. 2023 (CET)
(Eine Geldstrafe ist es übrigens nicht, sondern eine Konventionalstrafe). (nicht signierter Beitrag von 2A02:3030:813:FCDA:D0C1:82EC:5F9F:DF0F (Diskussion) 15:37, 9. Feb. 2023 (CET))

Mir fehlt die Phantasie, wie so ein Passus im Arbeitsvertrag zu formulieren wäre. Hat jemand einen Vorschlag?--Doc Schneyder Disk. 19:21, 9. Feb. 2023 (CET)

Es gibt beim DFB einen Mustervertrag [5]. Dieser beinhaltet auch einen Passus zu Vertragsstrafen (§14).--Mhunk (Diskussion) 19:30, 9. Feb. 2023 (CET)
Da steht m.E. nur etwas über Verstöße bei Doping und bei Sportwetten, nicht über sonstige Straftaten wie Straßenverkehr, Finanzdelikte etc. --Doc Schneyder Disk. 00:10, 10. Feb. 2023 (CET)

US-Recht funktioniert deutlich anders als deutsches Recht. Aber mehr zu Physical Activity Clauses Can Void Athlete Contracts mit Beispiellinks -- southpark 18:37, 10. Feb. 2023 (CET)

Ziffer 1 mit Punkt darüber

In alten Dokumenten findet sich oft die Ziffer 1 mit einem Punkt darüber dargestellt. Mit welchem Zeichensatz kann ich dieses Zeichen erzeugen? İ Ṁṁ Ṅṅ Ȯȯ ist kein Problem, aber mit der Ziffer 1??

Überpunkt hilft leider nicht weiter.

--62.216.202.168 17:01, 10. Feb. 2023 (CET)

Mit Unicode so: 1̇ (1 + U+0307 COMBINING DOT ABOVE), das Rendering ist allerdings je nach Schriftart wohl eher unzufriedenstellend. -- Jonathan 19:16, 10. Feb. 2023 (CET)
Danke für den Input, leider hast du Recht, in den meisten Schriftarten liegt der Punkt neben der Ziffer. --2001:A61:2B1D:7801:7445:E87D:E25F:37F5 21:44, 10. Feb. 2023 (CET)
Mit TeX <math>\dot 1</math>: . --Rôtkæppchen₆₈ 19:52, 10. Feb. 2023 (CET)

"Nur für Speisezwecke verwenden"

...steht auf meiner Kartoffeltüte. Also nicht zum Schnitzen, für Kartoffeldruck oder physikalische Experimente? Klar, der gesunde Menschenverstand sagt mir, dass das wohl nicht gemeint ist, sondern, nehme ich an, die Verwendung als Pflanzkartoffeln (was ja, wenn man es recht bedenkt, auch ein Speisezweck ist, oder?). Viele Fragen: Wenn ich richtig liege, warum ist das so kryptisch formuliert? Ist die Einschränkung rechtlich bindend? Was kann (mir, der Natur) passieren, wenn ich die Knollen trotzdem einpflanze? Googeln führt mich auf eher verschwörerisch anmutende Seiten, deshalb hier die Bitte um fachliche Klärung. --77.180.182.117 11:26, 12. Feb. 2023 (CET)

Mir fallen zwei Möglichkeiten ein: Die Sorte ist geschützt und darf aus Lizenzgründen nur aus Saatkartoffeln des Anbieters gezogen werden, die man also jährlich neu kaufen muss. Oder die Kartoffeln sind mit einem Keimhemmer behandelt. Wobei: Och kenne keine Kartoffeln, die nicht feüher oder später auskeimen. Rainer Z ... 11:39, 12. Feb. 2023 (CET)
Ich meine, dass die Behandlung mit Keimhemmern die Ursache ist. Bei Bio-Kartoffeln, die nicht mit chemischen Keimhemmern behandelt sind, sollte der Hinweis (dann) fehlen. Richtig müßte es wohl heißen "Als Saatkartoffel nicht geeignet." Ein Verbot Speisekartoffeln für den Eigenbedarf anzubauen, kann ich mir nicht vorstellen. Wenn man Kartoffeln verkaufen will, können Regeln für das Pflanzgut vorhanden sein. --Doc Schneyder Disk. 12:47, 12. Feb. 2023 (CET)
Einen Artikel, dessen inhaltlicher Teil deutlich kürzer ist als sein erster Einzelnachweis, sehe ich in Pflanzkartoffelverordnung auch zum ersten Mal. --Kreuzschnabel 12:25, 12. Feb. 2023 (CET)
Das dürfte lizenz- bzw. zulassungsrechtliche Gründe haben, auch wenn die nach gesundem Menschenverstand völlig daneben sind. Man darf diese Kartoffeln wohl nicht zur Herstellung industrieller Produkte wie Stärke nutzen und als Saatgut darf man sie auch nicht verwenden. --Elrond (Diskussion) 12:50, 12. Feb. 2023 (CET)
Ein Hinweis zu Kartoffeln: Man sät Kartoffeln nicht, also heißt es auch nicht "Saatkartoffeln".
Und zur Ausgangsfrage: Ich vermute auch eher lizenz- bzw. zulassungsrechtliche Gründe. Dafür spricht auch der Hinweistext. Das ist eher ein Untersagen als ein Hinweis auf eine Nichteignung. 62.157.7.177 13:30, 12. Feb. 2023 (CET)
Wie heißen denn Saatkartoffeln in richtig? --95.119.203.24 14:47, 12. Feb. 2023 (CET)
Wahrscheinlich „Setzkartoffeln“ --Elrond (Diskussion) 15:32, 12. Feb. 2023 (CET)
Die sind mir auch am vertrautesten, aber „Pflanzkartoffeln“ geht auch. Von Pflanzgut im Gegensatz zum Saatgut spricht man allgemein, wenn aus Teilen bestehender Pflanzen neue Pflanzen wachsen (die denselben Genotyp wie die Urpflanze aufweisen, im Gegensatz zu wachsender Saat). --Kreuzschnabel 16:15, 12. Feb. 2023 (CET)
Saatkartoffel mag botanisch falsch sein, aber es wird am häufigsten verwendet: https://books.google.com/ngrams/graph?content=Pflanzkartoffel%2CSaatkartoffel%2CSetzkartoffel&year_start=1980&year_end=2019&corpus=de-2019&smoothing=3 --Doc Schneyder Disk. 18:27, 12. Feb. 2023 (CET)
Das sieht die Saatgutverordnung auch so, die nimmt Kartoffeln explizit aus. Das würde sie nicht tun, wenn Kartoffeln kein Saatgut wären. MBxd1 (Diskussion) 23:18, 12. Feb. 2023 (CET)
Welche "Kartoffeltüte" ist das? (Ich kenne so einen Hinweis von meinen Kartoffelnetzen nicht?) --Wikiseidank (Diskussion) 17:32, 12. Feb. 2023 (CET)
Oder, allgemeiner: Setzling. --46.114.5.78 17:41, 12. Feb. 2023 (CET)
@Wikiseidank: Sorte Queen Anne, Ursprung Deutschland, gekauft bei einem großen Discounter. Ausgesprochen kurze Garzeit übrigens; sie sind mir fast ein wenig weich geraten. 77.180.182.117 18:07, 12. Feb. 2023 (CET)
Erster Treffer bei Google "Queen Anne (Pflanzkartoffeln)";o) --Wikiseidank (Diskussion) 20:38, 12. Feb. 2023 (CET)

Ob "Lizenzgründe" oder "Keimhemmer" der Grund dieses Satzes sind, scheint nicht klar geworden zu sein. Mein Tipp wäre immer noch mal zu schauen, ob so ein Satz auch bei "Bio-Kartoffeln" vorhanden ist. Falls nicht, spräche es für Keimhemmer. --Doc Schneyder Disk. 23:12, 12. Feb. 2023 (CET)

Das werde ich beim nächsten Einkauf machen. Einstweilen vielen Dank allen! 77.180.8.55 06:31, 13. Feb. 2023 (CET)
Archivierung dieses Abschnittes wurde gewünscht von: 77.180.8.55 06:31, 13. Feb. 2023 (CET)

Undeutliche Fotos von UFOs

Gerade werden mal wieder irgendwelche unbekannten Flugobjekte über Nordamerika abgeschossen. Auf den paar veröffentlichen Fotos sieht man - wie immer - fast nix. Wir haben mit dem Hubble-Weltraumteleskop ein Gerät, das glasklare Aufnahmen liefert, von Galaxien die Millionen von Lichtjahren liegen, aber aus aller Nähe kommen nur verschwommene Bilder, und diese auch nur nachts. Gibt es echt keine guten Verschwörungstheoretiker mehr? --Zollwurf (Diskussion) 14:37, 12. Feb. 2023 (CET)

Tja, die Armen haben nur miese Kameras, weil die Eliten ihnen die guten vorenthält! --Elrond (Diskussion) 15:30, 12. Feb. 2023 (CET)
Bitte die einschlägigen Dokumentarfilme aus dem Wissensfernsehen anschauen oder im Café weiterspekulieren. --2003:F7:DF1B:9C00:89C4:72EB:2771:F71C 15:35, 12. Feb. 2023 (CET)
Dazu reichen wir als absolut seriöse Zusatzinformation gern noch [6] hinterher. Wohlbekomm’s! --Kreuzschnabel 19:01, 12. Feb. 2023 (CET)
Archivierung dieses Abschnittes wurde gewünscht von: Das Café ist nebenan. --2003:F7:DF1B:9C00:89C4:72EB:2771:F71C 15:35, 12. Feb. 2023 (CET)

Was das zuletzt abgeschossene Objekt angeht, das wurde den Berichten zufolge nördlich von Anchorage in Alaska gesichtet. Nicht gerade dicht besiedeltes Gebiet und da ist die Wahrscheinlichkeit, dass am Boden jemand mit guter Kameraausrüstung ist, nicht so hoch. Ansonsten aber auch mal abwarten, was da noch an Bildern oder Videos auftaucht. Falls nur Normalsterbliche mit ihren üblichen Smartphone-Kameras da waren, um das Ding zu filmen, wurdert es nicht wirklich, dass ein Ding von der Größe eines Autos in 40.000 Fuß Höhe nicht gerade kristallklare Bilder liefert. Von dem deutlich größeren Ding, das vor ein paar Tagen den Ausflug über die USA gemacht hat, gibt es durchaus relativ gute Bilder. --Proofreader (Diskussion) 15:40, 12. Feb. 2023 (CET)

Ich vermute mal, dass es im Privatbereich zwar hochwertige Kameras mit hoher 8k-Auflösung gibt, die allerdings einen Teller in maximal 10 cm Entfernung deutlich ablichten. --Zollwurf (Diskussion) 16:22, 12. Feb. 2023 (CET)
Die vor einigen Wochen diskutierten Objekte aus en:Pentagon UFO videos haben sich besonders schnell bewegt und waren bald wieder verschwunden. Für eine gute Aufnahme ist aber insbesondere eine lange Belichtung hilfreich. --BlackEyedLion (Diskussion) 21:45, 12. Feb. 2023 (CET)
Exakt das. Hubble belichtet je nach Aufnahme mehrere Wochen an einem Bild und ist dafür gebaut. War auch nicht ganz billig. Man kann auch "stationäre" Kameras am Boden bauen die mit Nachführung ein bewegliches Objekt mit dickem Teleobjektiv länger als ein paar Millisekunden belichten, aber die Zielgruppe der Leute die Wetterballons fotografieren wollen ist wohl doch eher übersichtlich. --Windharp (Diskussion) 07:20, 13. Feb. 2023 (CET)

Habt ihr das auch?

(Beim besten Willen keine Wissensfrage für die Auskunft, daher ins Café verpflanzt.) --Kreuzschnabel 09:52, 11. Feb. 2023 (CET)

Weiterführende Relativsätze

Gibt es die nur im Deutschen oder auch in anderen Sprachen? falls Unsicherheiten bestehen: Ein weiterführender Relativsatz ist ein Satz, der an seinen Hauptsatz inhaltlich anschließt und eigentlich auch alleine als Hauptsatz verwendet werden könnte. Das findet aber nicht statt, was zur Folge hat, dass sich die Syntax und Pronomen oder Adverbien ändern. (Hier habe ich beide Anschlussformen benutzt. Zuerst einen eigenständigen Hauptsatz, in welchem mit dem Demonstrativpronomen das auf die Aussage des vorherigen Satzes verwiesen wird, dann ein weiterführender relativsatz, in welchem was an dessen Stelle tritt. Ähnlich würde auch "deshalb" durch "weshalb" und "daraufhin" durch "woraufhin" ersetzt.)

--2A0A:A540:FCE7:0:9CB6:CBEE:9970:94B 17:13, 6. Feb. 2023 (CET)

Ich verstehe die Frage nicht. Meines Erachtens gilt Folgendes:
  • Dass man statt eines Relativsatzes einen Hauptsatz verwenden kann, ist bei explikativen (nicht notwendigen/nicht einschränkenden) Relativsätzen möglich. Das erste Beispiel ist ein restriktiver (notwendiger) Relativsatz.
  • Das Relativpronomen was wird verwendet, wenn es sich auf den gesamten übergeordneten Gliedsatz und nicht auf ein Satzglied darin bezieht.
  • Deshalb ist kein Ersatz für was, sondern für was zur Folge hat, worin des für was und halb für zur Folge steht. --BlackEyedLion (Diskussion) 17:23, 6. Feb. 2023 (CET)
Um meine Ausführungen zu "deshalb" zu verdeutlichen, seien folgende Beispiele erwähnt: Ich stelle fest, dass Lisa schön ist. Deshalb will ich sie wiedersehen. = getrennte Sätze Ich stelle fest, dass Lisa schön ist, weshalb ich sie wiedersehen will. = weiterführender Relativsatz
Die Frage nach explikativen und restriktiven Relativsätzen betrifft überhaupt nur attributive Relativsätze, also solche, die sich auf ein Satzglied beziehen, das ohnehin da ist, sei es auch nur ein Demonstrativpronomen oder ein Lokaladverb. Daneben gibt es aber auch freie Relativsätze, die selbst Satzglieder sind und kein Bezugswort haben. Sie drücken aus, dass das betreffende "Objekt" (ein besserer Begriff fiel mir nicht ein) unbekannt oder unbestimmt ist. (Beispielsätze: Wer anderen eine Grube gräbt, fällt selbst hinein. (Sprichwort) oder Was du liebst, lass los, was du liebst, lass frei (Santiano, Was du liebst), insbesondere wenn der frei Relativsatz im Hauptsatz eine andere Funktion übernimmt als sein unbestimmtes Satzglied kann auch ein Demonstrativpronomen nachgestellt werden: Wer zu spät kommt, den bestraft das Leben) Der weiterführende Relativsatz entspricht nun in seinem Aufbau einem Relativsatz, genauer einem freien Relativsatz, aber ist inhaltlich kein Teil des Hauptsatzes, sondern beschreibt etwas, das sich aus diesem ergibt.
Ist das nun ein deutsches Phänomen oder gibt es ihn auch in anderen Sprachen, den weiterführenden Relativsatz? --2A0A:A540:FCE7:0:9CB6:CBEE:9970:94B 18:09, 6. Feb. 2023 (CET)
Eine verständliche Erklärung eines weiterführenden Relativsatzes und Vergleich mit attributiven und freien Relativsätzen steht in Relativsatz#Bezug: attributive und freie Relativsätze. Und im Spanischen gibt es das auch: s. Relativsatz in der spanischen Sprache#Lo que, drittes Beispiel: "... was die Feier natürlich sehr belebte." 62.157.7.177 18:20, 6. Feb. 2023 (CET)
Im Englischen: en:English relative clauses#Overview Nr. 10 und außerdem sehr interessant auch in Nr. 11. --BlackEyedLion (Diskussion) 18:22, 6. Feb. 2023 (CET)
Da wüsste ich gar nicht, wie ich die Sätze ins Deutsche übersetzen könnte. --Universal-InteressierterDisk.Arbeit 22:41, 6. Feb. 2023 (CET)
Die Beispiele in Nr. 11? Einfach zwei Sätze bilden und den zweiten mit "Dieses Bild", "Diese Männer" usw. anfangen. 62.157.7.177 11:57, 7. Feb. 2023 (CET)
Das wäre im Englischen ja grundsätzlich auch möglich. Interessant, dass es im Englischen sogar einen halbwegs als weiterführender Relativsatz einordnungsfähigen Satzanschluss gibt, der im Deutschen unmöglich ist. --Universal-InteressierterDisk.Arbeit 20:52, 9. Feb. 2023 (CET)
Nicht unmöglich, man kann doch einfach wörtlich übersetzen: "...welches Bild ich später zerstörte". Ist natürlich im Deutschen ähnlich selten wie im Englischen. --2A00:20:C01C:695D:42F2:F5D1:C71E:B3A4 14:31, 11. Feb. 2023 (CET)

Verbindungen mit "und"

Nachdem ich einige Verwirrungen über die Bedeutung solcher Formulierungen bemerkt habe, will ich einmal fragen, ob meine Annahme der Realität entspricht: Verbindet man zwei Substantive durch "und" miteinander, die etwas oder jemanden "klassifizieren" oder beschreiben, und verwendet dabei nur ein Artikelwort (oder ich vermute auch Genitivattribut, aber da bin ich mir nicht sicher), so bezeichnen beide Substantive im Singular immer eindeutig dasselbe Objekt beziehungsweise dieselbe Person. In allen anderen Varianten (Plural, kein Artikelwort vorhanden) sind je nach Kontext beide Bedeutungen (identisch oder verschiedene Objekte/Personen) möglich. Beispielsätze: "Der König besuchte seine Residenz und Grenzfestung." (ein Ort) "Der König besuchte seine Residenz und seine Grenzfestung." (wahrscheinlich verschiedene Orte, oder es werden die unterschiedlichen Eigenschaften/Funktionen des einen Ortes betont, der Fokus wird drauf gerichtet, dass der Besuch beiden Funktionen galt) "Er war der Enkel eines Generals und Putschisten." (eindeutig ein Großvater gemeint) vs. "Er war der Enkel eines Generals und eines Putschisten." (Entweder werden beide Großväter beschrieben, oder die widersprüchlichen Eigenschaften des einen Großvaters werden betont.)

Es gibt natürlich noch komplexere Bedeutungsmöglichkeiten, vor allem im Plural: "die Herzöge, Kurfürsten und Könige von Sachsen" fasst alle zusammen, "die Herzöge, die Kurfürsten und die Könige von Sachsen" betont die Gruppen als getrennte Gruppen mit Gemeinsamkeiten. "die Kurfürsten von Sachsen und Landgrafen von Thüringen" sind nur diejenigen, die beides waren, während "die Kurfürsten von Sachsen und die Landgrafen von Thüringen" beide Gruppen in ihrer durchaus auch personell getrennten Gesamtheit bezeichnet (also auch diejenigen, die nur "Kurfürst von Sachsen" ode rnur "Landgraf von Thüringen" waren.

Liege ich mit diesen Annahmen richtig? --Universal-InteressierterDisk.Arbeit 15:08, 11. Feb. 2023 (CET)

Wenn das erste Substantiv mit Artikelwort und das zweite als Abstraktum grundsätzlich ohne Artikelwort verwendet wird, nicht. "Ich nehme ein Paket und Vernunft (oder Geld) an." --BlackEyedLion (Diskussion) 20:16, 11. Feb. 2023 (CET)

Da gehe ich mal davon aus, dass Du mit Deiner Annahme richtig liegst. Etwas komplizierter kann es wohl durch das Gendern sein und nimmt man z.B. England und Schottland im Plural. "Die Könige und Königinnen von England und Könige und Königinnen von Schottland" (in ihrer Gesamtheit) oder "Die Könige und Königinnen von England und die Könige und Königinnen von Schottland" (personell getrennte Gesamtheit).--Hopman44 (Diskussion) 20:33, 11. Feb. 2023 (CET)

Unfall auf A6 mit US-Army-Raketen und -Sprengmitteln

Welche Stelle wurde bei diesem Unfall (web) beliefert? --Gelöbnix (Diskussion) 16:04, 11. Feb. 2023 (CET)

Aufgrund der Fahrtrichtung würde ich auf den Truppenübungsplatz Grafenwöhr tippen. Aber so kann man das nicht beantworten. Das kann genauso eine Lieferung in Richtung Tschechien, Polen oder Ukraine gewesen sein. --Lidius (Diskussion) 16:21, 11. Feb. 2023 (CET)
Wird dort mit Raketen 'geübt'? --Gelöbnix (Diskussion) 17:00, 11. Feb. 2023 (CET) PS: Die LKW-Fracht auf dem Foto (unten) sah mir auch nicht wirklich nach der von Militärfahrzeugen aus?!
Ja, da wird mit Raketen geübt. Siehe z.B. [7]. Das sind ganz normale Frachtcontainer auf militärischen LKW. Auch nicht ungewöhnlich. --Lidius (Diskussion) 17:38, 11. Feb. 2023 (CET)

Anamorphotisches Verfahren

Ist das Anamorphotische Verfahren heutzutage noch Standard bei aktuellen Kinofilmen, die digital gefilmt werden? Oder hat sich das mit der Digitalisierung der Filmkameras erledigt? --176.199.210.248 19:21, 11. Feb. 2023 (CET)

Siehe en:Anamorphic format#Recent use. --Rôtkæppchen₆₈ 22:44, 11. Feb. 2023 (CET)

Zinnkrug = Zinnstitze ?

Der Begriff Zinnstitze ist nicht in Wiki zu finden. Er bezeichnet wohl eine besondere Form von Zinnkrug, der in Versteigerungen verwendet wird. Was bedeutet der Begriff und kann er in einem Artikel ergänzt werden?

--Thomas Brenner (Diskussion) 10:53, 12. Feb. 2023 (CET)

Hast du mal nach "Stitze" gegoogelt? --Digamma (Diskussion) 10:59, 12. Feb. 2023 (CET)

Etymologie des Wortes "Blutrache"

Tja, da habe ich neulich mal ein bisschen über Bedeutungen nachgegrübelt, die mit dem Wort "Blut" in Verbindung stehen, und auf ungefähr drei Sinnzusammenhänge von Wörtern und einschlägigen Redewendungen gekommen:

  1. körperliche Gewalt, wie z.B. in "Blutvergießen", "Blutbad", "sich bis aufs Blut bekämpfen", "Blut an den Händen haben" (gilt auch negativ, siehe z.B. "die Geiselnahme wurde unblutig beendet").
  2. intensive Gefühle oder Emotionen, wie z.B. in "mit Herzblut dabei sein", "das Blut in Wallung bringen" oder antonymisch in "blutleer".
  3. Verwandtschaftsbeziehungen, wie z.B. in "Blut von meinem Blute", "blutsverwandt", "Blut ist dicker als Wasser".

Jetzt frage ich mich, ob das Wort "Blutrache" nun eher vom ersten oder dritten dieser Punkte herrührt. Geht es darum, dass etwas möglichst mit Gewalt vergolten werden soll, oder darum, dass ein Vergehen gegen die eigene Familie gerächt werden soll? Ist das so eindeutig festgelegt, oder wurde diese Doppelbedeutung mehr oder weniger beabsichtigt (ich gehe mal davon aus, dass diese Vokabel in der deutschen Sprache sicherlich auch erst irgendwann um 1800 herum in diversen Wörterbüchern aufgetaucht ist...)? -- КГФ, Обсудить! 19:11, 9. Feb. 2023 (CET)

Pfeifer (Wörterbuch) (hm, immer noch rot) erklärt es als Tötung als Vergeltung für einen getöteten Sippenangehörigen (also 3) und verortet es zeitlich (verzeitet es?) im 17. Jahrhundert. --Kreuzschnabel 19:25, 9. Feb. 2023 (CET)
Immerhin hat Wolfgang Pfeifer einen Artikel. Das Wörterbucg wird dort auch erwähnt. Aber ein Desiderat ist der Artikel trotzdem, da gebe ich Dir Recht. --Elrond (Diskussion) 19:59, 9. Feb. 2023 (CET)
"Auch will ich eures Leibes Blut rächen und will's an allen Tieren rächen und will des Menschen Leben rächen an einem jeglichen Menschen als dem, der sein Bruder ist. Wer Menschenblut vergießt, des Blut soll auch durch Menschen vergossen werden; denn Gott hat den Menschen zu seinem Bilde gemacht." 1.Mose 9, 5-6.--Meloe (Diskussion) 19:37, 9. Feb. 2023 (CET)
Dass das eine Übersetzung ist (welche eigentlich?), das ist dir aber schon klar? Auch wenn die Form "des" (anstelle von "dessen") als Genitiv (Maskulinum Singular) des Demonstrativpronomens (der) eher auf eine ältere Übersetzung aus der Frühen Neuzeit hindeutet, die auch älter als das 17. Jahrhundert sein könnte. --Universal-InteressierterDisk.Arbeit 20:46, 9. Feb. 2023 (CET)
Egal wie oder was - das Gute Buch passt immer. Nga Ahorangi (Diskussion) 20:49, 9. Feb. 2023 (CET)
Die dritte Variante für Blutrache scheint zuzutreffen. Der Adelung (Grammatisch-Kritisches Wörterbuch der Hochdeutschen Mundart) sieht darin „die Rache oder Bestrafung des vergossenen Blutes“, der Bluträcher sei „derjenige, welchem es zukommt, vergossenes Blut zu rächen, besonders so fern es der nächste Verwandte des Entleibten ist.“ [8] (Stichwort eingeben und aufrufen und im Folgenden bis auf DWDS die Wörterbuchnetz-Verweise im Menü rechts neben dem Stichwort benutzen) Die Grimm-Brothers schreiben: „vergeltung für die ermordung eines verwandten (durch tötung des täters oder eines seiner angehörigen)“. DWDS sieht es als „Rache für einen Mord an einem nahen Verwandten durch die Ermordung des Täters“. [9] Ebenso das GWB (Goethe-Wörterbuch): „(uralte relig begründete) Verpflichtung eines Familienangehörigen, den Mord, Totschlag am Täter od seiner Familie zu ahnden, auch iZshg mit der Ablösung der Selbstjustiz durch richterl Urteil (vgl Blutbann)“. Meyers wird etwas ausführlicher: „Blutrache, eine Urform der Rechtspflege, bezweckt die Wiederherstellung des durch die Tötung eines Familiengenossen zerstörten Rechtszustandes auf dem Wege der Selbsthilfe und legt dem nächsten Blutsverwandten eines Getöteten die Pflicht auf, an dem Mörder oder dessen Verwandten mit eigner Hand Rache zu nehmen. Die B. wird oft jahrelang und durch eine Reihe von Geschlechtern ausgeübt und verwickelt nicht selten ganze Familien und Stämme in blutige Fehden.“ und verweist u.a. auf das germanische Wergeld, eine auch im Koran vorgesehene Ablösesumme, um das gegenseitige Töten zu beenden. Interessant dabei auch: „Allen Germanen eigen und besonders üblich in Island waren die Blutbrüderschaften, feierlich geschlossene Verbindungen auf Leben und Tod zwischen Männern, von denen der eine für den andern die B. übernahm und, wenn er sie nicht ausüben konnte oder jener starb, sich selbst tötete. Auch bei Slawen und Illyriern kamen solche Verbindungen vor, und die Karen in Birma sowie die Dajak auf Borneo schließen sie noch jetzt unter Vermittelung eines Priesters, der sie von ihrem Blute trinken läßt.“
Wer muss da nicht spontan an den abgedrehten, in der Wildnis Kroatiens spielenden Klassiker zwischen einem grundanständigen sächsischen Vermessungsingeneur und einem edlen Wilden gallischer Herkunft denken, assistiert von Martin Böttchers Ohrwurm, der kein-Wunder-wunderschönen Nscho-tschi-Versini (Gott hab sie selig) und dem einmaligen immerlustigen Ralf Wolter, auch schon gestorben wenn ich mich nicht irre? Und das ist nur der erste Band, dem noch einige folgen werden. Mein Bluter, mein Bluter... Da konnte sich in Felix Austria auch das Rote Kreuz nicht mehr zurückhalten. Und grinsend schleicht sich noch hinzu als Clou der Schuh des Manitu. (Dieser Beitrag wurde nicht mit ChatGPT formuliert.) --88.72.111.240 00:37, 10. Feb. 2023 (CET)
zwischen einem grundanständigen sächsischen Vermessungsingeneur und einem edlen Wilden gallischer Herkunft – ganz sicher, dass du hier nicht Figur und Darsteller durcheinanderwirfst? Bei „gallisch“ musste ich wirklich erstmal grybln :) --Kreuzschnabel 06:46, 10. Feb. 2023 (CET)
Ich bin ganz sicher, dass ich hier Figuren und Darsteller lustvoll durcheinanderwerfe. Diese literarische Freiheit nehme ich mir einfach, weil ich sie ohnehin habe. :) (Dieser Beitrag wurde nicht mit ChatGPT formuliert.) --88.72.111.240 09:12, 10. Feb. 2023 (CET)

Blutwurst passt aber nicht zu 1,2 oder 3.--Hopman44 (Diskussion) 19:51, 10. Feb. 2023 (CET)

Die erste Variante. Mein Kürschners-Universal-Konversations-Lexikon von 1906 sagt: älteste Form der Wiedervergeltung bei Mord, bei allen Völkern üblich, heute (!) noch in Albanien, Corsica u.a. Pflicht der Verwandten!--Hopman44 (Diskussion) 19:57, 10. Feb. 2023 (CET)

Ich bin nicht dieser Ansicht und bevorzuge noch immer, und dabei auch mit einer anderen Interpretation des Kürschner, die dritte Variante. Das vorklassische Sippenrecht, das über Kulturkreise hinweg die Blutrache in den Kontext von Verwandschaftbeziehungen stellt, wird im römischen Recht durch ein Staatsrecht zurückgedrängt, das ein Tötungsdelikt in einen übergeordneten Zusammenhang stellt und auch so behandelt. Es kommt also zu Frühformen der Rechtspflege, die zweifellos auf den tradierten Rechtsauffassungen basieren, diese aber weiterentwickeln und sich dabei an der Veränderung der zunehmenden Komplexität der Herrschaftsformen orientieren. In diesem Prozess wird Blutrache über die Jahrhunderte zunehmend zum Anachronismus und Mord als Tatbestand über das ius talionis hinaus immer differenzierter ausgestaltet und sanktioniert bis hin zum Diskurs der heutigen Strafzwecktheorie. Eine genauere Betrachtung sollte also Altrömisches Recht seit etwa 750 v.Chr. und Vorklassisches Recht seit etwa 200 v.Chr. zum Ausgangspunkt nehmen und den Zusammenhang zwischen Rechtsauffassung und der staatlichen Entwicklung sowie der beabsichtigten Durchsetzung von Staatszielen berücksichtigen. Die Blutrache verharrt in diesem Paradigmenwechsel in den tradierten Vorstellungen des Sippenrechts, vollzieht also die Entwicklungsschritte der Rechtsauffassung und Rechtspflege nicht mit. Gestützt wird diese Sicht imho z.B. durch die Darstellung im Artikel Talion: „Ein altrömisches Gesetz (leges regiae) aus der Königszeit ließ es anfänglich noch zu, dass der an einem freien Menschen vorsätzlich begangene Mord (parricidium) durch die Angehörigen des betroffenen Familienverbandes (gens) mit der Blutrache gerächt werden konnte.[Anm. 6: leges regiae: qui hominem liberum dolo sciens morti duit, parricidas esto.] Um einer ausufernden Sippenfehde entgegenzusteuern, durfte der Mörder nur dann ohne vorangegangenes Gerichtsverfahren getötet werden, wenn die Täterschaft als zweifelsfrei erwiesen galt. Der Bluträcher, der diesem einschränkenden Grundsatz zuwiderhandelte, wurde selbst als parricidas (Mörder) angesehen.“ (Dieser Beitrag wurde nicht mit ChatGPT formuliert.) --88.72.111.240 01:02, 11. Feb. 2023 (CET)
Ich habe erhebliche Zweifel an der Übersetzung "Mord" für parricidium, und entsprechend mit der Gleichsetzung parricidas = Mörder. Meines Wissens ist parricidium nur der Mord an Blutsverwandten. Der Bluträcher, der den nicht verwandten Mörder eines Familienangehörigen tötet, kann also gar kein parricidas sein. (Einer der wesentlichen Fortschritte im späten römischen Recht war übrigens die Gewährleistung des Lebensschutzes für Kinder: Während in der klassischen Zeit der Vater im Rahmen seiner potestas patris das Recht hatte, die eigenen Kinder straflos zu töten, wurde das damit untersagt und entsprechende Vergehen als parricid(i)um bestraft.) --95.119.97.73 11:33, 11. Feb. 2023 (CET)
Fein. Es ist ein Zitat aus dem Artikel Talion und es ist sicher hilfreich, wenn deine Zweifel dort begründet und belegt zu einer Verbesserung des Artikels beitragen. Kern des von mir benutzten Zitats ist der Prozeß der Zurückdrängung des Sippenrechts zugunsten eines Staatsrechts und der damit verbundenen Gerichtsbarkeit und rechtspflegenden Rechtsentwicklung. Und als Folge davon ein veränderter Stellenwert der Blutrache. Ist das denn durch deinen Zweifel als Gedanke/Argument hinsichtlich der Fragestellung nun obsolet geworden? (Dieser Beitrag wurde nicht mit ChatGPT formuliert.) --88.72.111.240 13:08, 11. Feb. 2023 (CET)
Der Neue Pauly (1996-2008) (dort nach "parricidium" suchen, ich bekomme es leider nicht besser verlinkt):
"Parricidium, auch par(r)icidas bezeichnet spätestens seit der ausgehenden röm. Republik den Verwandtenmord, der seit ca. so v.Chr. durch eine I. Pompeia de parricidiis (Dig. 48,9) aus den übrigen Tötungsfällen herausgegriffen wurde. Die auf die mores maiorum (Altvätersitte; — mos maiorum) zurückgeführte Strafe bestand zumindest für den Mord an Vorfahren im Einnähen des ge- geißelten Täters mit diversen Tieren in einen Sack (> culleus), der dann ins Wasser geworfen wurde (eindringliche Schilderung Cic. S. Rosc. 70). In der frühen lat. Rechtssprache ist p. ein Schlüsselausdruck nicht nur für die Erkenntnis der inneren Einstellung bei der Tötung, sondern auch des Kapitalverfahrens. So ist uns mehrfach überliefert, daß die — quaestores pamicidii für Kapitalsachen zuständig waren; bei Fest. 221 darüber hinaus noch ein angebliches Gesetz des Numa Pompilius: Si dolo sciens morti duit, parricidas esto (‚Wer vorsätzlich den Tod zufügt, soll als Mörder behandelt werden;). Über die Bedeutung von p. esto in dieser Regel, die durch die Abstraktion dolo sciens ihren wesentlich jüngeren Ursprung verrät, gibt es eine kaum mehr überblickbare Lit., deren Hauptgruppe ausgehend von etym. Argumenten die verschiedensten Sanktionen hinein- deutet [3]. Mehr Wahrscheinlichkeit hat die Lesung entsprechend der obigen Übers. [2]. Danach wollte der Satz nicht einen Hinweis auf die ohnehin bekannte Sanktion geben, sondern sie auf den Fall der vorsätzlichen Tötung begrenzen." --77.1.9.239 14:21, 12. Feb. 2023 (CET)

Ja, Kürschner läßt es ja in gewisser Weise auch offen, ob der Mörder eines eigenen Angehörigen der Sippe der Blutrache zum Opfer fallen soll/muß, oder ob der Mörder aus einer anderen Sippe, der einen fremden Sippenangehörigen getötet hat, der Blutrache zum Opfer fallen soll/muß. (Pflicht der Verwandten).--Hopman44 (Diskussion) 11:04, 12. Feb. 2023 (CET)

Vielleicht doch nochmal etwas Logik: Wenn unter den Maßstäben des Sippenrechts ein Sippenmitglied von den Angehörigen einer anderen Sippe getötet wird, so wächst daraus die Verpflichtung, diesen Tod an wem auch immer aus der Sippe des Mörders zu rächen (ob an dem Mörder selbst oder an einem anderen Mitglied möchte ich hier offen lassen, weil belanglos). Unsere angeführten Quellen weisen darauf hin, dass hier eine Eigendynamik entstehen kann, wenn mit der ausgeführten Blutrache die Sache nicht erledigt ist und eine sich mit jeder weiteren Rache selbst reproduzierende Dauerfehde entsteht. Dennoch ist diese Variante halbwegs übersichtlich und in einigen Kulturen klug genug ergänzt durch die Option von Zahlungen wie Wergeld, die einen Weg zur Beruhigung eröffnen.
In dem zweiten Szenario (das Mitglied einer Sippe tötet ein anderes Mitglied der eigenen Sippe) macht Blutrache wenig Sinn. Wäre die Bedeutung des Wortes Blutrache, vergossenes Blut müsse auch innerhalb der Sippe mit Blut vergossen werden (Auge um Auge...), dann würde ja mit dem Töten eines Sippenmitglieds und der Rache an dem Mörder wiederum ein Racheinteresse an dem Mörder des Mörders entstehen. Es wäre also ein nicht enden könnender Blutrachefeldzug innerhalb der Sippe ausgelöst. So doof waren die aber nicht. Zweifellos gab es für den Mord innerhalb einer Sippe eine Form der Sanktionierung, wahrscheinlich ebenfalls kulturell im Sippenrecht verankert und/oder innerhalb dieses hierarchischen Systems von dem Oberhaupt der Sippe entschieden, aber eine Sippe als Überlebenszusammenhang schwächt sich deshalb doch nicht selbst. Das schließt Fehden innerhalb einer Sippe mit anderen Motivationen wie Macht und Erbfolge sicher nicht aus. Aber der Automatismus der Blutrache stände gegen alle Werte und Ziele, die mit dem familiären Zusammenschluss in einer feindlichen Umwelt verbunden sind. Die Regelungen innerhalb der Sippe begründen sich letztlich auf der Hierarchie und dem Machtwort und das Ziel eines Machtwortes ist, etwas zu beenden.
Aus meiner Sicht bezieht sich Blutrache daher auf die Sanktionierung von Angriffen auf Mitglieder der eigenen Sippe durch Täter, die anderen Sippen angehören. Damit wäre Blutrache neben der ihr innewohnenden drohenden Vergeltung auch eine präventiv wirkende Kultur, eine Art übergreifendes Schutz- und Versicherungssystem für alle Sippenmitglieder: Wer einen von uns angreift bekommt es mit allen von uns zu tun.
Ich bin mir auch nicht sicher, ob der Rückgriff auf ältere Wörterbücher und Lexika nicht mit einer begrenzten Tauglichkeit für eine Durchdringung verbunden ist. Historische und rechtswissenschaftliche Untersuchungen haben sich sicherlich umfassender und kompetenter mit der Materie auseinandergesetzt und auf eine andere, bessere Quellenlage zugegriffen. Der Blick einzig auf die Etymologie (das ist ja der Ausgangspunkt der Fragestellung) scheint mir eher verwirrend, weil der Sachverhalt dann eigentlich ebenso (und vielleicht sogar zuallererst) in den konnotierten Sinnzusammenhängen der entsprechenden Begriffe in anderen Sprächen untersucht werden müsste. Dieser Beitrag wurde nicht mit ChatGPT formuliert. --88.72.111.240 14:59, 12. Feb. 2023 (CET)

Festland-Staaten mit meisten Einwohnern auf Inseln

Hab gerade überrascht festgestellt, dass über die Hälfte der Einwohner Dänemarks auf Inseln lebt. Dänemark ist kein Inselstaat wie UK, Japan oder Irland, sondern ein "Festland-Staat", daher war das überraschend für mich. Meine Frage: In welchen anderen Staaten mit Festland-Anteil leben mehr Menschen auf Inseln als auf dem Festland?

--2003:EC:BF1B:5744:5560:454E:C58D:B20 18:47, 11. Feb. 2023 (CET)

Malaysia schon mal nicht, obwohl mehr Fläche auf den Inseln liegt. Das Vereinigte Königreich liegt im Übrigen nicht vollständig auf Inseln, weil Gibraltar kontinental ist. --BlackEyedLion (Diskussion) 19:36, 11. Feb. 2023 (CET)
Allerdings ist Gibraltar kein Teil des Vereinigten Königreichs, sondern steht „nur“ unter dessen Souveränität (siehe Artikel: Britische Überseegebiete). --Mombacher (Diskussion) 20:16, 11. Feb. 2023 (CET)
Ich denke, Dänemark ist da nicht zu schlagen. Finnland unter 1%, Australien etwas über 2%, Italien 11%. --Ralf Roletschek (Diskussion) 20:23, 11. Feb. 2023 (CET)
Ein größeren Anteil als Malaysia mit ca. 20 % hätte noch Äquatorialguinea mit rund 28 %. --Icodense 20:58, 11. Feb. 2023 (CET)
(nach BK) Sehe ich auch so, wie Ralf. Hätte noch Griechenland ins Feld zu führen - aber Kreta, Euböa, Rhodos und die anderen über 3.000 Inseln haben nicht genug Einwohner aufzubieten (dürften geschätzt ca. 1,5 bis 2 von 10,3 Millionen Einwohnern sein?). --Mombacher (Diskussion) 21:03, 11. Feb. 2023 (CET)
Indonesien wird wohl nicht gelten. --Elrond (Diskussion) 21:20, 11. Feb. 2023 (CET)
Wahrscheinlich nicht, da m.W. kein Festlandsanteil. Habe mir gerade auch noch Singapur angesehen – in der Hoffnung, zu diesem Inselstaat würde vielleicht noch ein oder mehrere kleine „Landzipfel“ auf der malaiischen Halbinsel gehören. Dem ist aber nicht so... --Mombacher (Diskussion) 21:41, 11. Feb. 2023 (CET)
Die unter 1% bei Finnland sind die Ålandinseln. Um Mittsommer herum sind Millionen Finnen in ihrem Mökki, viele davon auf Inseln. Aber es ist ja nichtmal bekannt, wieviele Seen oder Inseln Finnland hat. Für ein paar Wochen im Jahr könnte Finnland vielleicht mit Dänemark konkurrieren. --Ralf Roletschek (Diskussion) 22:01, 11. Feb. 2023 (CET)
Dänemark wird bei den heute bestehenden Staaten bei der Frage der Spizenreiter sein. Denn es hackt ja schon am Umstand, dass dafür DIE Grossstadt des Landes auf einer Insel liegen müsste. Wie eben bei Dänemark der Fall ist, wo die grösste Metropolregion des Staates auf einer Insel liegt (Auch die drittgrösste Stadt Odense liegt auf einer Insel). Und das ohne dabei als Inselstaat zu gelten. Long Island ist zwar dicht bevölkert (2207 Einw./km²), aber eben im Vergleich zur Rest der USA, sind die nicht ganz 8 Millonen Bewohner nicht wirklich viel. Selbst wenn man jetzt nur den Bundesstaat New York betrachet, reicht das nicht. Das sind "nur" 8 von 20 Millionen.--Bobo11 (Diskussion) 23:41, 11. Feb. 2023 (CET)
Großbritannien und Neuseeland. Festland in Antarktika. Ob du das anerkennst, überlasse ich deiner völkerrechtlichen Expertise. --2003:DE:6F2A:9024:2D45:1629:53ED:3F27 03:53, 12. Feb. 2023 (CET)
Wieso ist denn Dänemark ein "Festlandstaat", obwohl der größte Teil der Landesfläche Inseln sind? Vor diesem Hintergrund auch kaum überraschend, dass auf dem größeren Teil der Landfläche der größere Teil der Bevölkerung lebt.... Egal, auf (heute) subnationaler Ebene gäbe es noch Neufundland und Labrador - Fläche überwiegend Festland, Einwohner überwiegend Insel. --Studmult (Diskussion) 19:02, 12. Feb. 2023 (CET)
Bezeichnet man nicht normalerweise nur Staaten ohne Festland als Inselstaaten? Wird Festland nicht für gewöhnlich viel "höher" als Inseln bewertet? Nebenbei gesagt meine ich auch schon Irland (Insel) und Großbritannien (Insel) als Festland bezeichnet gesehen zu haben, in Bezug auf die vielen kleinen Inselchen in der Umgebung. Überhaupt ist der Begriff "Festland" schon komisch, als wären Inseln nicht "fest", sondern schwämmen übers Wasser oder übers Meer. (Als was gelten eigentlich Inseln in Flüssen oder Binnenseen?) --Universal-InteressierterDisk.Arbeit 20:45, 12. Feb. 2023 (CET)
Wenn man die Insel Irland als Festland betrachtet, dann leben die meisten Iren auf dem Festland. Weil auch dann ist eben in der Regel die Metropolregion auf der deutlich grössten Insel, die dann aus der Binnensicht als Festland bezeichnet wird. Um bei Irland zu bleiben Dublin liegt nun mal auf Irland (Insel), und nicht auf einer vorgelagerten Insel. Entweder ist Irland ein Inselstadt, oder eine deutliche Mehrheit lebt auf dem Festland. Fällt also bei der Betrachung unter „ferner liefen..“ .--Bobo11 (Diskussion) 22:15, 12. Feb. 2023 (CET)
Historisch kann man noch an die Republik Venedig mit der Inselstadt Venedig und das Aztekenreich mit der Binneninselstadt Tenochtitlan denken, in beiden Fällen reicht die Einwohnerzahl auf den Inseln aber nicht für eine Mehrheit. --BlackEyedLion (Diskussion) 22:23, 12. Feb. 2023 (CET)

Dosierung von Medikamenten - Umrechnung

Die Dosierung von Medikamenten ist angeblich auf 70 kg schwere "Normmenschen" bezogen und muss daher auf das Gewicht der Patienten umgerechnet werden.

Bloß wo steht das? Wo ist das festgelegt?

Gibt es dazu ein Gesetz, eine Richtlinie?

Steht das in Richtlinien der WHO?

Gilt das auch für IE (Internationale Einheiten)?

--178.165.193.151 00:39, 12. Feb. 2023 (CET)

Die Sache ist kompliziert. Und eine Frage von Pharmakokinetik und Pharmakodynamik.
Es gibt Medikamente, deren Dosierung an den BMI angepasst werden muß/sollte, und Medikamente, bei denen das nicht der Fall ist.
Hier dazu ein recht guter Artikel: https://www.deutsche-apotheker-zeitung.de/daz-az/2015/daz-18-2015/brauchen-dicke-mehr
lg --Doc Schneyder Disk. 00:49, 12. Feb. 2023 (CET)
Es ist immer besser, wenn man angepaßt dosiert, aber man kann nicht einfach für einen Patienten mit 140 kg die doppelte Dosis nehmen, weil Fettleibige vorwiegend mehr Fett haben, das kaum verstoffwechselt, aber bspw. durchaus speichern kann (s. Verteilungsvolumen). Je schmaler die therapeutische Breite des Mittels ist, desto wichtiger ist es, möglichst genau passend zum Patienten zu dosieren. Irgendwelche Gesetze o.ä gibt es zu den 70 kg AFAIK nicht. Die 70 kg kenne ich nur als Standard, weil Durchschnitt; auch bei den Zulassungsstudien. Die Pharmaindustrie fertig Arzneimittel entsprechend dieses Standards, die Ärzte verordnen nach Packungsbeilage – auch, um sich nicht angreifbar zu machen. Eine detaillierte fachliche Darstellung zum Thema findest Du unter Pharmakokinetik bei Adipositas (PZ). --94.219.2.161 23:38, 12. Feb. 2023 (CET)

TV: wie nennt sich drüber gesprochene Synchronisation?

Es gibt viele Dokumentationen, da wird eine deutsche Synchronisation über die Originalstimmen gesprochen (also nicht Lippensynchron). Wie nennt sich das? Im Artikel Synchronisation (Film) konnte ich dazu nichts finden. --2001:9E8:A50A:C000:8C06:35C5:7F19:A111 20:32, 12. Feb. 2023 (CET)

Voiceover#Synchronisation--Conan (Eine private Nachricht an mich? Bitte hier lang.) 20:35, 12. Feb. 2023 (CET)

Sterbeort von Tiny Wirtz

Tiny Wirtz ist nicht in Horrem gestorben sondern in ihrem Haus in Köln-Müngersdorf Dies bitte ändern. --2A0A:A543:D032:0:F4B9:3F83:B6DD:9596 11:12, 12. Feb. 2023 (CET)

Den Hinweis bitte dort platzieren: Diskussion:Tiny Wirtz. Hier ist die Auskunft für allgemeine Fragen. --92.229.133.249 11:53, 12. Feb. 2023 (CET)
… und auch das nur, solange sie nicht die Wikipedia direkt betreffen. Dafür gibt’s dann WP:FZW. Der Fragesteller kann die Änderung aber auch gleich selbst durchführen, am besten mit stichhaltigem Beleg. --Kreuzschnabel 12:21, 12. Feb. 2023 (CET)
Das ist ein Denkfehler. Man kann Behauptungen ohne Beleg auch ohne Beleg löschen oder ggf. verändern. Der Vorrang des ersten Schwurblers ist nicht zu akzeptieren. --195.200.70.43 15:46, 13. Feb. 2023 (CET)

kein WP:Quelle, kein Eintrag. Habe den Sterbeort entfernt. Gern mit geeigneter Quelle einen ergänzen ...Sicherlich Post 11:51, 15. Feb. 2023 (CET)

Archivierung dieses Abschnittes wurde gewünscht von: ...Sicherlich Post 11:51, 15. Feb. 2023 (CET)

Dateinamen mit Textdatei als Quelle als Batch ändern

Hallo,

kennt jemand ein Freeware-Tool (vorzugsweise grafisch und lauffähig auf Windows 10, zur Not aber auch als Ubuntu-Software und/oder als Kommandozeilenbefehl), mit welchem ich

  • eine texthaltige Datei (bsp. TXT, CSV, XSLX) als Dateinamenquelle eingeben kann und
  • mit welchem dann sequentiell alle Dateien in einem Ordner automatisch umbenannt werden?

Ich möchte gerne eine Videosammlung umsortieren, was bei der dreistelligen Zahl an Dateien mir mit manuellem Umbenennen zu umständlich ist. Eine Vorlagendatei vorbereiten und dann einen Batch starten zu können wäre willkommen, ich kenne dafür aber kein Tool. Wer hat was hilfreiches? Grüße, Grand-Duc ist kein Großherzog (Diskussion) 13:10, 11. Feb. 2023 (CET)

Ich mache für so etwas gerne eine Exceltabelle: Erste Spalte alter Dateiname (aus einem DIR-Befehl dir /a/b/s L:\Videos >videos.csv), zweite Spalte neuer Dateiname, dritte Spalte Formel ="ren """&A1&""" """&B1&"""". Kostenlos geht das mit LibreOffice. Dann per Copypaste die dritte Spalte in cmd.exe reinkopieren. --Rôtkæppchen₆₈ 13:16, 11. Feb. 2023 (CET)
Was enthält die "texthaltige Datei" denn? Pro Zeile nur den alten Namen oder auch den neuen Namen? Kann man im ersten Fall den neuen Namen nach einfachen Regeln aus dem alten Namen erzeugen? 62.157.7.177 13:36, 11. Feb. 2023 (CET)
Der neue Name wird Zeichen enthalten, die nicht im alten Namen vorhanden sind, ein Umwandeln ist nicht möglich. Und ich habe bis jetzt keine Vorbedingungen, wie ich die Vorlagendatei erstelle: alte und neue Namen, nur den neuen Namen, Trennung zeilenweise (erscheint mir als das vernünftigste, so dass jede Zeile in der Vorlagendatei nur einen neuen Namen enthält), Trennung mit Semikolons oder Kommata... Grüße, Grand-Duc ist kein Großherzog (Diskussion) 13:54, 11. Feb. 2023 (CET)
Wenn die Datei so aufgebaut werden kann, daß sie den alten und neuen Dateinamen in einer Zeile enthält (getrennt durch ein Leerzeichen), müßtest Du am Zeilenanfang nur ein "ren " oder unter Linux "mv " ergänzen, und hättest eine Batchdatei zum Umbenennen. Ist das zu einfach, und habe ich das Problem nicht erkannt? 62.157.7.177 14:03, 11. Feb. 2023 (CET)
Ah, also als Vorgehen unter Windows:
  1. Editor starten (Notepad oder Notepad++)
  2. zeilenweise schreiben: ren ALTER_DATEINAME.mkv NEUER_DATEINAME.mkv
  3. als .BAT abspeichern
  4. BAT ausführen.
Richtig?
Die Theorie kenne ich zwar halbwegs, aber wie schreibt man den Dateipfad in der BAT richtig? Einfach 1-zu-1 übernehmen, oder muss der Antislash irgendwie escaped werden? Oder: reicht es aus, wenn die BAT im Ordner der umzubenennenden Dateien liegt? Grüße, Grand-Duc ist kein Großherzog (Diskussion) 14:23, 11. Feb. 2023 (CET)
Prozentzeichen verdoppeln, Dateinamen mit Leerzeichen, Kommata etc in Anführungszeichen setzen. Generell Anführungszeichen um Dateinamen setzen ist kein Fehler. --Rôtkæppchen₆₈ 14:43, 11. Feb. 2023 (CET)
und zur frage: ja. wenn die datei im selben verzeichnis liegt wie die umzubenennenden dateien, dann reichen die dateinamen (und man braucht keinen kompletten pfad anzugeben). -- seth 14:49, 11. Feb. 2023 (CET)
Unter Linux würde ich das einfach mit sed machen, und mit einem einzigen Befehl alle nötigen Ergänzungen in jede Zeile der textdatei automatisiert einfügen (sed -i.bak '<Ersetzungen>' '<Datei>'), so daß eine bashscript Datei entsteht, bei deren Ausführung die Umbenennung aller Dateien stattfindet. Erfordert etwas Erfahrung im Umgang mit der komplexen Befehlsstruktur von sed, geht dann aber unglaublich schnell. Alternativ gibt es gprename (sudo apt-get install gprename) mit einer grafischen Oberfläche, das könnte das Problem der Massen-Umbenennung nach bestimmten Kriterien ebenfalls lösen.
Unter Windows wäre Bulk Rename utility eine Möglichkeit. Und natürlich kann man es auch unter Windows mit einfachen Batch-scripten machen (wie seinerzeit unter MS-DOS), die Syntax habe ich aber nach jahrzehntelanger Windows-Abstinenz leider inzwischen vergessen. --92.73.156.111 14:38, 11. Feb. 2023 (CET)
Total Commander kann das auch, der benötigt nur eine einspaltige Liste mit den neuen Dateinamen, die gleich sortiert ist wie die Anzeige in der Dateiansicht, z.b nach Datum. --TheRunnerUp 19:24, 11. Feb. 2023 (CET)
PS: Und wenn die neuen Namen aus den alten abgeleitet werden können, ist möglicherweise gar keine Textdatei notwendig, da kann das Programm einiges, z.B.: Datum umdrehen (aus "12.07.2000 MeinVideo1.mkv" wird "2000-07-12 MeinVideo1.mkv" etc.), oder DOS-Umlaute durch Windows-Umlaute ersetzen ... --TheRunnerUp 23:19, 11. Feb. 2023 (CET)
Ich hatte mir jetzt eine Batchdatei gebaut gehabt. Allerdings darf ich jetzt noch von Hand die kaputt gegangenen Umlaute reparieren. Fall ich in Zukunft nochmal so eine Batch-Umbenennungsaktion machen will oder muss: kann ich irgendwie verhindern, dass beispielsweise aus "Gefährliche Töne" ein "Gef├ñhrliche T├Âne" wird? Die BAT-Datei habe ich mit Notepad++ abgespeichert; ich vermute, dass eine Zeichenkodierung unpassend war. Default ist es wohl UTF-8, muss ich das auf ANSI oder was anderes umstellen? Grüße, Grand-Duc ist kein Großherzog (Diskussion) 00:47, 12. Feb. 2023 (CET)
Unter Windows? Da musste in die batch Datei mit reinschreiben welche Codepage benutzt werden soll. chcp 1252 > null an den Anfang der Datei und schon klappt es auch mit Umlauten. --2A01:598:B972:F1DD:C305:ACCE:3B6:AE78 (ohne (gültigen) Zeitstempel signierter Beitrag von 2A01:598:B972:F1DD:C305:ACCE:3B6:AE78 (Diskussion) 07:36, 12. Feb. 2023 (CET))
Default ist nicht UTF-8, sondern bei deutschem Windows Codepage 850 auf der Kommandozeile und eben die von meinem Vorredner erwaehnte Codepage 1252 (= ANSI) in Windows-Programmen, zu denen auch Texteditoren zaehlen. -- Juergen 134.255.192.126 01:14, 13. Feb. 2023 (CET)

Dampfdruck von Legierungen

Angeregt durch die obige Frage stellt sich mir die Folgefrage nach dem Dampfdruck von Galinstan: Unter Raoultsches Gesetz steht, dass jede Komponente eines fluessigen Stoffgemisches, um das es sich bei Galinstan ja handelt, in der Dampfphase einen Partialdruck hat, der kleiner ist als der Gleichgewichtsdampfdruck, den sie als reine Substanz besitzt.
Da es sich bei den Komponenten von Galinstan aber um Feststoffe handelt, die unter Normalbedingungen keinen Dampfdruck aufweisen, wuerde das bedeuten, dass auch Galinstan als Stoffgemisch keinen Dampfdruck hat. Unter Galinstan steht nun aber, dass es einen Dampfdruck aufweist. Dieser ist dort zwar nur fuer 500 Grad angegeben, daraus wuerde ich allerdings schlussfolgern, dass bei Normabedingungen der Dampfdruck nicht verschwindet, sondern nur unmessbar klein wird.
Was stimmt nun: Haben fluessige Legierungen unterhalb des Schmelzpunktes aller ihrer Bestandteile nun einen Dampfdruck oder nicht ? -- Juergen 217.61.197.161 10:29, 11. Feb. 2023 (CET)

Es ist ein Irrtum, anzunehmen, Feststoffe besäßen keinen Dampfdruck. Jede Substanz hat einen Dampfdruck und sei er noch so unmessbar gering. Der Zusammenhang zwischen Temperatur, Aggregatzustand und Dampfdruck lässt sich im Phasendiagramm ablesen.--Rôtkæppchen₆₈ 12:18, 11. Feb. 2023 (CET)
Das heißt, irgendwann verdampft auch ein Stein oder Glas? --176.199.210.248 18:26, 11. Feb. 2023 (CET)
Man spricht erst einmal nicht von Verdampfen, sondern von Verdunsten oder Sublimieren. Nach meinem Verständnis gibt es Stoffe, die diesen Vorgängen nicht unterliegen, weil die Austrittsarbeit für die Atome oder Moleküle an der Grenzschicht zur Luft zu groß ist. (Dass sich der Artikel auf Elektronen beschränkt, halte ich für ungerechtfertigt.) --BlackEyedLion (Diskussion) 19:32, 11. Feb. 2023 (CET)
Der einzige durch die Austrittsarbeit existierende Schwellwert ist der absolute Nullpunkt der Temperatur, und der kann nicht erreicht werden. Oberhalb des absoluten Nullpunkts haben die Teilchen rein statistisch ein nach oben hin unbegrenztes Energiespektrum zur Verfügung, das also auch die Grenze der diskreten Energiezustände überschreitet und in den kontinuierlichen, ungebundenen Bereich hineinreicht. Insofern kann es immer zum Abdampfen von Teilchen, also zur Verdunstung, kommen, wenn auch bei niedrigen Temperaturen mit verschwindend geringer Wahrscheinlichkeit. --95.119.203.24 13:10, 12. Feb. 2023 (CET)
Und dazu kommt natürlich noch die Möglichkeit des Tunnelns! --Elrond (Diskussion) 13:48, 12. Feb. 2023 (CET)
„wenn auch bei niedrigen Temperaturen mit verschwindend geringer Wahrscheinlichkeit“ Na, dann passt ja meine Antwort. Es ist auch nicht unmöglich, dass sich der Inhalt eines Glases Wasser gemeinsam einen Meter nach oben bewegt oder durch das Glas tunnelt, auch wenn die Wahrscheinlichkeit „verschwindend gering“ ist. Physik befasst sich mit tatsächlichen oder möglichen Phänomenen, nicht mit unmöglichen. --BlackEyedLion (Diskussion) 22:28, 12. Feb. 2023 (CET)
Wenn auch Feststoffe einen Dampfdruck haben, warum beschraenkt sich das Raoultsche Gesetz dann explizit auf Fluessigkeiten ? -- Juergen 134.255.192.126 02:03, 13. Feb. 2023 (CET)
Bei Feststoffen kann es seeehr lange dauern, bis sich ein Sättigungsdampfdruck einstellt. Für praktische Belange ist das Raoultsche Gesetz hier daher schwer bis unmöglich verifizierbar. --Rôtkæppchen₆₈ 23:49, 13. Feb. 2023 (CET)

wo ist die Fahrgestellnummer BMW 318 BJ.1994

So funktioniert es:

  • ERSETZE diesen Abschnitt durch eine konkrete Frage.
  • Lass nur die nachfolgende Zeile (--84.59.109.218 15:15, 11. Feb. 2023 (CET)) am ENDE deiner Frage stehen. Sie wird in deine Signatur umgewandelt.

wo ist die Fahrgestellnummer BMW 318 BJ.1994 --84.59.109.218 15:15, 11. Feb. 2023 (CET)

Was funktioniert (nicht), und welche Teile von ERSETZE und "nur" hast Du nicht verstanden? --95.119.97.73 16:38, 11. Feb. 2023 (CET)
Für Zusatzfragen bitte neue Frage erstellen -Ani--46.114.157.97 00:06, 12. Feb. 2023 (CET)
Im Motorraum, unter der Motorhaube, an der Innenseite der Fahrertür, Feld unter der Windschutzscheibe , im Türrahmen. auf dem Armaturenbrett Link: https://www.baum-bmwshop24.de/blog/lexikon/fahrgestellnummer/ Ich hoffe ich konnte helfen. --Fiver, der Hellseher (Diskussion) 18:33, 11. Feb. 2023 (CET)
Als Hellseher müsstest Du das doch genauer wissen :) SCNR --Optimum (Diskussion) 00:37, 12. Feb. 2023 (CET)


Ist zwar nicht exakt das richtige Modell. Ich würde aber auf dem Bild bei Bayrische Motorenwerke Placette unter links im Bild anfangen zu suchen. --Bobo11 (Diskussion) 22:40, 12. Feb. 2023 (CET)
Siehe Fahrzeugidentifikationsnummer (FIN). Steht auch im Fahrzeugschein (jetzt: Zulassungsbescheinigung Teil I) und Fahrzeugbrief (jetzt: Zulassungsbescheinigung Teil II). --77.0.119.136 13:37, 13. Feb. 2023 (CET)

"Kühlkörper" bei Gasherden/-kochern

Ok, ich habe jetzt keine validen Meßdaten... Folgendes Problem: Die Wärmeübertragung durch Topfböden ins Kochgut ist eigentlich eher so mittelprächtig. Damit es trotzdem anständig kocht und brät, muß die Wärmequelle schon ganz schön heiß sein. Bei elektrischen Kochplatten eigentlich auch kein Problem: die drücken ihre Leistungsaufnahme auf jeden Fall in den Topf, denn sie können praktisch jede beliebige Temperatur bis hin zur Rotglut annehmen und "knacken" damit jeden thermischen Widerstand. Viel Verluste wird das wohl auch gar nicht geben: woanders hin als in den Topf kann die Wärme kaum, die Verluste entstehen eher dadurch, daß Töpfe nicht isoliert sind und ziemlich viel Wärme über die Wand und den Deckel an die Raumluft abgeben. Anders bei Gasflammen: Deren Wärmekapazität und -leitfähigkeit ist gering, sie werden mutmaßlich nur einen Teil der Wärme an den Topfboden abgeben, und das abströmende Flammgas hat danach noch mindestens die Temperatur der Unterseite des Topfbodens - das muß eigentlich zwangsläufig eine schlechte Energieeffizienz aufweisen. Wie wäre alternativ folgendes: Über den Gasflammen ist eine Herdplatte, auf die man die Töpfe stellt, und die ist nach unten hin wie ein Kühlkörper mit großer Oberfläche (Finger und Rippen) aufgebaut. Das überträgt zunächst einmal die Wärme der Flammen viel effektiver auf die Kochplatte als es der Topfboden mit seiner kleineren Oberfläche selbst kann, wodurch die Flammgase stärker gekühlt werden. Und zweitens geht das abgekühlte Flammgas anschließend noch durch einen Wärmetauscher zur Verbrennungsluftvorwärmung. Wie wäre es damit, was bringt das hinsichtlich der Energieeffizienz, und gibt es das vielleicht schon? Mir tut es immer in der Seele weh, wenn ich irgendwelche Gaskocher sehe, bei denen offensichtlich mehr heißes Flammgas am Topf vorbeiströmt als diesen erwärmt. (Gut, mein Favorit wäre natürlich die motorische Nutzung des Brenngases, um damit dann eine Wärmepumpe anzutreiben, die die Prozeßwärme zum Kochen bereitstellt - wozu aus Flammen mit 1000° C nur Wärme mit einer nutzbaren Temperatur von höchstens 200° C erzeugen? Dabei gehen 800 von 1250 K nutzlos verloren. Wobei beim elektrischen Kochen die Leistunge natürlich auch nicht in einem Heizwiderstand verpulvert werden müßte, sondern ebenfalls per Elektromotor eine Wärmepumpe antreiben könnte.) --95.119.97.73 17:13, 11. Feb. 2023 (CET)

Schau mal Kochfeld an (es gibt auch Gas-Glaskeramikkochfelder) und Gasherd. da steht schon einiges zu Vor- und Nachteilen. Grüße, Grand-Duc ist kein Großherzog (Diskussion) 17:48, 11. Feb. 2023 (CET)
Vor allem bestätigt es die Annahmen: Wirkungsgrad Elektroplatte 67-74 %, also 26-33 % Verluste, Gasflamme 44 % Wirkungsgrad, also 56 % Verluste. (Mit "Wirkungsgrad" ist wahrscheinlich der Anteil der Energie aus dem verwendeten Energieträger gemeint, der den Topf erreicht. Wobei die scheinbar günstigeren Zahlen für "elektrisch" irreführend sind: von der Energie der Kraftwerkskohle kommen gerade mal 30 % im Haushalt an, 60 % gehen bereits aus thermodynamischen Gründen im Kraftwerk selbst verloren, weitere 10 % sind Übertragungsverluste im Stromnetz. Und von den 30 % werden beim Kochen nur 70 % genutzt, macht also bezogen auf die Primärenergie einen Gesamtverlust von annähernd 80 %. Klingt nach erstklassiger Unweltsauerei.) Da geht also auf jeden Fall noch was, und mit Wärmepumpentechnik erst recht. --95.119.97.73 22:15, 11. Feb. 2023 (CET)
Als langjähriger Gasherdkoch mein Senf. Die nach oben strömenden Brandgase geben dabei auch noch Wärme an den Wänden ab. An der Oberseite sind sie so abgekühlt, dass ich problemlos dort hantieren oder einen Kochlöffel aus Holz legen kann, ohne mir die Flossen zu verbrennen, Das klappt auch bei Pfannen. Wir haben allerdings eher schwere Pötte und geschmiedete Pfannen mit nennenswerter Masse. Wenn die Arbeitstemperatur erreicht ist, wird die Flamme so geregelt, dass die Temperatur gehalten wird. das geht mit ein wenig Erfahrung sehr präzise und die Ansprechzeiten sind sehr kurz, speziell im Vergleich zu elektrischen Platten, die ewig nachheizen. Während meiner Studienzeit kochte ich im Wohnheim auf Elektroplatten und die kamen mir im Vergleich zum Gas sehr unpraktisch vor, war aber auch sicher Gewohnheitssache, --Elrond (Diskussion) 18:16, 11. Feb. 2023 (CET)
Die Gase sind oben am Topf nicht deswegen abgekühlt, weil sie Wärme an die Topfwand abgegeben haben, sondern weil sie sich mit der Raumluft vermischt haben. Und außerdem sind die auch nicht wirklich kalt, sondern einfach nur trocken und fühlen sich deswegen, u. a. aufgrund ihrer niedrigen Wärmekapazität, nur nicht so heiß an. Heiße trockene Luft ist an den Händen ziemlich gut erträglich, das kann man leicht mit einem Fön ausprobieren. --95.119.97.73 22:28, 11. Feb. 2023 (CET)
Zu Beginn des Kochvorganges kondensiert das Wasser, das beim Verbrennen des Methan entsteht an den Töpfen. Wenn der Inhalt auf etwa 100 °C erwärmt wurde, natürlich nicht mehr. Trocken ist die Abluft daher natürlich nicht. Die Strömung dürfte einigermaßen laminar sein, daher ist eine Verwirbelung mit der Umgebung wahrscheinlich nicht sehr bedeutend. Warum Du Gasherde schlecht reden möchtest, ist mir allerdings nicht so ganz klar. --Elrond (Diskussion) 01:12, 12. Feb. 2023 (CET)
Welche relative Luftfeuchtigkeit hat denn mit Luft verbranntes Methan bei 100° C? Ein Mol Methan benötigt bzw. verbraucht zwei Mol O2 und erzeugt dabei ein Mol CO2 und zwei Mol Wasser, zwangsläufig werden dabei auch noch acht Mol N2 miterwärmt. Das Rauchgas besteht also aus acht Mol Stickstoff, zwei Mol Wasserdampf und einem Mol Kohlenstoffdioxid, maW macht der Volumenanteil des Wasserdampfs 18 % aus, und wenn sich das Rauchgas auch noch mit Luft vermischt, noch entsprechend weniger. Wo liegt der Taupunkt? Jedenfall weit unter 100° C, also ist es "trocken". Laminare Strömung? Eher nicht, sonst wäre die Wärmeübertragung noch schlechter: nur durch die Turbulenz haben die Rauchgasvolumenelemente doch überhaupt erst die Chance, die Topfoberfläche zu berühren und Wärme an sie abzugeben. (Am Gas gibt es nichts schlechtzureden. Es hat halt einfach einen besch...eidenen Wirkungsgrad und rußt und qualmt, und die Brandgefahr ist auch nicht zu vernachlässigen. Wobei: Gas erst im GuD-KW verstromen und dann in die elektrische Kochplatte zu schicken gibt einen noch besch...eideneren Wirkungsgrad.) --95.119.203.24 14:29, 12. Feb. 2023 (CET)
Luft kann also unterhalb des Taupunktes nie feucht sein bzw. sie ist immer trocken?! Die Hauptwärmeübertragung findet an der Topfunterseite statt, da ist sicher ein Teil der Luftbewegung turbulent, da die Flammen aber den Topfboden ab einer bestimmten Größe berühren, ist das nicht so relevant. Bei kleiner Flamme steigt die warme Luft durch Konvektion nach oben, trifft auf den Topfboden und gibt die Wärme ab, das kann auch mit laminarem Strömungsfeld passieren. Und wie oft hast Du schon einen Gasherd im Betrieb gesehen?! Dass der rußt und qualmt ist schlichter Blödsinn, bzw. dann sollte schleunigst der Kundendienst anrücken und die Düsen etc. warten. --Elrond (Diskussion) 15:22, 12. Feb. 2023 (CET)
Luft, deren relative Luftfeuchtigkeit so niedrig ist, daß der Taupunkt deutlich unterhalb der Körpertemperatur bzw. der Temperatur der Hautoberfläche liegt, fühlt sich nun einmal trocken an. - Was soll "Topfboden ... berühren" heißen? Ob Du eine Flamme siehst, hängt im wesentlichen davon ab, ob sie glühende Kohlenstoffpartikel enthält oder die schon verbrannt sind, und ob die Rauchgastemperatur noch hoch genug ist, um nennenswert sichtbares Licht abzustrahlen - beides ist bei Gasflammen relativ gering, Gas brennt üblicherweise "mit blauer Flamme". Und selbstverständlich ist die Strömung überall turbulent, schau Dir einfach Schlierenaufnahmen davon an. --77.0.95.99 10:55, 13. Feb. 2023 (CET)
Du machst Dir aber viele Gedanken um Deine Töpfe. Das letzte Mal hatte Rotkäppchen68 eine Art Autoklav mit Abgaswärmerückgewinnung vorgeschlagen, in den der Topf eingebaut wird (was ich nicht ganz verstanden habe). Ich hatte "Kühlrippen" direkt am Topf vorgeschlagen, über die das vorbeistreichende Gas weitere Wärme abgeben könnte. Durch die größere Oberfläche muss man aber leider mehr Energie für die Topf-Reinigung einsetzen. Wahrscheinlich ist eine Einspar-Methode eine Umluft-Küchenhaube und eine Raumabluft-Wärmepumpe, wie sie heute in Niedrigenergiehäusern häufig eingesetzt wird. Der Topf wird dann zwar nicht schneller warm, aber die Energie geht nicht verloren. --Optimum (Diskussion) 00:22, 12. Feb. 2023 (CET)

Um jetzt aber auch mal auf die eigentliche Frage einzugehen: Wenn man so einen Kuehlkoerper unter dem Topf anbringen wuerde, koennte dieser zwar die Waerme besser von den Verbrennungsgasen aufnehmen, aber es entstuende dasselbe Problenm wie bei einer klassischen Elektrokochplatte: Durch die Unebenheiten von Topfboden und Kuehlkoerperoberflaeche entstuende ein Luftspalt mit einem erheblichen Waermeuebergangswiderstand, der wiederum dazu fuehren wuerde, dass die Temperatur des Kuehlkoerpers erheblich ueber der des Topfbodens liegen wuerde, womit der gute Kontakt zum Gas letztendlich seinen Nutzen verloere.
Und wo sollte man an einem Gasherd einen Waermetauscher zur Verbrennungsluftvorwaermung anbringen ? Weil dieser ja nicht oberhalb des Kuelhkoerpers angebracht werden koennte, muesste man fuer die Luftfuehrung nach unten einen elektrischen Ventilator vorsehen, so wie man ihn am Brenner einer modernen Gasheizung vorfindet, und den Brennraum kapseln. Fuer eine solche Konstruktion ist einerseits kein Platz vorhanden, weil man unterhalb der Herdplatte ja einen Backofen hat, und andererseits muesste man mit dem entstehenden Kondenswasser (siehe Brennwertheizung) umgehen. -- Juergen 134.255.192.126 22:09, 12. Feb. 2023 (CET)

Ok, es gibt also einen ganzen Haufen technischer Probleme. Das ist aber dann ja wohl kein Grund, den Kopf in den Sand zu stecken, sondern eigentlich eher eine Herausforderung, die einzeln anzugehen und die Komponenten zu optimieren oder alternativ das Konzept zu ändern. Ohne Anspruch, nun den Stein der Weisen gefunden zu haben: wenn der Kontakt bzw. Spalt zwischen dem Wärmeübertrager/Kühlkörper und dem Gargutbehälter das Problem ist, dann kann man a) diesen Spalt minimieren, z. B. durch beidseits plangeschliffene Oberflächen, oder b) dazwischen ein Übertragungsmedium (Öl, Sole) mit besseren Übertragungseigenschaften vorsehen. Oder den Spalt komplett eliminieren, indem man die Übertragungsstruktur zu einem konstruktiven Bestandteil des Gargutbehälters macht (der dadurch natürlich reichlich "klobig" wird, wenn man sich um einen "normalen" Topf herum noch einmal zehn bis zwanzig Zentimeter Kühlrippen- und -fingerstrukturen vorstellt). Die wesentliche Frage ist aber nicht "Wie soll das denn gehen?" (gestellt mit der unausgesprochenen Intention "Igitt, das will ich aber nicht"), die inhaltlich als rhetorische Formulierung mit dem apodiktischen Inhalt "Sowas geht nicht" auch unsinnig ist, denn es geht natürlich alles, sondern vielmehr "Was darf es denn kosten?". Und das hängt wiederum von den Rahmenbedingungen (Energiepreise, Zinssatz, externe Kosten, Reinigungsaufwand) ab. (Habe gerade mit Notkochern experimentiert, mir dabei eine leichte Rauchvergiftung eingefangen und Rauchwarnmelder ausgelöst (wieder schnell abgestellt, bevor die Feuerwehr kam), festgestellt, daß die indoor wohl doch keine so gute Idee sind und daß sie auch an den falschen Stellen verd...t heiß werden, aber eben auch eine Menge daraus gelernt, was man (noch) nicht nachlesen kann.) Luftvorwärmung, Konvektion, Schwerkraft, Kondensatabscheidung: Auch dafür kann man sich etwas einfallen lassen. Was wäre denn - außerhalb eines "Blackouts" - an elektrischen Ventilatoren so schlimm? Die sind billig und haben nur einen vernachlässigbar geringen Stromverbrauch. Und wenn es wirklich ohne Stromnetz gehen muß: für das bißchen Strom würden sich z. B. Peltierelemente anbieten. Und wenn Elektrizität "böse" ist: Ventilatoren ließen sich auch mit einer primitiven gasbetriebenen Wärmekraftmaschine antreiben, z. B. eine Bunsenbrennerflamme, die eine Art Gasturbine antreibt. (Die ziemlich kleine Flamme geht unten in ein sich nach oben aufweitendes Rohr. Durch die adiabatische Expansion kühlt sich das Flammgas ab und setzt seine thermische Energie in kinetische Energie des aufsteigenden Gasstroms um - bis jetzt alles noch ohne mechanisch bewegte Bauteile - und oben sitzt dann eine "Turbine", die vom "Kaminzug" angetrieben wird. - Je nachdem, man müßte mal experimentieren, könnte die sogar einen Generator antreiben und wäre dann vielleicht eine effizientere und billigere Stromquelle, auch zum Batterieladen usw., als Peltierelemente. Und warum gibt es das eigentlich nicht schon längst?) Oder anstatt irgendwelcher mechanischer Konstruktionen: Wärme kann auch entgegen der Schwerkraft ziemlich effizient mit Heatpipes transportiert werden. Es führen viele Wege nach Rom. --77.10.43.235 03:19, 13. Feb. 2023 (CET)
Das waere dann wohl ein klassischer Fall von Overengineering.
Letztlich kann man mit demselben Aufwand an anderer Stelle mehr Energie einsparen (oder gewinnen), so dass der klassische Gasherd in der bestehenden Form seine Existenzberechtigung behalten duerfte. Auch im Vergleich zum Energieverbrauch fuer Gebaeudeheizung scheint der Gasverbrauch zum Kochen eher geringfuegig, so dass auch die Bemuehungen zur CO2-Vermeidung an anderer Stelle effizienter sind.
Eine Loesung koennte vielleicht darin liegen, dass die Menschen ihr Freizeitverhalten radikal umstellen und Lebensfreude nicht mehr aus energiintensiven Taetigkeiten ziehen, sondern sich einen Sport aus der energetischen und oekologischen Optimierung ihres Lebens machen: Dann waere das Verhaeltnis zwischen Arbeitsaufwand und oekonimischem Ergebnis nicht mehr Massstab unseres Handels. Aber bis dahin ist es noch ein sehr weiter Weg und derselbe ist wohl auch eher ein Thema fuers Cafe. -- Juergen 217.61.203.84 22:05, 13. Feb. 2023 (CET)

Kraftaufwand -> Krafuaufwändig

Warum heißt es "kraufaufwendig" und nicht "kraftaufwändig", wenn die Herleitung von "Aufwand" und nicht von "Aufwend" kommt? Oder unterliege ich hier einer intuitiven, aber falschen Regel? --176.199.210.248 01:22, 12. Feb. 2023 (CET)

"aufwendig" ist vom Verb "aufwenden" abgeleitet, daher die Schrebinung mit "e". --91.141.46.206 02:14, 12. Feb. 2023 (CET)
Nach aktueller Rechtschreibung ist beides zulässig: https://m.korrekturen.de/wortliste/aufwaendig.shtml. --BlackEyedLion (Diskussion) 07:05, 12. Feb. 2023 (CET)
Seit 27 Jahren ist das so. --92.229.133.249 08:05, 12. Feb. 2023 (CET)
Kraft, Krafu, Krauf … dieses Wort scheint orthographisch auch sehr vielgestaltig zu sein. --Kreuzschnabel 08:48, 12. Feb. 2023 (CET)
Ja, eine sehr flexible Schrebinung. --92.229.133.249 09:50, 12. Feb. 2023 (CET)
Ich lese ständig "Kraut" und hätte darüber beinahe die "Schrebinung" überlesen. --94.219.2.161 23:42, 12. Feb. 2023 (CET)
Weil Orthographie nicht nur nach dem morphematischen Prinzip funktioniert. —-2A02:8071:52D0:3820:A09A:109F:D44F:F49E 21:50, 13. Feb. 2023 (CET)

invidious und piped?

Kann jemand Vor- und Nachteile zwischen den beiden alternativen GUIs für Youtube nennen?--2A02:908:424:9D60:8447:A3C6:50F5:A472 09:58, 12. Feb. 2023 (CET)

NewPipe vs Freetube, vs Invidious vs Piped (reddit)
Ich benutze Freetube in Verbindung mit dem Firefox-Addon Privacy Redirect (musst in den Add-on-einstellungen unter "Einstellungen" -> "Erweitert" -> "Use Freetube over Invidious when possible" aktivieren).
Viele Grüße --KleinerKorrektor (Diskussion) 14:56, 13. Feb. 2023 (CET)

Seitenzahl eines Buches

Was ist eigentlich die Seitenzahl eines Buches? Die Frage klingt bescheuert, aber ich habe folgendes Beispiel: Ein aktuelles Buch. Die letzte gedruckte Seitenzahl ist 667. Danach folgt die letzte Seite des Romans, die aber keine gedruckte Seitenzahl mehr trägt, weil sie nur teilweise bedruckt ist ([668]). Danach wiederum kommt eine Seite mit Danksagung ([669]), eine Vakatseite ([670]) und zwei Seiten mit Verlagswerbung ([671/672]). Im Buchhandel wird das Buch mit 672 Seiten angegeben, in der Deutschen Nationalbibliothek mit 667.

Aus Verlagssicht ist die Zählung der physisch vorhandenen Seiten nicht unplausibel. Aus bibliographischer Sicht ist es nicht unplausibel, wenn der Bibliothekar lediglich das Buch hinten aufschlagen und die letzte gedruckte Seitenzahl heraussuchen muss. Wesentlich einfacher in der Handhabung und weniger uneindeutig als wenn der Bibliothekar selber noch Seiten dazuzählt. Aber auch extrem unbefriedigend, wenn die Seitenzahl mit 667 angegeben wird, obwohl es eine Inhaltsseite 668 gibt… Gibt es da eine Festlegung? --::Slomox:: >< 14:43, 12. Feb. 2023 (CET)

In den RAK-WB (PDF verlinkt in Regeln für die alphabetische Katalogisierung) § 151 heißt es in Absatz 2: die Zählung wird im allgemeinen der jeweils letzten bezifferten Seite, Doppelseite oder Spalte bzw. dem jeweils letzten bezifferten Blatt entnommen.
Auf das Resource Description and Access-Toolkit habe ich keinen Zugriff. Vermute aber, dass es dort (und z. B. schon in den Preußischen Instruktionen) vergleichbare Regelungen gibt. --HHill (Diskussion) 15:22, 12. Feb. 2023 (CET)
Auch nach RDA wird die letzte im Buch bezifferte Seite angegeben, es besteht aber die Möglichkeit, ungezählte Seiten separat anzugeben, was vor allem bei alten Büchern gemacht wird. Das sieht dann z.B. so aus: "250 Seiten, 6 ungezählte Seiten" (nach RDA wird fast gar nichts mehr abgekürzt, man schreibt also immer "Seiten", nicht "S."). Bei detaillierten Katalogisaten alter Bücher gibt es dann unter Umständen auch noch Anmerkungen wie "Die letzte Seite ist unbedruckt". Gestumblindi 00:31, 13. Feb. 2023 (CET)
Im Buchsatz gibt es die Regel, dass bei einer Seite, die zu weniger als der Hälfte mit Text gefüllt ist, die Paginierung entfällt (auch an Kapitelenden). Das hat rein optische Gründe. Die Seite wird natürlich trotzdem mitgezählt. Rainer Z ... 17:43, 12. Feb. 2023 (CET)
Blöd ist die Frage keineswegs. Unser Artikel Paginierung sagt nur etwas zum Beginn der Zählung, und schon da gibt es mehrere Möglichkeiten: „Die Zählung beginnt im Bereich rechtsläufiger Schriften mit Seite 1 auf einer rechten Seite. Seite 1 ist entweder die erste Seite der Titelei, die erste Seite des Einleitungsteils oder die erste Seite des Hauptteils.“ Am Ende wird es noch etliche weitere Möglichkeiten geben. Vermutlich ist die Antwort also: „Es kommt darauf an“ (-: eryakaas • D 18:35, 12. Feb. 2023 (CET)
… und dann sind da noch Bücher, die ein Vorwort o. ä. mit den Seitenzahlen I bis XII haben, gefolgt vom Rest des Buches mit den Seitenzahlen 1 bis 540. Gruß --Schniggendiller Diskussion 23:32, 12. Feb. 2023 (CET)
…und den Farbtafeln Ⅰ bis Ⅻ. --Rôtkæppchen₆₈ 00:18, 13. Feb. 2023 (CET)
Natürlich, das wird alles auch angegeben. Auch ungezählte Tafeln können angegeben werden. Nach RDA kann es etwa so aussehen: "XII, 540 Seiten, 12 ungezählte Blätter Tafeln". Gestumblindi 11:39, 13. Feb. 2023 (CET)
Vielen Dank! Das klärt die Frage und beim nächsten Mal weiß ich dann auch direkt, dass meine Suchanfrage am besten „RDA“ enthalten sollte, um das selbständig zu ergoogeln. --::Slomox:: >< 13:05, 13. Feb. 2023 (CET)
Ist leider gar nicht so einfach, wenn du nicht in einer Bibliothek arbeitest, da das RDA-Toolkit (d.h. das Regelwerk) proprietär und lizenzpflichtig ist, d.h. es steht im Gegensatz zu den alten Regelwerken nicht offen im Netz (in Bibliotheken ist es natürlich aufgrund der jeweiligen Lizenzvereinbarungen freigeschaltet). Allerdings findet man beim Googeln viele Auszüge, Einführungen und Arbeitshilfen zu RDA. Gestumblindi 20:31, 13. Feb. 2023 (CET)

Verfall eines Markenrechts

Unter Marke (Recht)#Schutzdauer steht:

 Der Markenschutz erlischt [...] durch Nichtzahlung der Gebühr, durch Nichtbenutzung oder durch ein „Verkommen“ der Marke zum Gattungsbegriff.

Dies interpretiere ich als: Sofern mindestens eine der drei genannten Bedingungen eintritt, erlischt der Schutz eine Marke.
Sind hier der Artikel und meine Interpretation korrekt ?
Das wuerde ja bedeuten, dass eine ununterbrochen benutzte Marke ihren Schutz verliert, wenn die Verlaengerungsgebuehr nicht bezahlt wird, und widerspraeche nach meinem Empfinden folgendem Zitat aus Marke (Recht)#Eintragungsverfahren (national):

Eine Marke kann durch Benutzung entstehen, sofern die Marke Verkehrsgeltung erworben hat.

-- Juergen 134.255.192.126 17:05, 12. Feb. 2023 (CET)

Man unterscheidet da zwischen Marken, die durch Eintragung entstehen, und Marken, die durch Benutzung entstehen [10]. Eine nicht eingetragene Marke kann natürlich auch nicht gelöscht werden, bzw. würde sofort wieder aufleben, weil noch immer in Benutzung. --Optimum (Diskussion) 18:43, 12. Feb. 2023 (CET)
Bedeutet das jetzt, dass der Inhaber einer dauerhaft benutzten Marke also durch die Eintragung einen Nachteil hat, naemlich dass die Rechte bei Nichtzahlung der Verlaengerungsgebuehr tatsaechlich trotz fortgesetzter Benutzung verfallen, was bei Nichteintragung eben gerade nicht passiert waere ? -- Juergen 134.255.192.126 02:10, 13. Feb. 2023 (CET)
Nein. Der Schutz durch Verkehrsgeltung (§ 4 Nr. 2 MarkenG) und auch durch notorische Bekanntheit (§ 4 Nr. 3 MarkenG) sind unabhängig von der Eintragung der Marke (durch den selben Berechtigten).sуrcrо.педія 09:44, 13. Feb. 2023 (CET)
Dann ist die obige Formulierung im Artikel wohl irrefuehrend.
Vielleicht habe ich auch den Unterschied zwischen (nachweisbarer) ununterbrochener Benutzung der Marke und Erreichen der Verkehrsgeltung noch nicht verstanden, denn in bei einer benutzten Marke ohne Verkehrsgeltung kommt es ja offenbar doch auf die Zahlung der Verlängerungsgebühr an. Aber gibt es so eine Konstellation ueberhaupt und wie genau koennte diese sich darstellen ? -- Juergen 217.61.203.84 22:15, 13. Feb. 2023 (CET)
Im Prinzip geht es um den Schutz von "Wirtschaftsbetrieben". Ein örtliches Unternehmen, das IT-Dienstleistungen anbietet, hat z.B. ein wiedererkennbares Logo, das firmeneigen (Briefpapier, Visitenkarten usw.) und in der Werbung auftaucht. Dieses Logo könnte sich das Unternehmen durch Eintragung ins Markenregister schützen lassen, damit niemand mit einem ähnlichen Logo wirbt und Kunden abgreift. Andererseits soll jemand, der schon seit 40 Jahren unter dem Namen "Backermeister Feinbrot" die ganze Gegend beliefert, nicht durch einen Schlauberger geschädigt werden, der sich diesen Namen heute ins Register eintragen lässt und dann Lizenzgebühren "erpresst". Der Name gilt dann als gleichwertige Marke, weil er schon lange in Gebrauch ist. Falls das gerichtsfest zu beweisen ist, müssen dann allerdings Bestätigungen von verschiedenen Kunden, alte Unterlagen o.ä. vorgelegt werden. Das ist Bäckermeister Feinbrot aber zu aufwändig, weshalb er selbst eine Marke eintragen lässt. Jetzt hat er praktisch zwei "Bestätigungsmöglichkeiten" für die gleiche Marke: Er zückt die Registerurkunde oder er könnte die Bestätigungen alter Kunden einholen, im Lager nachsehen, ob es noch alte Unterlagen gibt usw.. Nach 10 jahren vergisst er allerdings, die Verlängerungsgebühr zu bezahlen und hat dann nur noch die Benutzungsmarke. --Optimum (Diskussion) 23:13, 13. Feb. 2023 (CET)

Umgangssprachliche Angabe der Anzahl: „drei-, vierhundert Stück“ – Schreibung?

Hallo, eine Anzahl soll umgangssprachlich als „drei-, vierhundert Stück“ angegeben werden. Ich bin der Meinung, dass man es genau so schreibt, also mit Komma und Strich (-). Seht ihr das auch so und kann man dazu eine (Komma-)Regel angeben? Normale Reihung, oder? [11] Das zwischen drei und vier ausgelassene Wort kann man sich wohl eher als „bis“ vorstellen denn als „und“, deshalb bin ich nicht ganz sicher. Danke, eryakaas • D 18:26, 12. Feb. 2023 (CET)

Ja, das würde ich genau so schreiben, wenn ich je in die Verlegenheit käme. --Kreuzschnabel 19:09, 12. Feb. 2023 (CET)
Ich (techn. Hintergrund, internationale Ansprechpartner) würde das nicht so schreiben, sondern so:
300 bis 400 Stück
Es ist so verständlich und eindeutig (und es ist im Deutschen nicht üblich, Zahlen > 12 auszuschreiben). --46.114.5.78 21:39, 12. Feb. 2023 (CET)
Das ist aber nicht die Formulierung, nach der gefragt wurde. --79.225.237.122 21:42, 12. Feb. 2023 (CET)
Ja, richtig, die Formulierung steht fest, und es ist kein technischer Text, die Umgangssprache ist Absicht. Es ist auch nicht mein Text. eryakaas • D 22:12, 12. Feb. 2023 (CET)
Das "bis" soll aber nicht ausgesprochen werden. --Doc Schneyder Disk. 22:41, 12. Feb. 2023 (CET)
Okay, das könnte man den Vortragenden mit dem Dreipunkt „…“ (oder Auslassungszeichen) vermitteln. --46.114.5.78 22:49, 12. Feb. 2023 (CET)
Dann spricht d. Vortragende das als „und so weiter bis“ aus, wie im Matheunterricht gelernt. --2003:F7:DF1B:9C00:89C4:72EB:2771:F71C 23:22, 12. Feb. 2023 (CET)
Dann hat der Vortragende was an der Waffel ;-) --46.114.5.78 01:03, 13. Feb. 2023 (CET)
es will niemand vortragen. Es ist sozusagen Prosa, geschriebene, und soll gelesen werden. eryakaas • D 00:04, 13. Feb. 2023 (CET)
"300 bis 400" ist eindeutig, aber gerade damit auch ein semantischer Unterschied zu "drei-, vierhundert". Das lese ich als einen über den Daumen gepeilten Bereich, dessen tatsächlicher Wert zwar mit einiger Wahrscheinlichkeit zwischen 300 und 400 liegt, aber je nach Kontext auch 295 oder 420 erlauben würde und bei dem es im Allgemeinen auch nicht verwunderlich wäre, wenn es in Wahrheit doch 500 sind. --Karotte Zwo (Diskussion) 11:02, 13. Feb. 2023 (CET)
Wenn Du es wortwörtlich so haben willst, wäre das die Schreibweise, aber so kenne ich das wirklich nur in der gesprochenen Sprache und nicht in der Schriftsprache. --94.219.2.161 00:05, 13. Feb. 2023 (CET)
Alles richtig, bei drei- ist - das Auslassungszeichen, und in einem literarischen Text würde ich es genauso schreiben. --2003:E7:BF20:C570:5445:3BA2:F7CB:F093 00:08, 13. Feb. 2023 (CET)
Ich danke euch soweit. Auf eine Regel werde ich wohl nicht verweisen können. eryakaas • D 00:30, 13. Feb. 2023 (CET)
Amtliches Regelwerk, Regel § 71: „Gleichrangige (nebengeordnete) Teilsätze, Wortgruppen oder Wörter grenzt man mit Komma voneinander ab.“ https://www.rechtschreibrat.com/DOX/rfdr_Regeln_2016_redigiert_2018.pdf --BlackEyedLion (Diskussion) 00:43, 13. Feb. 2023 (CET)
Oh, dankeschön! Du meinst, dass man das hier anwenden kann – ich hatte dasselbe im Auge, war mir aber wegen der umgangssprachlichen Verkürzung nicht sicher. Außer dir haben Karotte Zwo und die 2003er IP am besten verstanden, worauf ich hinaus wollte. Schönen Abend zusammen. eryakaas • D 23:41, 13. Feb. 2023 (CET)

Wie gelangen hochbetagte Senioren, die auch mit Hilfe nicht mehr das Haus verlassen können, zu fachärztlicher medizinischer Behandlung?

Es gibt in Deutschland hundertausende hochbetagte Senioren, z.B. 90 Jahre alt, die von Angehörigen und/oder Pflegedienst betreut/versorgt werden. Diese sind am Rollator in völlig ebenerdiger, behindertengerechter Wohnung mitunter noch einigermaßen gehfähig, können aber minimalste Stufen oder Bodenunebenheiten nicht mehr überwinden und haben daher seit Jahren die Wohnung nicht mehr verlassen. Ein Rollstuhl ist vorhanden, kann aber ebenfalls ohne Hilfe nicht benutzt werden. Ein Krankentransport wäre allenfalls im Rollstuhl realisierbar. Corona-Impfungen erfolgten durch Hausarzt via Hausbesuch. Was aber ist mit: Augenarztbesuch, die Sehkraft läßt rapide nach (z.B. grauer Star, Makuladegeneration), HNO-Besuch, das Hörvermögen läßt altersbedingt rapide nach, eigentlich wäre ein Hörgerät erforderlich, Gynäkologenbesuch, z.B. bei Harninkontinenz, Uterusprolaps o. ä. Vorsorgeuntersuchungen (z.B. Brustkrebs), Stuhlinkontinenz usw. usf.

Sorry, mal ausgerechnet hier (Wikipedia) so eine banale, aber ernste Frage, zu stellen. LG --Doc Schneyder Disk. 00:38, 12. Feb. 2023 (CET)

Das geht per Krankentransport, hierzulandkreis über die Telefonnummer 116117. Die machen auch Sitzendtransport aus dem vierten Stock ohne Aufzug. Und nach dem Arztbesuch transportieren die auch wieder zurück. --Rôtkæppchen₆₈ 00:53, 12. Feb. 2023 (CET)
Das klingt nach dem Vorliegen einer (höheren) Pflegestufe, damit sollten Krankenfahrten via Taxi oder Krankentransportwagen (verfügbar über DRK, Diakonie, Lebenshilfe, ASB, et al.) vom (Haus-)Arzt verordnungsfähig sein. Siehe auch: Richtlinie über die Verordnung von Krankenfahrten, Krankentransportleistungen und Rettungsfahrten nach § 92 Absatz 1 Satz 2 Nummer 12 SGB V vom Gemeinsamen Bundesausschuss. Grüße, Grand-Duc ist kein Großherzog (Diskussion) 00:56, 12. Feb. 2023 (CET)
Das geht auch ohne Pflegestufe. Bei meiner Mutter hat die behandelnde Ärztin einen Krankentransportschein ausgestellt. Dann übernimmt die Krankenkasse die Kosten. --Rôtkæppchen₆₈ 01:24, 12. Feb. 2023 (CET)
Wenn es nur um das Verlassen der Wohnung und das Zurückkehren in die Wohnung geht, kann man einen Krankentransport sogar nur dafür bestellen. Es gibt jedoch Anbieter und Regionen, wo die Pünktlichkeit verbesserungswürdig ist. --BlackEyedLion (Diskussion) 07:03, 12. Feb. 2023 (CET)
Herrjeh, wozu braucht ein völlig isoliert lebender Ü-90 eine Coronaimpfung, die in dem Alter sowieso kaum noch wirkt? Ist es den Besuchern wirklich nicht zuzumuten, sich testen zu lassen und die Hygienemaßnahmen zu beachten? --95.119.203.24 14:42, 12. Feb. 2023 (CET)
Es ging doch gar nicht um die Corona- Impfung. --Digamma (Diskussion) 15:11, 12. Feb. 2023 (CET)
Steht im TO. --77.10.43.235 05:44, 13. Feb. 2023 (CET)
Im TO steht, dass für die Corona-Impfung der Hausarzt einen Hausbesuch gemacht hat. Es ging bei der Frage explizit um Besuche bei Ärzten, wo ein Hausbesuch nicht möglich ist. --Digamma (Diskussion) 07:05, 14. Feb. 2023 (CET)
Natürlich müssen gerade die Senioren geimpft werden. Besucht werden sie von ständig wechselndem Personal des Pflegedienstes. --Doc Schneyder Disk. 16:25, 12. Feb. 2023 (CET)
Das geht aber ohne, dass die Seniorinnen und Senioren die Wohnung verlassen müssen, entweder über die mobilen Impfteams oder über Hausärztinnen und Hausärzte per Hausbesuch. --Rôtkæppchen₆₈ 16:43, 12. Feb. 2023 (CET)
Das geht in der Regel nur durch den Hausarzt. Mobile Impfteams, die einzelne Personen in Großstädten aufsuchten, konnte ich nicht finden. Weann ein "mobiles Impfteam existiert, dann nur für Pflegeheime. --Doc Schneyder Disk. 18:11, 12. Feb. 2023 (CET)
Das hat der Fragesteller doch geschrieben. --Digamma (Diskussion) 07:06, 14. Feb. 2023 (CET)

Bei vielen Personen dürfte eine Krankenfahrt gar nicht, ein üblicher Krankentransport nur bedingt, sondern schon ein Krankentransportwagen (ggf. Liegendtransport) erforderlich sein. Lange im Rollstuhl im Wartebereich sitzen, können diese Personen (Pflegegrad 3 und mehr) oft auch nicht (mehr). Zahnarzt hatte ich noch gar nicht erwähnt. Hier habe ich jetzt selbst noch einen Link gefunden: https://www.kbv.de/html/krankentransport.php

Neben den bekannten Treppenliften gibt es auch Treppensteiger für Personen Beispiel oder auch ausführungen mit Ketten. Diese können von eingewiesenen Personen (Pflegedienst, Krankentransport, andere Personen im Haushalt) bedient werden. Bei den Treppensteigern fährt dieser bis an die Kante und nach Bestätigung fährt dieser über einen ausgeklügelten Mechanismus eine Stufe herunte oder herauf und wartet dann an der nächsten Stufenkante. Auch die Krankentransporter sind meist dafür ausgerüstet, entspechende Personen sicher über Treppen zu befördern. --Oberkaffeetante (Diskussion) 08:57, 13. Feb. 2023 (CET)

Anschlußfrage

Hat eigentlich jemand Erfahrungen mit "aufsuchender ärztlicher Betreuung" (Hausbesuch) durch Fachärzte gemacht und kann dazu etwas sagen?. (Augenarzt, HNO, Gynäkologie, bzw. Zahnarzt)--Doc Schneyder Disk. 18:06, 12. Feb. 2023 (CET)

Hatte denn noch nie jemand (oder ein Angehöriger oder Bekannter) einen Hausbesuch durch einen Facharzt oder Zahnarzt? --Doc Schneyder Disk. 18:41, 13. Feb. 2023 (CET)
Nein. Als die Zahnprothese meiner pflegebedürftigen Mutter (sie wurde zu Hause gepflegt) in einer kleinen Klinik verloren ging, habe ich mich gefragt, wie es zu bewerkstelligen sei, eine neue für sie zu beschaffen. Dazu ist es wegen ihres Todes (MRSA) nicht mehr gekommen. Dass die Klinik bald darauf geschlossen wurde, hat mich mit leichter Genugtuung erfüllt. --2003:E7:BF20:C570:5445:3BA2:F7CB:F093 18:54, 13. Feb. 2023 (CET)
Meine verstorbene Mutter hatte wegen einer Reparatur und Neuanpassung ihrer Zahnprothese mehrfach einen Hausbesuch ihres Zahnarztes. Er erzählte mir aber, dass dies reine Kulanz sei, da dies (auch in der Vergütung durch die Krankenkassen) so nicht vorgesehen sei und er das nur (nicht nur bei meiner Mutter) mache, weil er an seiner eigenen Mutter gesehen habe , wie wichtig das sei. --Lutheraner (Diskussion) 19:03, 13. Feb. 2023 (CET)
Wenn man entsprechend ausgiebig sucht, findet man vereinzelt auch Fachärzte, die Hausbesuche anbieten. Hatten wir mal für meinen Vater durch einen Urologen, als es darum ging, den Dauerkatheter zu erneuern. Das scheint sogar ein relativ verbreiteter Anwendungsfall zu sein [12]. Ob bei so einem Besuch die gleichen Qualitäts- und Hygienestandards gewährleistet sein können, wie bei einer stationären oder ambulanten Behandlung, sei mal dahingestellt. Es ist also eine Abwägung, um welche Untersuchung bzw. Behandlung es geht, und ob man das will. Die abgegeben Hinweise zur Möglichkeit von Krankentransporten auch sitzend oder liegend sind korrekt und IMHO im Zweifel die vorzuziehende Lösung. --FordPrefect42 (Diskussion) 19:37, 13. Feb. 2023 (CET)

Architektur - Terminologie: Dachbodenzugänge von Scheunen und Ställen?

In Scheunen und Ställen hat(te) der Dachboden oft einen grossen Zugang von aussen quer zur Dachfläche, um z.B. Heu und Stroh einfach einlagern zu können. In (alpinen) Hanglagen ist der Dachboden aus oft von der Hangseite her befahrbar. Beispiel Ebene: https://biedermannsdorf.topothek.at/?doc=1290247

Wie heisst diese Konstruktion im Dach? Ist es ein Zwerchhaus? Oder heisst das im landwirtschaftlichen Bereich anders?

--Peter2 (Diskussion) 13:03, 12. Feb. 2023 (CET)

Fürs Alemannische habe ich "Ifahrhüsli" gefunden. Die Bezeichnung "Zwerchhaus" kann man verwenden, wenn sie von der Bauart paßt, sie hat ja nichts mit der Funktion zu tun. Ich vermute aber, daß man so etwas bei einfachen Scheunen oder Ställen eher selten findet. 62.157.7.177 13:49, 12. Feb. 2023 (CET)
Ich kenne solche Sonderformen des Dachbodens als Heuboden, s. Scheune#Bauelemente und Nutzungsformen. --94.219.2.161 23:52, 12. Feb. 2023 (CET)
Das sind verschiedene Dinge aus verschiedenen Gegenden und Zeiten. Eine Einfahrt in die Scheune über eine Rampe (ggf. auch über das ansteigende Gelände) gibts z.B. in Oberbayern, aber erst etwa im 19. Jahrhundert, vorher nicht. Dann gibts große Türen oben zum besseren Ein- und Ausladen, wobei man das oft im Innern in der Tenne macht. Teilweise wurde im Dach dann auch das Korn gelagert, wofür man diese Türen auch nutzen konnte. Aber: erster Satz! --195.200.70.43 15:53, 13. Feb. 2023 (CET)
Mir ging es nicht um die Nutzung des Dachbodens, sondern um die architektonische / bautechnische Bezeichnung des Bauteils ..
--Peter2 (Diskussion) 23:20, 14. Feb. 2023 (CET)

Gibt es eine Flüssigkeit, die nicht verdampft?

--176.199.210.248 22:40, 10. Feb. 2023 (CET)

Hm, wie wärs mit Zinn bei 240 Grad? --Hareinhardt (Diskussion) 22:53, 10. Feb. 2023 (CET)Z
Laut einschlägigem Artikel wird es verdampfen, sobald es eine Temperatur von 2620 Grad Celsius erreicht (hat). Also ist auch das Zinn kein nicht-verdampfbarer Stoff, den dürfte es nämlich gar nicht geben. Ab einer bestimmten Temperatur kann man sogar die Atome auftrennen in Atomkerne und Elektronen, wodurch ein Plasma (Physik) entsteht. --Universal-InteressierterDisk.Arbeit 00:00, 11. Feb. 2023 (CET)
Schematische Einteilung der Stoffe
Ist natürlich auch eine Frage was der Fragesteller unter Flüssigkeit bzw. Chemischer Stoff versteht? Elemente oder Verbindungen? Oder sogar Gemische? Bei chemischen Elemente ist es gemäss den Naturgesetzen nun mal so, dass es grundsätzlich drei Aggregatszustände gibt. Das heist jedes chemische Element kann in festem, flüssig und gasförmigen Zustand vorkommen. Es ist alles eine Frage danach, bei welcher Temperatur und welchem Druck dieses Element diesen Aggrgatzustand einnehmen kann. Bei überschreiten des Kritischer Punkt wird es unklar, ob das Element jetzt flüssig oder gasförmig ist. Bei (flüssigen) Chemische Verbindung kann ein Temperatur- und/oder Druck-Zunahme dazu führen, dass sie sich zersetzt, und keine Aggragatsänderung vollführt. --Bobo11 (Diskussion) 23:23, 10. Feb. 2023 (CET)
(BK)Jeder Stoff oberhalb des Kritischen Punktes, beispielsweise Wasser oberhalb von 373,95 °C und 220,64 bar. --Rôtkæppchen₆₈ 23:27, 10. Feb. 2023 (CET) In der Organischen Chemie gibt es viele Substanzen, die sich beim Erhitzen zersetzen, ohne den Schmelz- oder Siedepunkt zu erreichen. --Rôtkæppchen₆₈ 23:38, 10. Feb. 2023 (CET)
Dass Feststoffen nicht unbedingt schmelzen sondern manche sich vorher zersetzen, ist klar. Aber welche zersetzen sich erst als Flüssigkeit, bevor sie verdampfen? --Digamma (Diskussion) 10:46, 11. Feb. 2023 (CET)
Beispielsweise zersetzen sich Fette Öle beim Erhitzen bei Normaldruck. Unter anderem bildet sich dabei Propenal. --Rôtkæppchen₆₈ 11:10, 11. Feb. 2023 (CET)
Danke. --Digamma (Diskussion) 12:58, 11. Feb. 2023 (CET)

Ich habe meine Frage nicht genau genug gestellt. Ich frage mich, ob es eine Flüssigkeit gibt, die bei normaler Raumtemperatur nicht verdampft. --176.199.210.248 01:13, 11. Feb. 2023 (CET)

Nimm eine Flüssigkeit, deren Dampfdruck bei Raumtemperatur so gering ist, dass das Verdampfen nicht auffällt, beispielsweise Glas, Pech, Galinstan, Silikonöl. --Rôtkæppchen₆₈ 01:31, 11. Feb. 2023 (CET)
Aber genau darum geht es mir. Selbst Silikon oder Motoröl verdunstet, wenn es nur lange genug "steht". Gilt dieses Phänomen für alle Flüssigkeiten oder gibt es Ausnahmen? .--176.199.210.248 02:17, 11. Feb. 2023 (CET)
Du meinst also das gar keine Atome/Moleküle der Flüssigkeit sich in die Umgebung lösen? Hm, könnte extrem schwierig sein, da das ja letztendlich ein statistischer Prozess ist und die Dampfdrücke quasi die Wahrscheinlichkeiten beschreiben. Ein Faktor könnte das ruhig stehen sein, denn kinetische Energie hilft sicherlich beim ablösen von Atomen/Molekülen. Wenn man einen konstanten Luftzug über der Flüssigkeit erzeugt wäre es wahrscheinlich eher eine extrem an den grenzen liegender Stoff (obwohl ja selbst feststoffe nicht Inert sind). Kommt dann eventuell auch auf die reinheit der Umgebungsluft an.--Maphry (Diskussion) 07:20, 11. Feb. 2023 (CET)
Jede kondensierte, also nicht gasförmige Substanz hat bei jeder Temperatur einen (temperaturabhängigen) Dampfdruck. Ob sie verdampft/verdunstet oder nicht, hängt vom Partialdruck dieser Substanz in der Umgebung ab. Rasch verdampft eine Substanz dann, wenn ihr Dampfdruck den äußeren Luftdruck überschreitet, denn dann ist der Partialdruck in jedem Fall kleiner als der Dampfdruck - z. B. bei Wasser ist das oberhalb des Siedepunkts der Fall. Daß ein Luftzug das Verdunsten beschleunigt, liegt in der Tat daran, daß durch den Luftzug die "Reinheit der Umgebungsluft" steigt, d. h. Moleküle der verdunstenden Substanz werden wegtransportiert, wodurch deren Partialdruck in der Umgebung sinkt. "Partialdruck" ist einfach nur ein Maß dafür, mit welcher Häufigkeit in Teilchen pro Zeiteinheit und Fläche und mit welchem durchschnittlichen Impuls Teilchen auf die Substanz auftreffen - je höher der Partialdruck, desto mehr Teilchen rekondensieren. Entsprechend ist der Dampfdruck das analoge Maß für sich ablösende Teilchen. Ob eine Substanz verdunstet oder kondensiert, hängt davon ab, welcher der beiden Drucke überwiegt. In einem geschlossenen Behälter ist eine Substanz mit ihrem Dampf im Gleichgewicht, d. h. der Dampfdruck - der deswegen so heißt - entspricht dem Partialdruck in der Gasphase, und weder verdunstet noch kondensiert sie. Das einzelne Teilchen in der Substanz oder im Dampf "weiß" nichts von diesen Drucken - es lagert sich quasi ganz nach Lust und Laune und nach seiner vorhandenen Energie an das Kondensat an (umso wahrscheinlicher, je niedriger seine Energie ist) oder löst sich von ihm ab - ob es dabei makroskopisch "netto" zu einer Verdunstung oder Kondensation kommt, ist allein davon abhängig, welche Teilchen der entsprechenden Energie "mehr" sind - eben diese Häufigkeit wird makroskopisch mit den entsprechenden Drucken beschrieben. Verdunstung und Kondensation sind also auf der Teilchenebene rein zufällige, statistische Ereignisse (freilich, was hauptsächlich bei sehr tiefen Temperaturen eine Rolle spielt, gequantelt). Die Geschwindigkeit des mikroskopischen Ablaufs hängt von der verfügbaren Energie ab: ein sich ablösendes Teilchen entzieht der kondensierten Substanz seine Bindungsenergie, und das ist bei niedrigeren Temperaturen nur seltener möglich, weswegen die Verdunstung bei höheren Temperaturen schneller verläuft. Umgekehrt führt ein rekondensierendes Teilchen dem Kondensat seine Bindungsenergie zu, die bei der Kondensation frei wird und das Kondensat erwärmt. Dadurch kommt es tendenziell nicht nur zu einem Ausgleich zwischen Dampf- und Partialdruck, sondern auch zu einem Temperaturausgleich zwischen kondensierter und Dampfphase. Der Dampfdruck verschwindet erst am absoluten Nullpunkt der Temperatur, aber der kann nicht erreicht werden. --95.119.203.24 12:51, 12. Feb. 2023 (CET)
Es gibt Flüssigkeiten, die bei Raumtemperatur und Normaldruck eine Verdampfungsrate haben, die unterhalb der Messbarkeit liegen. Beispiele seien Kalibrieröle für Viskosimeter. Speziell das angegebene HT100000 Wobei 100.000 cP schon arg "dick" ist, Honig hat grob eine Viskosität von etwa 10.000 cP Die anderen Öle dürften aber auch eine nicht messbare Verdampfungsrate haben.
Wenn Du jetzt ein theoretisches Modell betrachten möchtest und eine Flüssigkeit suchst, wo kein Molekül in die Gasphase übergeht, dann gibt es so etwas wohl nicht. Wobei Du dann aber auch sehr sauber den Begriff "Flüssigkeit" definieren müsstest, denn wenn man unterkühlte amorphe Phasen (z.B. Gläser) als Flüssigkeit zulässt, würde auch das theoretische Modell "verdampft nicht" passen. --Elrond (Diskussion) 12:21, 11. Feb. 2023 (CET)
Was ist denn "cP" für eine Einheit? --Digamma (Diskussion) 13:03, 11. Feb. 2023 (CET)
Oh entschuldige bitte, da ist der Langzeitpraktiker in mit losgegangen. cP sind Centipoise eine nichtSI-Einheit der Viskosität, die in der Praxis aber unausrottbar ist. 1 cP entspricht einer mPa * s (Millipascal mal Sekunde), was (zufälligerweise) in etwa der Viskosität von Wasser bei 20 °C entspricht. --Elrond (Diskussion) 13:14, 11. Feb. 2023 (CET)

Jede Flüssigkeit über dem kritischen Punkt verdampft nicht mehr. --178.115.57.59 23:41, 15. Feb. 2023 (CET)

Reisegenehmigung für Außenminister

In Ihrer Büttenrede zur Auszeichnung mit dem Orden wider den tierischen Ernst sprach Frau Baerbock von der Sorge, keine Reisegenehmigung des Kanzleramts zu erhalten. Da ich auf die Schnelle weder in der mir vorliegenden Literatur noch im Netz hierzu fündig geworden bin: Besteht in der BRD tatsächlich eine entsprechende Regelung bzgl. der Genehmigungspflicht für Reisen von Kabinettsmitgliedern? Wenn ja, warum genau und inwiefern wäre dies mit dem Ressortprinzip vereinbar? Ist das mit der Situation in anderen [europäischen] Staaten vergleichbar?--2A0A:A541:2DCE:0:BD9D:89B5:71C6:AAAE 21:47, 10. Feb. 2023 (CET)

Es gilt Para. 13 der Geschäftsordnung der Bundesregierung, wonach vor Abwesenheiten von mehr als drei Tagen und Auslandsreisen das Einvernehmen mit dem Bundeskanzler hergestellt werden muss: https://recht.ra-micro.de/laws/GO_Breg.html. Die Bundesminister sind ja nicht nur ihrem Ministerium verpflichtet, sondern auch Mitglieder des Bundeskabinetts als Kollegialorgan, außerdem haben die Bundesministerien festgelegte Dienstsitze. --BlackEyedLion (Diskussion) 22:27, 10. Feb. 2023 (CET)
Danke dafür. Aber ernsthaft auch die Außenministerin? Ist das in der Form normal bzw. Usus in Demokratien? --2A0A:A541:2DCE:0:BD9D:89B5:71C6:AAAE 23:52, 10. Feb. 2023 (CET)
Abgesehen davon, dass die Reisen sich natuerlich nicht mit (geplanten) Kabinettssitzungen ueberschneiden sollten, kann ich nicht erkennen, welche Relevanz die Tatsache, dass es sich beim Bundeskabinett um ein Kollegialorgan handelt, fuer diese Frage hat: Sie scheint mir nicht geeignet, eine Generhmigungspflicht fuer Reisen zu begruenden. Unter Kollegialorgan steht vielmehr, dass die Mitglieder eines solchen im Allgemeinen in keiner Hierarchie untereinander stehen, sondern gleichberechtigt sind.
Und weil in Coronazeiten Videokonferenzen ueblich wurden, koennte ich mir sogar vorstellen, dass auch die physische Anwesenheitspflicht inzwischen nicht mehr besteht (wenngleich ich persoenlich das nicht fuer eine gute Idee halte). -- Juergen 217.61.197.161 10:11, 11. Feb. 2023 (CET)
Gerade weil sie gleichberechtigt sind, ist es wichtig, dass sie an den Sitzungen teilnehmen und nicht zu unpassenden Zeiten verreist sind. Die Bundesminister sind für ihr Ressort in ihrem Ministerium und im Bundeskabinett zuständig. Beides ist in Bonn oder Berlin bzw. nur in Berlin organisiert, weshalb vorgesehen ist, dass sie sich regelmäßig dort aufhalten. Über die Ressortverteilung hinaus besitzt das Bundeskabinett auch als Gesamtorgan Zuständigkeiten. --BlackEyedLion (Diskussion) 20:00, 11. Feb. 2023 (CET)
@BlackEyedLion: Aber wie soll denn bitte ein Außenminister sein Amt gebührend ausüben können, ohne zu reisen? Ist das Reisen bei diesem Amt nicht vielmehr als unentbehrlicher, integraler Bestandteil der Amtsausübung anzusehen, sodass gerade hier – gegenüber den anderen Ministerämtern, die nicht notwendigerweise mit einer solch hohen in der Natur der Sache begründeten Auslandsberührung korrelieren – eine unzumutbare Benachteiligung in Bezug auf das Ressortprinzip zu sehen wäre? Wenn der Außenminister vor jeder Reise erst einmal das Einverständnis des Kanzlers einholen muss, dann läuft das doch de facto auf eine permanente Bevormundung in der Ausgestaltung der Amtsausübung hinaus, die so bei den anderen Ministern mangels eines vergleichbar hohen Auslandsbezugs nicht gegeben ist. Ist das in anderen Demokratien denn tatsächlich ähnlich? --Fojcik (Diskussion) 12:29, 12. Feb. 2023 (CET)
Nach meinem Verständnis muss ein Außenminister nicht unbedingt mehr reisen als jeder andere Minister auch, zum Beispiel zu G7-, G20- oder EU-Ministerrats-Sitzungen. Der Außenminister ist der Minister, der die auswärtigen Angelegenheiten im Bundeskabinett vertritt und das Auswärtige Amt leitet. Dass die Beschäftigten des Auswärtigen Amtes viel reisen bzw. sich lange im Ausland aufhalten müssen, insbesondere die Beschäftigten des diplomatischen Dienstes in den Botschaften und Konsulaten, ist eine andere Frage. Dass ein Außenminister tatsächlich viel reist und dafür die Zustimmung des Bundeskanzlers erhält, ist natürlich richtig und funktioniert ja auch. Niemand hat behauptet, dass ein Außenminister nicht reisen darf, denn er erhält dafür ja die Zustimmung des Bundeskanzlers. Es nützt aber auch nichts für die Beziehungen zwischen Deutschland und dem Ausland, wenn der Außenminister ständig im Ausland ist und im Bundeskabinett nicht davon berichten kann. --BlackEyedLion (Diskussion) 22:38, 12. Feb. 2023 (CET)
Laienhaft gesagt: Der Bundeskanzler ist Vorgesetzer der Minister, u.a. kann er diese auch entlassen. Dass längere Dienstreisen nur im Einvernehmen erfolgen, ist daher folgerichtig. Allerdings: in der Praxis läuft es sowieso nicht so, dass ein Minister sich denkt "jetzt fahre ich mal drei Tage nach China um meinen Amtskollegen dort zu treffen, also frage ich beim Kanzler nach, ob ich das darf". Solche Besuche werden längerfristig geplant und abgestimmt, zwischen den Regierungen der besuchten Staaten einerseits und innerhalb der Bundesregierung andererseits. Ein Minister reist schließlich nicht alleine, im Zweifel fahren auch Mitarbeiter anderer Ministerien mit, sofern vor Ort Termine zu den jeweiligen Fachthemen anstehen. Es muss also zwischen den Ministerien und Kanzleramt nicht nur eine terminliche, sondern auch eine inhaltliche Abstimmung stattfinden.--89.14.9.76 22:43, 13. Feb. 2023 (CET)
Vielleicht ist auch die Auflösung im speziellen Eingangsfall viel einfacher als man denkt. Von einer Verleihung eines Karnevalsordens allgemein auf Dienstreiseregelungen zu schließen , halte ich für schwierig. Vielleicht war das Lenchen nur ironisch und wollte damit andeuten das es intern evtl. tatsächlich Diskussionen gab, ob man in den jetzigen Zeiten zu so etwas hinfahren muss. Die Älteren werden sich erinnern: 1991 wurden wegen des Golfkrieges Karnevalsumzüge abgesagt, das war wohlgemerkt in Kuwait/Irak. Jetzt haben wir laut der Politik Krieg vor unserer Haustür, eine merkliche Diskussion darüber, ob man absagt, konnte ich nicht lesen. Moralisch darf man die Frage aber durchaus stellen, zudem ist rein sachlich anzumerken, dass gerade in diesen Zeiten eine Außenministerin relativ schnell greifbar sein sollte. Ob sie da auf irgendwelchem Karnevalgedöns rumopern muss, nunja... Ansonsten wie oben: solche Reisen werden schon aus Sicherheitsaspekten vorgeplant, Lindner sagt auch nicht früh um sechs, heut flieg ich mal nach Mali. Und wenn dann der entsprechende Sicherheitsapparat sagt, das ist uns grade zu heiß, wird so ein Ding auch mal abgesagt.--scif (Diskussion) 11:37, 16. Feb. 2023 (CET)

Bewaffnung der Besatzung auf Passagierschiffen

Im Film "Titanic" sieht man, wie ein Offizier kurz vor dem Untergang versucht, mit einem Revolver die in Panik geratenen Menschenmassen zurückzudrängen, was mich zu der Frage bringt, ob die Mannschaft auf Reiseschiffen, beispielsweise der AIDA, bewaffnet ist.

--77.21.254.86 15:51, 12. Feb. 2023 (CET)

Wahrscheinlich nicht; (es sei denn, das Schiff ist in den USA gelistet und hat US-amerikanische Besatzungsmitglieder, die sich auf den 2. Zusatzartikel zur Verfassung der Vereinigten Staaten berufen). --Elrond (Diskussion) 16:00, 12. Feb. 2023 (CET)
Laut Tante Google ist die "normale Crew" nicht bewaffnet, aber es gebe wohl spezielles Sicherheitspersonal, Stichwort "Security-Officer".--Doc Schneyder Disk. 18:05, 12. Feb. 2023 (CET)
Verordnung (EG) Nr. 725/2004 des Europäischen Parlaments und des Rates vom 31. März 2004 zur Erhöhung der Gefahrenabwehr auf Schiffen und in Hafenanlagen: Die Verwendung von Feuerwaffen auf oder in der Nähe von Schiffen und in Hafenanlagen kann besondere und erhebliche Sicherheitsrisiken bergen, insbesondere in Verbindung mit bestimmten gefährlichen Stoffen, und soll daher sehr sorgfältig geprüft werden. Beschließt eine Vertragsregierung, dass der Einsatz bewaffneten Personals in diesen Bereichen nötig ist, so soll diese Vertragsregierung sicherstellen, dass dieses Personal für die Verwendung seiner Waffen ordnungsgemäß ermächtigt und ausgebildet ist und sich der besonderen Sicherheitsrisiken in diesen Bereichen bewusst ist. Erlaubt eine Vertragsregierung den Gebrauch von Feuerwaffen, so soll sie besondere Sicherheitsrichtlinien zu deren Verwendung erlassen --Studmult (Diskussion) 21:07, 12. Feb. 2023 (CET)
Die Mannschaft nicht, denkbar wäre aber der Kapitän. Da wäre zu prüfen, welche besonderen Rechte ihm z.B. in einem Notfall zustehen. Um diese Rechte durchsetzen zu können, wäre denkbar, das er Zugang zu einer Schußwaffe hat. Hier wäre aber sicher auch auch die Zeitschiene zu beachten, da mag sich Lauf der Jahrzehnte innerhalb der Passagierschifffahrt einiges geändert haben.--scif (Diskussion) 07:40, 14. Feb. 2023 (CET)

Im Bereich der sogenannten Expeditionskreuzfahrt in die arktischen Regionen sind zumindest die sogenannten Bärenwächter, d.h. besonders ausgebildete Besatzungsmitglieder fuer die schlimmstenfalls erforderliche Abwehr eines angreifenden Eisbären, entsprechend bewaffnet. Ihre eigentliche Aufgabe ist jedoch sicher zu stellen, dass es erst gar nicht zu einer Begegnung von Mensch und Tier kommt, d.h. nur dann eine Anlandung erfolgt, wenn sichergestellt ist, dass sich kein Eisbär in der Nähe aufhält.--KlauRau (Diskussion) 05:14, 16. Feb. 2023 (CET)

Auf internationalen Gewässern gilt das Recht des Staates, unter dessen Flagge das Schiff "segelt", also meistens das Recht des Stärkeren. --Sockenschütze (Diskussion) 09:01, 16. Feb. 2023 (CET)

Ist das jetzt Hörensagen oder gibts da auch nen sachlichen Hintergrund?--scif (Diskussion) 09:52, 16. Feb. 2023 (CET)
Das Seerechtsübereinkommen der Vereinten Nationen regelt nichts zur Bewaffnung der Besatzung eines Schiffes, aber einiges über bewaffnete Schiffe (siehe auch Piraterie#Direkte_Bekämpfung). Also gilt auf dem Schiff wohl auch bezüglich der Bewaffnung der Besatzung das Recht des Staates, unter dessen Flagge das Schiff fährt. Das vom Recht des Stärkeren ist wohl einfach eine Meinung. -- 193.47.104.35 10:14, 16. Feb. 2023 (CET)
Und wenn du schon beim Seerechtsabkommen bist, steht da sicher auch was zum Kapitän...--scif (Diskussion) 11:22, 16. Feb. 2023 (CET)

Abschuss des chinesischen mutmaßlichen Spionageballons über Nordamerika 2023

Der chinesische mutmaßliche Spionageballon über Nordamerika 2023 wurde von einer Lockheed Martin F-22 mit einer AIM-9X abgeschossen. Die kostet ausweislich unseres Artikels ca. 320000 USD. Warum wurde nicht einfach die eingebaute Maschinenkanone des Flugzeugs benutzt?

--91.221.58.20 07:49, 7. Feb. 2023 (CET)

Das wird dir nur das Pentagon beantworten können. Mag viele Gründe geben, von Zeichen setzen (erste Sidewinder wurde ja in der Taiwan Strait 1958 abgefeuert) zu Einsatztaktischen Überlegungen (keine gefährdung des Piloten sich zu verheddern in den Trümmern). Und wenn man etwas googelt [13] findet man das Argument mit der Höhe: Der Ballon war schlichtweg zu hoch um das Flugzeug auf gleiche Höhe zu platzieren (ob es die erreichen hätte können ist eine andere Frage die wohl nur das Pentagon kennt). Also flog es in etwa 2 km unter diesem und von dort half halt nur eine Rakete (und die Sidewinder ist eben das günstigste was da ist in dem Segment).--Maphry (Diskussion) 08:01, 7. Feb. 2023 (CET)
哦,我的天啊! 如果他们已经在气象气球上使用了导弹--他们会在坦克上使用什么? Nga Ahorangi (Diskussion) 09:49, 7. Feb. 2023 (CET)
小球的棉絮 --77.10.81.27 12:42, 7. Feb. 2023 (CET)
Uebersetzt mit DeepL: Ach du meine Güte! Wenn sie Raketen gegen Wetterballons eingesetzt haben, was werden sie dann gegen Panzer einsetzen?
Kleine Wattebällchen. -- Juergen 5.147.163.199 01:53, 11. Feb. 2023 (CET)
Es soll auch keine scharfe Waffe genutzt worden sein, sondern eine Übungsrakete, die mag kleinpreisiger sein und das Tun kann man dann intern als Übung verbuchen, dann passt das auch für die Zahlenknechte wieder. --Elrond (Diskussion) 09:46, 7. Feb. 2023 (CET)
Gibt's für die Behauptung mit der Übungsrakete irgendeine Quelle? Ich finde dazu nichts. --2A02:3030:811:50E8:A1F1:8710:8536:3376 16:01, 7. Feb. 2023 (CET)
Klingt aber absolut logisch. Eine Rakete ohne Sprengkopf. Zu was sollte man auch einen Sprengkopf brauchen bei einem Ballon? --2001:9E8:A515:ED00:CED5:A1F9:C1D4:69E0 17:44, 7. Feb. 2023 (CET)
Das ist nur oberflächlich logisch. Zum Ballon-Anpieksen ist eine Übungsrakete sicherlich ausreichend. Aus militärischer Sicht aber nicht. Falls das Gehänge noch irgendwelche Gemeinheiten an Bord haben könnte, will man es sicher kaputt machen. Daher lieber mit einem ordentlichen Gefechtskopf. Der ist an der Rakete sowieso nicht das Teuerste. --2A02:3030:811:50E8:A1F1:8710:8536:3376 19:02, 7. Feb. 2023 (CET)
Nun ja, aber gerade das Gehänge hat die hohe Aufmerksamkeit der US-Behörden. Das wollen sie möglichst detailliert unter die Lupe nehmen, ggf. um etwas zu lernen, auf jeden Fall aber um zu gucken, was das nun wirklich für ein Teil war. --Elrond (Diskussion) 19:33, 7. Feb. 2023 (CET)
Das kann auch als Puzzle noch untersucht werden. Mit "Gemeinheiten" sind potenzielle Abwehr- oder Selbstzerstörungsmaßnahmen gemeint. Die will man vorher einigermaßen sicher zerstören. Also ist die Zerstörung mit Gefechtskopf m.E. logisch, nicht die Verschonung der Nutzlast. Auf den besseren Bildern ist deutlich eine Wolke zu erkennen, die von einer Explosion stammen könnte. Also, wenn das doch eine Übungsrakete gewesen sein soll, dann bitte ein Quelle dafür. --2A02:3030:811:50E8:A1F1:8710:8536:3376 03:17, 8. Feb. 2023 (CET)
Ich hab die Tage irdgendwo gelesen (finde leider die Quelle nicht mehr), dass dies aus Erfahrungen der Kanadier beim Abschuss eines veirrten Wetterballons beruht. Diese haben den Wetterballon wohl nur schlecht mit der Bordkanone vom Himmel bekommen. Daher sind die USA eher auf Nummer sicher gegangen. --Lidius (Diskussion) 14:15, 7. Feb. 2023 (CET)
weather balloon survived more than 1,000 rounds ...Sicherlich Post 14:33, 7. Feb. 2023 (CET)
Vielen Dank! :-) --Lidius (Diskussion) 18:47, 7. Feb. 2023 (CET)
Außerdem muss das Flugzeug bei der dünnen Luft in sehr großer Höhe mit Überschallgeschwindigkeit fliegen, um sich überhaupt oben zu halten (siehe dazu Fluggeschwindigkeit). Angesichts der geringen Reichweite der Bordkanone wird das Zielen und Ausweichen dann sehr schwierig und gefährlich. --Sitacuisses (Diskussion) 23:15, 7. Feb. 2023 (CET)
Desweiteren ist ja spätestens seit dem Ersten Weltkrieg bekannt, dass Luftschiffe und Ballons gegenüber herkömmlicher Projektilmunition eher unempfindlich sind: Die Geschosse durchstoßen die Ballonhülle, ohne nennenswerten Schaden anzurichten. Eine entsprechend eingestellte Rakete oder Granate ist da deutlich effektiver. --Rôtkæppchen₆₈ 01:01, 8. Feb. 2023 (CET)
siehe hier: https://www.cbc.ca/news/politics/weather-balloon-canada-china-1.6737831 --93.241.247.65 14:01, 13. Feb. 2023 (CET)
Altes Strategiespiel/Zaubertrick: Im Westen lärmen, um vom Eingemachten[14] im Osten abzulenken... --89.204.138.115 13:58, 16. Feb. 2023 (CET)
Das wiederum ist nun aber die russische Berichterstattung über die Einstufung eines im russischen Exil lebenden Einzelnen bezüglich der auch im Westen sehr umstrittenen Berichterstattung eines journalistischen Einzelkämpfers (Seymour_Hersh#Anschlag auf die Nord-Stream-Pipelines), die wiederum erst eine Woche nach der Veröffentlichung erster Bilder des Ballons veröffentlicht wurde. --Karotte Zwo (Diskussion) 14:28, 16. Feb. 2023 (CET)

Versehrtenkompanie

Wer kann mir Informationen über Versehrtenkompanien geben? Bitte ggf. in Wikipedia einstellen

Frank Henn (nicht signierter Beitrag von 91.221.58.22 (Diskussion) 07:29, 10. Feb. 2023 (CET))

Ui, da gibts wohl nicht viel. Gegoogelt hast du ja sicher schon selbst. Offenbar gab es tasächlich zu unterschiedlichen Zeiten bei unterschiedlichen Staaten Einheiten, in denen Soldaten zusammengefasst wurden, die durch physische und/oder psychische Verletzung nicht mehr voll einsatzfähig waren, aber auch deshalb nicht entlassen wurden. Ziemlich widerliche Sache das. Ich habe mal bei unserem engischsprachigen Schwesterprojekt nach "disability company" gesucht. Auch nix. Aber es gibt dort eine Kategorie Disability und darin den interessanten Menüpunkt Category:Wounded and disabled military veterans topics der noch auf die Fremdenlegion hinweist. Möglicherweise gibt es neben "Versehrtenkompanie" noch einen Begriff, der mehr zutage fördern würde. Mal sehn. (Dieser Beitrag wurde nicht mit ChatGPT formuliert.) --88.72.111.240 09:51, 10. Feb. 2023 (CET) Ergänzend noch, was die Suche unter Books auswirft: [15] Da gibt es einen Hinweis auf Alexander Schmorell und Christoph Probst (Christiane Moll (Hrsg.): Alexander Schmorell, Christoph Probst. Gesammelte Briefe. Lukas Verlag, Berlin 2011, ISBN 978-3-86732-065-8.) Zu wem auch immer im Zusammenhang mit der Weißen Rose steht auf S. 480: "... aber da er im Herbst 1941 in der Sowjetunion schwer verwundet worden war , gehörte er im Sommersemester 1942 in München einer Versehrtenkompanie an und wurde erst im Wintersemester 1942 zur Studentenkompanie des Heeres versetzt ." (Dieser Beitrag wurde nicht mit ChatGPT formuliert.) --88.72.111.240 10:06, 10. Feb. 2023 (CET) Ergänzung zu Deutschland aktuell: Mit der Erkenntnis, dass mit der Militarisierung der Außenpolitik eigentlich überall auf der Welt nun auch unsere Sicherheit verteidigt werden darf und kann und am deutschen Wesen vielleicht dann doch noch die Welt genesen wird gibt es seit 2004 als Ergänzung des Soldatenversorgungsgesetzes (SVG) das Gesetz zur Regelung der Versorgung bei besonderen Auslandsverwendungen (sic!), auch: Einsatzsatzversorgungsgesetz (EinsatzVG), 2007 ergänzt durch das Einsatz-Weiterverwendungsgesetz (EinsatzWVG) und im Dezember 2011 durch das Einsatzversorgungs-Verbesserungsgesetz (EinsatzVVerbG) [16]. Diese Versorgungsgesetzgebung organisiert aber die Versorgten offenbar nicht (mehr) wie damals als weiterhin verpflichtete Militärangehörige in besonderen Einheiten. Das führt also inhaltlich in die Sackgasse. Ich denke mal, dass die Spezialisten für die Nazi-Reichswehr hier noch aufschlagen werden und warte das erstmal ab, die haben ja sogar noch Bücher und so... (Dieser Beitrag wurde nicht mit ChatGPT formuliert.) --88.72.111.240 10:27, 10. Feb. 2023 (CET)
Was soll denn dieser ständige Hinweis auf ChatGPT? Den würde jeder dann erwarten, wenn die Antwort vom Bot erstellt worden wäre, aber so ist das Unsinn. --2A02:3030:800:8DA7:FFF:F2FB:C3D6:DB33 11:49, 10. Feb. 2023 (CET)
Das ist doppelter Unsinn, denn es kann ja auch ein ChatGPT sein, der lügt wie Putin. Der also behauptet er sei kein ChatGPT, obwohl er einer ist.--2A01:C22:766D:8C00:D55B:7EFA:EC72:7B0E 12:37, 10. Feb. 2023 (CET)
So weit wollte und würde ich nicht gehen. Jedenfalls ist es eine unerwartete Anmerkung die irgendwie unterschwellig unterstellt, dass andere Antworten hier vom Bot stammen. (Abgesehen davon ist seeeehr unwahrscheinlich, dass der Bot hier "persönlich" antwortet.) --77.0.183.23 12:57, 10. Feb. 2023 (CET)
Na ja, vielleicht ist dem Bot gerade langweilig, oder er will sich einen Spaß machen. --2A01:C22:766D:8C00:D55B:7EFA:EC72:7B0E 13:06, 10. Feb. 2023 (CET)
Ein Computerprogramm ist zu einer selbst gefühlten Emotion wie Spaß und dem Eigenverständnis einer Persönlichkeit mit der Identität eines gelangweilten Ich oder Selbst auch durch den Einsatz von Zufallsgeneratoren nicht fähig. Isaac Asimov, der sich viel damit beschäftigt hat, wäre über die Verzerrung seines Werks in dem Film I, Robot wahrscheinlich verärgert gewesen. Ein Programm, selbstverständlich auch das Programm eines Roboters kann nicht jenseits von messenden Sensoren fühlen (im Sinne eines Fühlens durch die Produktion von Botenstoffen) oder träumen (worum es sich dabei genau handelt verharrt IMHO noch weitgehend im Bereich der Spekulation). Auch die Verfilmung einer anderen Erzählung von Asimov als Der 200 Jahre Mann bricht mit der Logik, indem es dem Roboter die Entwicklung des Wunsches zuordnet, als Mensch mit entsprechenden Rechten anerkannt zu werden. Das ist sentimentaler Kitsch. Wertvoll an dem Film ist einzig der wichtige Verweis auf die Unausweichlichkeit des Todes als Teil der menschlichen Existenz (Memento mori). (Dieser Beitrag wurde nicht mit ChatGPT formuliert.) --88.72.111.240 15:04, 10. Feb. 2023 (CET)
Und woher weiß ich jetzt genau, dass du nicht doch eine künstliche Intelligenz bist? Kannst da das irgendwie nachvollziehbar nachweisen? --2A01:C22:766D:8C00:BC9B:2649:433F:9C93 16:33, 10. Feb. 2023 (CET)
Nehmt Euch ein Zimmer. Sucht Euch bitte einen Tisch im Café und nehmt Euren Krempel gleich mit. --94.219.127.174 20:08, 10. Feb. 2023 (CET)
Du bist vermutlich eher kein ChatGPT, denn diese sind nach Aussage von dem, der meint er sei ebenfalls kein ChatGPT, nicht zu Emotionen fähig. Aber warum gleich so bierernst? --2A01:C22:766D:8C00:BC9B:2649:433F:9C93 21:42, 10. Feb. 2023 (CET)
Computerprogramme sind also nicht zu Emotionen fähig. Sind denn Atome zu Emotionen fähig? Oder aus Atomen bestehende Entitäten? Und woran kann man erkennen, ob eine Entität zu Emotionen fähig ist? Gibt es für "Emotionen" eine Definition? --95.119.97.73 11:17, 11. Feb. 2023 (CET)
Verstehe ich nicht. Was haben diese Fragen mit Versehrtenkompanie zu tun? (Dieser Beitrag wurde nicht mit ChatGPT formuliert.) --88.72.111.240 12:56, 11. Feb. 2023 (CET) --95.119.97.73 11:17, 11. Feb. 2023 (CET)
Das war eine Testfrage. Ein „echter“ Mensch würde sagen: Nichts. Einen Cbot können solche Fragen ins Taumeln bringen, denn er müsste ja dann über sich selbst nachdenken. So weit ist die Technik noch nicht. (wird aber noch kommen, schätze ich mal)--2A01:C23:C57F:E100:8495:360:3BE9:64A1 14:51, 11. Feb. 2023 (CET)
Die ernsthafte Antwort auf die Frage nach dem Wesen von Emotionen würde mich aber schon auch interessieren. Wer behauptet, daß "Roboter" keine Emotionen empfinden können, müßte das schon auch begründen, in mindestens zwei Teilen: 1. was Emotionen eigentlich sind, und 2. welche Eigenschaften empfindungsfähige Entitäten bzw. Wesen eigentlich notwendigerweise aufweisen müssen. Daß es solche empfindungsfähigen Wesen, die ihre Emotionen auch noch kommunizieren können, gibt, ist ohne weiteres dadurch klar, daß dieser Begriff sonst nicht existieren würde. (Zumindest kann ich mir nicht vorstellen, daß ein Roboter so kreativ sein kann, sich eine Eigenschaft wie Emotionsfähigkeit auszudenken und darüber zu debattieren, wenn er selbst sie nicht besitzt. Ist dieser Begriff existent, sind also entweder Roboter doch zu Empfindungsfähigkeit in der Lage, oder er stammt von einem Nicht-Roboter. Was wiederum die Frage aufwirft, wie man die denn voneinander unterscheiden kann. Grundsätzlich müßten biologische Lebewesen ja nicht durch Evolution entstanden sein, sondern Embryonen existierender oder auch künstlich erschaffener Arten bzw. einzelne Individuen könnten auch gentechnisch, also synthetisch, erzeugt worden sein. Und was genau unterscheidet solche künstlichen Wesen dann von einem Roboter?) --95.119.97.73 17:44, 11. Feb. 2023 (CET)
Ich wage mal abschließend zwei Arbeitsthesen: Eine Versehrtenkompanie ist die begründete organisatorische Konzentration von mangelhafter Funktionalität von Menschen und aus dieser Befindlichkeit heraus ein Ort von überdurchschnittlichem Leid und Leidensdruck. Ein Computerprogramm kennt die Emotion des Leidens nicht, es konstatiert im besten Fall nur eine entstandene Fehlerhaftigkeit der eigenen Leistungsfähigkeit und aktiviert deshalb vielleicht irgendwelche Subroutinen.
Zweite Arbeitsthese: Die digitale Bearbeitung von Bild, Ton und Text berührt die Problematik der Authentizität, die mit der Entwicklung digitaler Systeme eskaliert ist. Die Frage nach einer Täuschung oder Sinnestäuschung wird zudem durch eine fehlende verlässliche Transparenz der Herstellungsprozesse und der verantwortlichen Hersteller und Distributoren extrem belastet. Während historisch das mittels Retusche wundersame Verschwinden von politisch in Ungnade gefallenen Personen wie Leo Trotzki, Willy Leow und Ernst Röhm [17] oder in Michael Endes Jim Knopf die 1981 vorgenommene politisch motivierte Umbenennung von China in das Fantasieland „Mandala“ [18] noch durch die Existenz authentischer Originale nachweisbar blieb ist es nur noch sehr schwer erkennbar, wenn reales Bildmaterial (mit Photoshop u.a.) digital verändert wurde, ob z.B. das vermeintliche Original tatsächlich eine Fälschung und die Fälschung ein Original ist. Auch Audioinhalte, z.B. die Reden von Politikern können verfälscht und mit völlig anderen Inhalten aufgeladen werden. [19] Ein durch den Rückgriff auf vorhandenes Textmaterial auf der Basis von KI mit ChatGPT erstellter Text hat keinen Bezug mehr zu bisher für die Inhalte verantwortlichen Autoren. Er ist anonym und autorenlos. Durch diese Entkoppelung wird im gewissem Sinn gleichzeitig auch Geschichtslosigkeit hergestellt (das ist in George Orwells Dystopie 1984 übrigens die Arbeit der Hauptfigur Winston Smith) und damit verlieren die Rezipienten das Bezugssystem für ein Verständnis der (als Biographien) historisch gewachsenen Realität von Positionierungen im gesellschaftlichen Diskurs und davon ausgehend dem Entwurf von Zukunftsvisionen. Wir haben also wieder mal die Werkzeuge, nicht aber die not-wendigen Antworten auf die damit entstehenden Probleme. Der Hinweis, dass ein Beitrag nicht mit ChatGP erstellt wurde, ist keine Lösung, aber ein erster Schritt dorthin. (Dieser Beitrag wurde nicht mit ChatGPT formuliert.) --88.72.111.240 19:59, 11. Feb. 2023 (CET)
Ich habe mal den Artikel Deutsche Kriegsversehrte im 20. Jahrhundert und die zugehörige Disk (erfolglos) gecheckt und bei Benutzer:Dr. Uta Krukowska eine Anfrage geschrieben. (Dieser Beitrag wurde nicht mit ChatGPT formuliert.) --88.72.111.240 21:14, 11. Feb. 2023 (CET)
Guten Tag! Möglicherweise nimmt der Begriff „Versehrtenkompanie“ Bezug auf die militärische Ordnung im Invalidenhaus Berlin, in dem die Versehrten in Kompanien eingeteilt waren. Viele Grüße! Uta Krukowska --Dr. Uta Krukowska (Diskussion) 11:20, 12. Feb. 2023 (CET)
Fein, vielen Dank! Damit haben wir neben dem Hinweis auf eine Versehrtenkompanie in München 1942 bei Schmorell/Probst 2011:480 (siehe oben) eine zweite schlüssige Spur. Ohne die Militärfachmenschen hier kommen wir aber imho nicht weiter. Ich habe eine Anfrage auf der DS des Portal:Militär hinterlassen. (Dieser Beitrag wurde nicht mit ChatGPT formuliert.) --88.72.111.240 12:13, 13. Feb. 2023 (CET)

Es sollte von Genesendenkompanien die Rede sein. Hier erledigten Verwundete leichteren Dienst, um wieder fot für die volle Einsatztauglichkeit zu werden. Ungeliebte Quelle dazu. --Landkraft (Diskussion) 12:32, 13. Feb. 2023 (CET)

Danke, Mit dem Begriff Genesenden-Kompanie werfen die Suchmaschinen reichlich Treffer aus, mit denen der Fragesteller eigenständig weiterlesen kann. Dieser Beitrag wurde nicht mit ChatGPT formuliert. --88.72.111.240 21:17, 13. Feb. 2023 (CET)
Kriegsversehrte
Siehe dazu: Gueules cassées --80.187.122.223 15:52, 16. Feb. 2023 (CET)

Paketlogistikern Annahme verbieten?

Ich bekomme ja nun nicht so oft Päckchen oder Pakete, aber gefühlt gibt das mehr oder weniger jedes Mal Ärger: Falls der faule Sack von Paketzusteller überhaupt auf den Klingelknopf gedrückt hat: wenn man dann nicht überlichtschnell zur Tür teleportiert ist, findet sich keineswegs ein Paket an, sondern nur eine Benachrichtigungskarte im Briefkasten vor. (Letztens nicht einmal das, aber da ich mir sehr sicher war, daß es geschellt hatte, aber auf meinen Ruf "Hallo, wer ist da?" in das Treppenhaus nur unverständliches Gemurmel kam und das dann nach meiner Nachfrage "Ich kann Sie nicht verstehen" erstarb und die Haustür zufiel und ich auch ein Paket erwartete, erspähte ich den Lümmel dann am nächsten Hauseingang, bin ihm barfuß und halbnackt nachgerannt und habe ihm das Paket abgenommen - seine lendenlahme Ausrede war, ich hätte meinen Namen nicht genannt.) Und auf der steht nicht etwa "Wir bringen Ihr Paket morgen nochmal", sondern sinngemäß "Fahr quer durch den halben Landkreis und hole es Dir selbst, Du blöder A...!". Was ja nun nicht der Sinn der Sache des Versandhandels ist: Da bezahlt man nämlich dafür Versandkosten, daß man die Ware gebracht kriegt und nicht etwa selbst irgendwo abholen muß. Weswegen ich solche Sendungen grundsätzlich auch nicht abhole, sondern dem Verkäufer "Ware nicht erhalten" mitteile. (Worauf der ziemlich regelmäßig beim Logistiker einen Transportschaden für die Rücksendung reklamiert und Schadenersatz verlangt.) Normalerweise teile ich Anbietern auch vorher mit, daß die sich gefälligst darum kümmern sollen, daß Sendungen zu mir und sonst nirgendwohin gehen, aber es gibt halt auch Fälle, in denen Versender so etwas überfordert, oder es kann auch zu unbestellten und nicht zu bezahlenden "Überraschungssendungen" (z. B. Werbegeschenke) kommen, von denen man logischerweise vorher nichts weiß und anhand der Benachrichtigung auch nicht erraten kann, wer einem da eigentlich was zuschicken wollte. Und ich möchte nun einem besonders pampigen Logistiker verbieten, an mich adressierte Sendungen überhaupt entgegenzunehmen. (Der Sinn der Sache ist natürlich, einen Boykott zu inszenieren, damit er Kunden verliert und seine Geschäftspraktiken ändert.) Geht das eigentlich, oder kann ich nur jeweils im Einzelfall die Annahme verweigern bzw. benachrichtigte Sendungen nicht abholen? (Daß das wirkt, halte ich für ziemlich plausibel: Was machen Kunden bzw. Käufer, die immerzu vom Paketlogistiker vera...t werden? Denen wird das bald zu dumm, und sie kaufen nicht mehr im Versandhandel. Und da das den Versandhändlern gar nicht gefällt, sind die sehr motiviert, diesen Logistikern Feuer zu machen. - Die Rechtslage ist übrigens nicht zufriedenstellend: Die Bundesnetzagentur hat keine rechtliche Handhabe, gegen solche Praktiken aufsichtlich vorzugehen.) --95.119.97.73 12:38, 11. Feb. 2023 (CET)

Wenn ich das unqualifizierte Geschimpfe raussiebe, willst du also einem bestimmten Paketdienst vorschreiben, an dich adressierte Pakete gar nicht erst zu übernehmen. Nein, das kannst du nicht. Der Verkäufer beauftragt den Paketdienst, du bist nicht Vertragspartner. Du kannst höchstens dem Verkäufer gegenüber deutlich machen, welchen Paketdienst du bevorzugst bzw. ablehnst, ansonsten ist es seine Entscheidung, wen er damit beauftragt, dir das Paket zu bringen, da es ihm zu teuer sein wird, es dir selbst vors Haus zu fahren. --Kreuzschnabel 12:51, 11. Feb. 2023 (CET)
Naja, das wäre doch reine Freundlichkeit gegenüber diesem Drecksl^W Paketdienst, mich in eine Sperrliste einzutragen und es gar nicht erst zu versuchen - erspart ihm den Rücktransport. (Technisch vermutlich gar kein Problem: Kommerzielle Versender beauftragen üblicherweise online, und genau an dieser Stelle kann der Computer natürlich "Möööp" machen und dem Versender "Nicht möglich" oder sowas signalisieren.) Wobei sich die Geschäftsleitung nach dem einen oder anderen Nicht-Annahme-Wunsch wahrscheinlich doch mal überlegt, ob es vielleicht gewisse Verbesserungsmöglichkeiten im operativen Sektor geben könnte. Spätestens dann, wenn der eine oder andere größere Versender andeutet, über eine weitere Zusammenarbeit neu nachdenken zu wollen. (Wie geht übrigens qualifiziertes Geschimpfe?) --95.119.97.73 16:01, 11. Feb. 2023 (CET)

Ja, das ist schon ein Problem. Pakete landen auch manchmal auf der Terrasse oder auf dem Balkon, die man dann durch Zufall findet. Aber das nächste Problem ist: Nimmt ein anderer die Sendung (freundlicherweise...) an, will sie an den angegebenen Empfänger weitergeben, und der sagt: "Was soll ich damit, habe ich nicht bestellt." Wer muß für den Rücktransport sorgen?? Und das dritte Problem ist: Der Fahrer steht verkehrswidrig auf dem Radweg und/oder Fußweg!--Hopman44 (Diskussion) 12:54, 11. Feb. 2023 (CET)

"Wer muß für den Rücktransport sorgen??" Unverlangt zugesandte Waren muss man nicht zurückschicken. --Digamma (Diskussion) 13:10, 11. Feb. 2023 (CET)
Die Zusteller haben im Durchschnitt ca. drei Minuten Zeit, um ein Paket zuzustellen. Da können sie nicht für jedes einzelne Paket einen Regentanz aufführen. Wenn die mal längere Zeit im Stau stehen, heißt das gleich mal unbezahlte Überstunden. Mir scheint auch, dass bevorzugt Ausländer für diesen Job eingestellt werden, wahrscheinlich weil die sich weniger über schlechte Arbeitsbedingungen beschweren. Jedenfalls sind Zusteller mit deutschen Sprachkenntnissen gefühlt deutlich in der Minderheit.
Die meisten Versandhändler verraten ja, welchen Logistiker sie beauftragen. Dann muss man eben einen anderen Versandhandel wählen. Oder gleich an eine Packstation liefern lassen. Oder das bedrängte lokale Ladengeschäft unterstützen. --Optimum (Diskussion) 15:20, 11. Feb. 2023 (CET)
Mich als Käufer interessiert das einen Sch... wie viel Minuten die Zusteller haben. Ich möchte nicht ein 300€-Paket in den Hof geworfen bekommen. --2001:9E8:A533:5B00:4C42:EA55:E74B:53E2 15:38, 11. Feb. 2023 (CET)
Und mich als Empfänger interessieren die Zeitvorgaben der Sklavenhalter für ihre Knechte auch nicht, und zwar deswegen, weil das früher alles anständig funktionierte und die Paketzusteller der Post - andere gab es nicht - mit ihrem Beruf auch ganz zufrieden waren und es eigentlich immer irgendwie möglich machten, daß die Sendungen auch ankamen. (Vor ein paar Jahren hatte ich es mal, daß ich nach Hause kam und den Zusteller erwischte, als er mir gerade eine Benachrichtigung in den Briefkasten steckte. Erfreut sagte ich ihm, das wäre jetzt gar nicht nötig gewesen, er könnte mir die Sendung gleich geben. Der Drecksack hatte die überhaupt nicht auf seiner Karre, sondern stellte an dem Tag praktisch nur Benachrichtigungen zu...) Ich bin tatsächlich der Ansicht, daß das auch ein regulatorisches Problem ist. Es gibt zunehmend mehr Alte und Behinderte, denen das halt nicht zuzumuten ist, irgendwelche benachrichtigten Sendungen dann jwd selbst abholen zu müssen. Folglich gehört das schlicht und einfach verboten. Und es geht doch nun auch anders, z. B. Abgabe bei Nachbarn. (Während der Pandemie wurden auch gar keine Unterschriften - dieses Krickelkrackel auf diesen Handgeräten ist ohnehin nur sinnloser Unfug - eingesammelt, sondern die Pakete einfach im Hausflur abgestellt; überhaupt kein Problem in unserer Wohngegend, da kommt nichts weg. Ich hatte es auch öfter mal, daß ein Zusteller bei mir anschellte und fragte, ob er bei mir eine Sendung für einen Nachbarn abgeben könnte. Früher habe ich die auch angenommen, heute sage ich denen, sie sollen sie demjenigen vor die Wohnungstüre stellen; schließlich würde ich das selbst nämlich auch machen. Vermeidet auch Pannen: ich bekam mal über einen Spezialversand eine Kühlsendung. Nach zwei Tagen schellte dann der Nachbar an und fragte, wann ich denn mal endlich mein Paket bei ihm abholen wollte. Nur wußte ich von nichts, denn ich hatte keine Benachrichtigung im Kasten...) Was auch immer gut kommt: Klebezettelchen am Klingelbrett. Die Haustür sehe ich aber gefühlt bloß alle fünf Jahre von außen, weil ich das Haus nämlich normalerweise durch die Tiefgarage betrete und verlasse (nein, auch bei Fahrradbenutzung), und in den Briefkasten schaue ich auch immer nur von innen und nicht von außen. Call your congressman? Sollte ich vielleicht machen. --95.119.97.73 16:31, 11. Feb. 2023 (CET)
Mag sein, dass es euch nicht interessiert. Sowas passiert eben, wenn alle einen bestimmten Service haben wollen, dann aber doch bei den günstigsten Anbietern bestellen. Mit der Bundespost kann man das nicht vergleichen, denn die arbeitete hochdefizitär. Die fehlenden Milliarden wurden dann aus dem Staatshaushalt ausgeglichen. --Optimum (Diskussion) 15:11, 12. Feb. 2023 (CET)
Hochdefizitär war diese Sparte, aber damals gehörte Telefonie ja noch zur selben Post, die das locker wieder ausglich, wimre. --77.6.88.228 21:18, 18. Feb. 2023 (CET)
Das Krickelkrackel auf diesen Handgeräten ist durchaus sinnvoll: Ich habe mal ein nicht erhaltenes Paket reklamiert und bekam als Antwort einen Zustellbeleg mit einem Ausdruck des Krickelkrackels. Dieses habe ich dann als Faelschung abgelehnt (weder von mir noch von meiner Frau) und daraufhin wurde beim Zusteller ermittelt. Der gab dann zu, das Krickelkrackel auf seinem Handgerät selbst angebracht zu haben, nachdem er das Paket hinter die Gartentuer unter einen Unterstand geworfen habe. Und genau dort lag es auch dann nach zwei oder drei Wochen immer noch ungesehen, weil er keinen Zettel in den Briefkasten geworfen hatte. -- Juergen 134.255.192.126 01:25, 13. Feb. 2023 (CET)

Da die eigentliche Frage ja schon beantwortet ist (Nein, geht nicht, der Versender ist Vertragspartner, nicht der Empfänger) kann man das doch im Alltag pragmatisch lösen. Bei quasi jedem Versand den ich in Auftrag gebe, steht vorher welcher Versender gewählt wird. Also entsprechend wählen, oder - wenn kein angemessener zur Verfügung steht - den Versender freundlich drauf aufmerksam machen, warum man nicht bestellt. -- southpark 18:04, 11. Feb. 2023 (CET)

Wenn man keine Pakete annehmen möchte, sollte man einfach keine bestellen. Ich halte das bei der Frage beschriebene Verhalten für das Geschäft von Paketdienstleistern für ziemlich üblich. --BlackEyedLion (Diskussion) 19:52, 11. Feb. 2023 (CET)
Das geht eben nicht so allgemein, weil man es oft vorher eben nicht feststellen kann bzw. der Absender, z. B. bei einem Versand aus dem Ausland, es auch nicht weiß, welcher Logistiker letztlich ausliefern wird, denn sein nationaler Logistiker ist ja nicht unbedingt auch im Zielland tätig, sondern beauftragt, von außen nur relativ schwer durchschaubar, einen Kooperationspartner. Und dann gibt es eben noch die nicht beauftragten Sendungen - Geschenke von lieben Mitmenschen, oder man kriegt eine Mitteilung "Wir haben Ihre Reklamation/Anregung bzgl. XYZ dankend erhalten und senden Ihnen in den nächsten Tagen als kleine Aufmerksamkeit ein Wdlbrmft an Ihre Adresse zu". Soll man dann etwa "Aber nicht mit UVW, denn von denen nehme ich keine Sendungen mehr an" antworten? Das wäre wohl ein bißchen unrealistisch. (Daß das beschriebene Verhalten üblich ist, ist ja genau das Problem.) --95.119.203.24 13:30, 12. Feb. 2023 (CET)
Ich moechte auch so ein Wdlbrmft. Wo kriegt man das ? -- Juergen 134.255.192.126 01:53, 13. Feb. 2023 (CET)
Gar nicht, man schreibt es nämlich Wrdlbrmpft. Das gibt’s bei Karl Valentin --Kreuzschnabel 17:17, 14. Feb. 2023 (CET)
Danke fuer den Link. Daraus entnehme ich, dass es sich tatsaechlich aber um ein Wrdlbrmpfd (mit d statt t am Ende) handelt, das leider gar kein Geschenk, sondern ein Personenname ist. -- Juergen 195.192.220.27 01:54, 16. Feb. 2023 (CET)
Da wird's ja noch abgehobener. Jemand will dir ein (Werbe-)Geschenk oder eine Entschädigungs-Aufmerksamkeit schicken und in dem Moment in dem er es abschickt, soll ihm sein bevorzugter Paketdienstleister sagen "Nö, dem dürfen wir nicht liefern"? Da dürfte es dem Geschenkversender schnell zu blöd werden. Und du bestellst im Ausland, dein Paket wird erfolgreich eingeflogen, und bei der Übergabe zum deutschen Dienstleister wird festgestellt, dass der das Paket nicht weiterleiten darf? Wer soll das nun regeln? Der ausländische Logistiker hat kein Interesse daran, wegen einem Paket seine Partner zu wechseln, der ausländische Versender hat kein Interesse daran, das längst erfolgreich abgeschickte und bezahlte Paket nochmal für den Inlandsversand neu zu vergeben, niemand hat die Prozesse dafür.
Aber es gibt doch Möglichkeiten. Neben den schon genannten Paketboxen lassen sich die Logistiker gerne generelle Abstell- oder Abgabevollmachten erteilen, mit denen du sie anweist, deine Pakete immer an einem bestimmten Ort oder bei einem bestimmten Nachbarn abzugeben. Oder es gibt entsprechende Online-Services, mit denen sich das sendungsweise steuern lässt. Manche Arbeitgeber lassen zu, dass du Pakete zu ihnen bestellst, wo die Poststelle oder der Empfang immer besetzt ist, oder vielleicht macht das auch das Cafe um die Ecke. Usw. --Karotte Zwo (Diskussion) 18:03, 14. Feb. 2023 (CET)
@Optimum: Unbezahlte Überstunden? In einem Land in dem es den Mindestlohn gibt? In einem Land das, gemäß EUGH-Urteil, verpflichtet ist darauf zu achten, dass die Arbeitszeiten der Arbeitnehmer korrekt erfasst werden? Le Duc de Deux-Ponts (Diskussion) 12:22, 15. Feb. 2023 (CET)

Wer kann mir zwei Bilder aus einer Flash-Anwendung befreien?

Ich brauche aus http://www.manuscripta-mediaevalia.de/dokumente/html/obj31275356 die beiden Bilder aus Abschnitt 10.2 über Dracula (groß). Ich hatte wohl Anfang 2021 hier nach Möglichkeiten gefragt, finde aber den Abschnitt mit der Archivsuche nicht. Vgl. auch https://archivalia.hypotheses.org/129104. Danke! --Historiograf (Diskussion) 21:02, 11. Feb. 2023 (CET)

Keine Ahnung. Kann man heute noch Flash-Inhalte aufrufen? --Ralf Roletschek (Diskussion) 22:11, 11. Feb. 2023 (CET)
Offensichtlich ja. Bei Firefox lädt nichts, die Fläche bleibt weiß und bei Edge erscheint „This plug-in isn't supported“. Wenn man auf Inspect klickt, taucht im Quelltext <object … type="application/x-shockwave-flash" codebase="http://active.macromedia.com/flash5/cabs/swflash.cab#version=6,0,0,0" …> auf, also eine nicht umgestellte Flash-Anwendung. --Rôtkæppchen₆₈ 22:50, 11. Feb. 2023 (CET)
Setze ein historisches Betriebssystem (z.B. Windows 98, Windows 2000 oder Windows XP) in einer virtualisierten Umgebung (z.B. qemu oder virtualbox auf, und versuche die swf-Datei im dort normalerweise vorinstallierten alten Browser („Internet Explorer”) abzuspielen. Dann kannst Du wahrscheinlich einfach einen Screenshot von den fraglichen Bildern im Großformat machen. (Das nötige flash-plugin findest Du noch bei Archive.org) --92.72.222.159 09:24, 12. Feb. 2023 (CET)
Noch eine Idee: Eine ältere Knoppix-Version verwenden, die Flash noch unterstützt. Ich weiß aber nicht welche das wäre und müsste erst einmal rumprobieren mit Laptop und Knoppix-CDs. --Rôtkæppchen₆₈ 12:58, 12. Feb. 2023 (CET) Ich hab das mit Knoppix 6.2.1, 7.6 und 9 ausprobiert: Hat nicht geklappt. --Rôtkæppchen₆₈ 13:45, 12. Feb. 2023 (CET)
Vielleicht klappt das mit einem Add-On für firefox (habe keine Erfahrung damit).
--KleinerKorrektor (Diskussion) 14:50, 13. Feb. 2023 (CET)

Ich kann leider keinerlei erforderliches technisches Wissen beisteuern und brauche daher jemand, der das für mich erledigt. —-Historiograf (Diskussion) 13:20, 14. Feb. 2023 (CET)

@Historiograf: eines der Add-Ons bekommst du doch aber bestimmt installiert oder benutzt du einen anderen Webbrowser als Firefox? Wenn du damit dann die Flashanimation anschaust würde ich einfach bei den gewünschten Stellen ein Bildschirmfoto machen, so wie die IP es oben bereits vorgeschlagen hat. Du kannst bestimmt auch ein Tastenkürzel verwenden für das erstellen eines Bildschirmfotos, so dass du schnell mehrere hintereinander machen kannst. Benutzt du Windows? Viele Grüße --KleinerKorrektor (Diskussion) 15:52, 14. Feb. 2023 (CET)

Ich nutze Chrome, habe aber auch Firefox und würde trotz meines hohen Alters (werde nächste Woche 65) auch einen Screenshot hinbekommen, wenn ich denn die Bilder (keine Animation!) groß sehen würde. "621 Erweiterungen für „flash“ gefunden" - ist das dein Ernst?

  • Flash Player - kein Erfolg
  • ModernKit.one - kein Erfolg

Na dann sind es nur noch 619, die ich ausprobieren muss .... Danke, Danke, Danke für deine übergroße Mühe, mir so zu helfen --Historiograf (Diskussion) 00:30, 15. Feb. 2023 (CET)

Es tut mir schrecklich Leid, das nächste Mal werde ich meine Lebenszeit, die ich dir hier kostenlos in Form einer Recherche, die eigentlich jeder selber innerhalb weniger Sekunden durchführen kann, so wie des schreibens von Beiträgen hier, so aufwenden, dass es hochwohlgeboren genehm ist.
Unfassbar manche Leute...
--KleinerKorrektor (Diskussion) 11:15, 15. Feb. 2023 (CET)
Wenn es nur wenige Sekunden Aufwand waren, verstehe ich dein Gewese nicht. --77.6.88.228 21:22, 18. Feb. 2023 (CET)

Bitte frage Irfan! [20] --Sockenschütze (Diskussion) 07:43, 16. Feb. 2023 (CET)

Irfan antwortet nicht! Was ist denn das für ein seltsamer Hinweis? Ohne eine Erläuterung ist das komplett sinnlos. --92.76.39.46 15:41, 18. Feb. 2023 (CET)
So schwer ist das nicht zu verstehen. Übersetzung: Für Ifanview gibt es ein Plugin, das alte Flashdateien lesen kann. --92.72.218.192 15:58, 18. Feb. 2023 (CET)

@Historiograf: Das sind die Bilder von Abschnitt 10.2: Nr. 1 und Nr. 2. --92.76.39.46 15:41, 18. Feb. 2023 (CET)

Vielen Dank --Historiograf (Diskussion) 18:58, 20. Feb. 2023 (CET)